• Bihar Board

SRM University

Cbse board result 2024.

  • CBSE 10th Result 2024
  • CBSE 12th Result 2024
  • JAC Board Result 2024
  • CG Board Result 2024
  • UK Board Result 2024
  • Karnataka Board Result 2024
  • Shiv Khera Special
  • Education News
  • Web Stories
  • Current Affairs
  • नए भारत का नया उत्तर प्रदेश
  • School & Boards
  • College Admission
  • Govt Jobs Alert & Prep
  • GK & Aptitude
  • CBSE Class 10 Study Material

CBSE Class 10 English Case Based Factual Passages with Answers for Term 2 Exam 2022

Cbse class 10 english case based factual passages (unseen passages) are provided here with answers. practice these unseen passages with answers to prepare for the reading section and score good marks in cbse class 10 english term 2 exam 2022..

Gurmeet Kaur

In CBSE Class 10 Term 2 Exam 2022, the English question paper will have a question based on a factual passage (unseen passage) of 5 marks. Students can easily score full marks in this question with a little practice and understanding of the question. To help students get the required dose of practice, we have provided below the ‘Case Based Factual Passages’ for the term 2 exam. These passages have been suggested by experienced teachers. These passages are provided with answers. You can download all the factual passages with answers in PDF from the link provided below in the article.

New!  CBSE Class 10 English Writing Section Formats, Questions and Marking Scheme for Term 2 Exam 2022

CBSE Class 10 English Discursive Comprehension Passages for Term 2 Exam 2022

Read the following passage carefully:

1. India is home to an estimated 10% of the global Snow Leopard population spread across five Himalayan states of Jammu and Kashmir, Himachal Pradesh, Uttarakhand, Sikkim and Arunachal Pradesh which is only 5% of the available global habitat for the animal.

2. Project Snow Leopard was introduced in India by the Project Snow Leopard Committee instituted by the Ministry of Environment and Forests, Government of India, in the year 2009 to safeguard and conserve India‘s unique natural heritage of high-altitude wildlife populations and their habitats by promoting conservation through participatory policies and actions.

CBSE Class 10 English Discursive Passages with Answers for Term 2 Exam 2022

3. Project Tiger and Project Elephant are prevalent in India with the former introduced in 1973 garnering worldwide attention.

4. The Snow Leopard usually occurs at an altitude of >2700m above sea level and is the apex predator in the Himalayan Region along with the Tibetan Wolf. The project is not only to protect the Snow Leopard, but as with Project Tiger, it is considered an umbrella species and with its protection, comes the protection of other species and the habitat which they are all a part of.

Top 3 National Parks in India where the Snow Leopard can likely be seen

Kibber Wildlife Sanctuary – Himachal Pradesh

(ii) Ulley Valley – Ladakh

(iii) Hemis National Park – Himachal Pradesh

case study passage class 10 english

The reasons for introduction of Project Snow Leopard are as follows

i) High altitudes of India > 3000m including the Himalayan & Trans-Himalayan biogeographic zones support a unique wildlife assemblage of global conservation.

ii) There has been relatively less attention on the region from the viewpoint of wildlife conservation.

iii) The region represents a vast rangeland system supporting important traditional pastoral economic and lifestyles.

iv) The region provides essential ecosystem services and harbours river systems vital for the nation‘s food security.

v) India has ratified international agreements promoting the conservation of high-altitude wildlife species such as the snow leopard.

vi) The region is important for the country‘s national security as well as international relations.

vii) The high-altitude wildlife in India today faces a variety of threats.

viii) The existing high altitude protected areas in India require considerable strengthening.

ix) Wildlife Management in the region needs to be made participatory.

Based on your understanding of the passage, answer ANY FIVE questions from the six given below.

1.What is the purpose of the above passage/study?

2.What do you mean by the phrase- "snow leopard is considered an umbrella species?"

3.According to the above passage why high-altitude regions should be protected?

4.According to the above passage what is integral to the nation‘s food security?

5.Which Indian states come under the Himalayan region?

6.List out two important factors why the high-altitude region is important.

1.The purpose of the passage is to show that snow leopards are vulnerable species and must be protected.

2.'Umbrella' species refers to such species whose protection will ensure the protection of other species. Hence, option (a).

3.High altitude regions should be protected as they support unique wildlife of global significance.

4.River systems are integral to the nation‘s food security.

5.Jammu and Kashmir, Himachal Pradesh, Uttarakhand, Sikkim and Arunachal Pradesh

6.Important for the country‘s national security as well as international relations and the high-altitude wildlife in India today faces a variety of threats.

CBSE Class 10 English Practice Paper for Term 2 Exam 2022 (Based on CBSE Sample Paper)

Get here latest School , CBSE and Govt Jobs notification in English and Hindi for Sarkari Naukari and Sarkari Result . Download the Jagran Josh Sarkari Naukri App . Check  Board Result 2024  for Class 10 and Class 12 like  CBSE Board Result ,  UP Board Result ,  Bihar Board Result ,  MP Board Result ,  Rajasthan Board Result  and Other States Boards.

  • SSC GD Result 2024
  • WBBSE Result 2024
  • wbbse.wb.gov.in Result 2024
  • WB Board 10th Result 2024 Jagran Josh
  • WB Madhyamik Toppers 2024
  • wbresults.nic.in Result 2024
  • West Bengal Madhyamik Supplementary Exam 2024
  • CGBSE Result 2024
  • AP SSC Results 2024 Manabadi by Jagran Josh
  • AP SSC Topper List 2024
  • CBSE Study Material
  • CBSE Class 10

Latest Education News

Who Won Yesterday IPL Match: KKR vs MI, Match 51, Check All Details and Latest Points Table

Picture Puzzle IQ Test: Find the mistake in the stack of shoes in 5 seconds!

Brain Teaser: Can You Guess the Vegetable? This Emoji Puzzle is Stumping 99%

MH CET 3-year LLB Result Declared At cetcell.mahacet.org, Check Steps To Download Scorecard

WBBME Madhyamik Result 2024 Announced for HM, Alim, Fazil at wbbme.org, Get Madrasah Board Result Updates

CBSE 10th Result 2024 Releasing After May 20, Check Latest News and Updates Here

India T20 World Cup Squad 2024: Complete List of Team Players and Name

SSC GD Result 2024 Live: Constable Results on ssc.gov.in; Check Expected Cut Off, Merit List Date, Marking Scheme

Indian Navy Agniveer Recruitment 2024: Apply Online for MR SSR Posts at joinindiannavy.gov.in

HBSE Class 11 Latest Physical Education Syllabus 2024-25: Download Free PDF Here With Question Paper Design

Current Affairs Hindi One Liners: 03 मई 2024- टी20 विश्व कप के ब्रांड एम्बेसडर

Current Affairs One Liners: 03 May 2024- ICC T20 World Cup 2024

Current Affairs Quiz: 03 May 2024- World Press Freedom Day

Optical Illusion Eye Test: Find the cat in the backyard in 7 seconds!

UPSC ESE Mains Question Paper: PDF Download

CBSE Allows Class 11 Students to Opt for Mathematics, Who Passed 10th With Basic Maths

[LIVE] CBSE Result 2024 Date: OFFICIAL Confirmation! 10th and 12th Results after 20 May at cbseresults.nic.in

HS Result 2024 Assam Releasing Shortly, Download AHSEC Mark Sheet at ahsec.assam.gov.in, Get Latest Updates

Picture Puzzle IQ Test: You Are Highly Attentive If You Can Spot A Hat In This Bathroom In 8 Seconds!

Pondicherry University Result 2024 Released on pondiuni.edu.in; Direct Link to Download UG and PG Marksheet

Case Based Factual Passage for Class 10

Table of Contents

In this post, I’m sharing with you a “ Case Based Factual Passage for Class 10 ” that’s often found in the CBSE Class 10th Board Exam. If you want to do well in English, you should practise this part a lot. It’s crucial for the upcoming CBSE board exam in 2023-24. Writing skills is very important for board exams.

Join Our Telegram Group

  • Subject verb Agreement mcqs
  • Analytical Paragraph Format Examples

Read the passage given below.

1. There are innumerable villages in India which have fewer than 500 residents. Many people from the other cities in India head to one of these sanctuaries to wash away all the hardships of an ultramodern life.

2. With a population of just 289, the tribal village of Ha is the smallest named place in the country which is located at an altitude of 4,780 ft. in the Kurung Kumey district of Arunachal Pradesh. The village is known for its brilliant natural beauty and the Menga caves which are dedicated to Lord Shiva.

3. Located about 27 km from Keylong in Himachal Pradesh, the village of Shansha has a population of 320 people in 72 houses. The hamlet is located close to the Tandi Kishtwar road which is generally a pit stop for those adventure-seeking individuals who ride through this dangerous route. The hamlet of Shansha is home to a greenhouse which is located at a height of 10,000 ft. and provides aid to the locals living and working in this highly challenging terrain.

Factual Passage for Class 10

4. One might think the entire state of Goa is filled with a bunch of beaches and shacks, but there is another side of the tiny state which is known for its isolated villages. Xelpem, also known as Shelpem, is an example of one such place which is located on the banks of Salaulim river and has a population of just 255.

5. The village of Sankri in Uttarakhand is the base camp of many treks and is well known for being the last village which has a market on the way to the Har ki Dun Trek, Kedarkantha. The hamlet has just 77 homes out of which three are small guest houses and has a population of about 270 people. Most families welcome the trekkers to stay at their homes as well.

6. One amongst the most scenic and hidden gems of Nagaland is the village of Nitoi. The village has a population of about 402 people and boasts of an 80% literacy rate even in its small population. The small hamlet can be reached from Kiphire which is an 8-hour drive from Kohima.

  • Important Passage for 10 CBSE Board

Case Based Factual Passage for Class 10 mcq

On the basis of your reading of the passage given above, answer the following questions.

(1) The purpose of this passage is to provide awareness regarding the _____. Choose the correct option. (a) tiny towns and cities in India (b) small villages and hamlets in India (c) small hamlets and cities in India (d) isolated tribes in India

(2) Select the option that is true for the two statements given below. (i) Shansha has a population of 320 people in 72 houses. (ii) The hamlet of Shansha is home to a greenhouse. (a) (i) contradicts (ii). (b) (ii) is the reason for (i). (c) (i) is independent of (ii). (d) (ii) is the result of (i).

(3) Select the option that gives the correct meaning of the following statement. “Most families welcome the trekkers to stay at their homes as well.” (a) The hamlet consists of only guest houses for trekkers. (b) People from Sankri village aren’t fond of any visitors. (c) The hamlet has just 77 homes with a population of about 270 people. (d) People from Sankri village keep their doors open for guests.

(4) According to the passage, the greenhouse in Shansha is essential for the people because______. (a) it provides aid to the trekkers living and working in this highly challenging terrain. (b) it is the only source of food in the mountains. (c) it provides aid to the locals at a height of 10,000 ft. in this highly challenging terrain. (d) it is a major tourist attraction of that village.

(5) Select the option listing what the given sentence refers to. ‘The village has a population of about 402 people and boasts of an 80% literacy rate even in its small population.’ (i) The village has a population of 402 literate people. (ii) The population of the village is 80% illiterate. (iii) The 80% of the population is not illiterate. (iv) The population of the village is limited to 402 individuals. (v) The village boasts high literacy rate even in its small population. (a) (i), (ii) and (v) (b) (ii), (iv) and (v) (c) (i), (ii) and (iii) (d) (iii), (iv) and (v)

(6) The Menga caves from the village of Ha in Arunachal Pradesh are dedicated to which deity? (a) Lord Shiva (b) Lord Krishna (c) Lord Ganesha (d) Lord Vishnu

  • Unseen Passage for Class 10

Factual Passage for Class 10 with Answers

(7) This passage lists an example of the tiny villages in India where _______. Select the correct option. (a) people come for good business opportunities (b) people come to wash away all their sins in the holy pilgrimage sites (c) people come to escape all the hardships of an ultra-modern life (d) people have just discovered their existence and are not yet listed on maps

(8) Choose the correct option to answer the following: According to paragraph 4, ‘One might think the entire state of Goa is filled with a bunch of beaches and shacks.’ This is so because: (a) Goa is famous for its small naturally scenic villages. (b) Goa is generally advertised as such in popular media. (c) not much is known about the beaches and shacks of Goa. (d) people don’t care about the natural diversity in Goa.

(9) The tribal village of Ha, the smallest named place in the country, is located at an altitude of…… (a) 4,280 ft. (b) 4,580 ft. (c) 4,780 ft. (d) 4,980 ft.

(10) The village of Nitoi is described as_______ (a) scenic and hidden gem of Himachal. (b) filled with a bunch of beaches and shacks. (c) a pit stop for adventure-seeking individuals. (d) scenic and hidden gem of Nagaland.

(11) Which word in the passage means ‘distress’? (a) dangerous (b) hardships (c) innumerable (d) isolated

(12) Which word in the passage is opposite to the meaning of ‘unstimulating’? (a) working (b) literacy (c) sanctuaries (d) challenging

Factual Passage with Answer & Explanation

1. Ans. (b) small villages and hamlets in India 2. Ans. (c) (1) is independent of (2). 3. Ans. (d) People from Sankri village keep their doors open for guests. 4. Ans. (c) it provides aid to the locals at a height of 10,000 ft. in this highly challenging terrain. 5. Ans. (d) (3), (4) and (5) 6. Ans. (a) Lord Shiva 7. Ans. (c) people come to escape all the hardships of an ultra-modern life 8. Ans. (b) Goa is generally advertised as such in popular media. 9. Ans. (c) 4,780 ft. 10. Ans. (d) scenic and hidden gem of Nagaland. 11. Ans. (b) hardships 12. Ans. (d) challenging

English for CBSE Board Exams

Sentence Reordering For Classes 9 & 10

Discursive Passage For CBSE Class 10th Board

A Letter to God Previous Year Questions

Letter to Editor Showing Concern

Discursive Passage Class 10 Board

Editing Exercise with Answer

How useful was this post?

Click on a star to rate it!

Average rating 5 / 5. Vote count: 3

No votes so far! Be the first to rate this post.

As you found this post useful...

Follow us on social media!

We are sorry that this post was not useful for you!

Let us improve this post!

Tell us how we can improve this post?

Similar Posts:

Leave a comment cancel reply.

Save my name, email, and website in this browser for the next time I comment.

  • New QB365-SLMS
  • NEET Materials
  • JEE Materials
  • Banking first yr Materials
  • TNPSC Materials
  • DIPLOMA COURSE Materials
  • 5th Standard Materials
  • 12th Standard Materials
  • 11th Standard Materials
  • 10th Standard Materials
  • 9th Standard Materials
  • 8th Standard Materials
  • 7th Standard Materials
  • 6th Standard Materials
  • 12th Standard CBSE Materials
  • 11th Standard CBSE Materials
  • 10th Standard CBSE Materials
  • 9th Standard CBSE Materials
  • 8th Standard CBSE Materials
  • 7th Standard CBSE Materials
  • 6th Standard CBSE Materials
  • Tamilnadu Stateboard
  • Scholarship Exams
  • Scholarships

case study passage class 10 english

CBSE 10th Standard English Subject Case Study Questions with Solutions

By QB365 on 20 May, 2021

QB365 Provides the updated CASE Study Questions for Class 10 , and also provide the detail solution for each and every case study questions . Case study questions are latest updated question pattern from NCERT, QB365 will helps to get  more marks in Exams

Related 10th Standard CBSE English Materials

10th standard cbse syllabus & materials, cbse 10th maths probability chapter case study question with answers, cbse 10th maths statistics chapter case study question with answers, cbse 10th maths surface areas and volumes chapter case study question with answers, cbse 10th maths areas related to circles chapter case study question with answers, cbse 10th maths circles chapter case study question with answers, cbse 10th maths some applications of trigonometry chapter case study question with answers, cbse 10th maths introduction to trigonometry chapter case study question with answers, cbse 10th maths coordinate geometry chapter case study question with answers, cbse 10th maths triangles chapter case study question with answers, cbse 10th maths arithmetic progressions chapter case study questions with answers, cbse 10th maths quadratic equations chapter case study questions with answers, cbse 10th social science the making of a global world chapter case study question with answers, cbse 10th social science nationalism in india chapter case study question with answers, cbse 10th social science the rise of nationalism in europe chapter case study question with answers, cbse 10th science metals and non metals chapter case study question with answers.

cbse logo

Class VI to XII

Tn state board / cbse, 3000+ q&a's per subject, score high marks.

cbse students

10th Standard CBSE Study Materials

case study passage class 10 english

10th Standard CBSE Subjects

  • CBSE Class 10
  • Unseen Passage for Class 10

CBSE Unseen Passage for Class 10 with Answers

Unseen passage for Class 10 is an important part of the CBSE English exam. Students who aim to score high marks in English must practise different types of unseen passages for Class 10. Doing so will help them to fetch all 20 marks from the reading section of the English Language and Literature subject. Moreover, CBSE Class 10 students will get a thorough study and understanding of the comprehension passages. They can easily tackle the questions of unseen passages in the exam. Therefore, to boost their exam preparation, we have provided the CBSE Unseen Passage for Class 10 with answers pdf.

Unseen Passage for Class 10 English Reading Skills

Before going into the details of the unseen passage for Class 10, let’s look at the marks weightage of the comprehension passage. As per the CBSE Class 10 English syllabus , the English unseen passage for Class 10 is asked under the reading section.

Two unseen passages are asked in the Class 10 CBSE English board exam as mentioned below:

Reading: 20 marks.

1) Multiple Choice Questions based on a Discursive passage of 400-450 words to test inference, evaluation and vocabulary. Ten out of twelve questions are to be answered in English unseen passage for the Class 10 exam. (10×1=10)

2) Multiple Choice Questions based on a Case-based factual passage (with visual input statistical data, chart, etc.) of 300-350 words to test analysis and interpretation. Ten out of twelve questions are to be answered. (10×1=10)

The total length of the two passages is around 700-750 words.

Unseen Passage for Class 10 – Tips to Handle Reading Comprehension

Below, we have provided some tips to answer the comprehension passage questions of Class 10 during the exam.

  • Go through the entire passage thoroughly.
  • Scan the passage quickly and try to find the main idea and theme on which the comprehension passage is based.
  • Underline the important information in the passage.
  • Now, read the question and go back to the unseen passage to find the answer.
  • Avoid copy-pasting the answer from the comprehension passage. Try to frame the answer in your own words.
  • To answer the vocabulary-based questions, like synonyms, antonyms, phrases, idioms etc., replace the word with the meaning. If the meaning is the same, then the answer is correct.
  • To answer multiple choice questions, go through all the options and then mark the correct answer.

CBSE Class 10 Unseen Passage and Reading Comprehension with Answers

Students can find different types of unseen passages for Class 10 with answers pdf for board exam preparation. At the end of the questions, we have also provided the answers to the unseen passages for students’ reference. Firstly, they should try to solve the questions of reading comprehension by themselves. Then upon completion of one passage, they must cross-check their answers with the one provided by us. In this way, they can evaluate their performance. Students who take more time in solving the unseen passages for Class 10 must put on a clock watch to monitor the time spent. Doing so will help them learn time management to solve the questions of unseen passages for Class 10 fast.

CBSE Class 10 English Discursive Passage

1. Read the below given English unseen passage for the Class 10 exam.

The choices we make on a daily basis—wearing a seatbelt, lifting heavy objects correctly or purposely staying out of any dangerous situation—can either ensure our safety or bring about potentially harmful circumstances.

You and I need to make a decision that we are going to get our lives in order. Exercising self-control, self-discipline and establishing boundaries and borders in our lives are some of the most important things we can do. A life without discipline is one that’s filled with carelessness.

We can think it’s kind of exciting to live life on the edge. We like the image of “Yeah! That’s me! Living on the edge! Woo-hoo!” It’s become a popular way to look at life. But if you see, even highways have lines, which provide margins for our safety while we’re driving. If we go over one side, we’ll go into the ditch. If we cross over the line in the middle, we could get killed. And we like those lines because they help to keep us safe. Sometimes we don’t even realize how lines help to keep us safe.

I’m not proud of this, but for the first 20 years of my life at work, I ignored my limits. I felt horrible, physically, most of the time. I used to tell myself, “I know I have limits and that I’ve reached them, but I’m going to ignore them and see if or how long I can get by with it.” I ran to doctors, trying to make myself feel better through pills, vitamins, natural stuff and anything I could get my hands on. Some of the doctors would tell me, “It’s just stress.” That just made me mad. I thought stress meant you don’t like what you do or can’t handle life, and I love what I do. But I kept pushing myself, travelling, doing speaking engagements and so on— simply exhausting myself.

Finally, I understood I was living an unsustainable life and needed to make some changes in my outlook and lifestyle.

You and I don’t have to be like everyone else or keep up with anyone else. Each of us needs to be exactly the way we are, and we don’t have to apologize for it. We’re not all alike, and we need to find a comfort zone in which we can enjoy our lives instead of making ourselves sick with an overload of stress and pressure.

On the basis of understanding the passage, answer ANY TEN questions from the twelve that follow. (1×10=10)

Q1) Which of the characteristics are apt about the writer in the following context: “I know I have limits and that I’ve reached them, but I’m going to ignore them and see if or how long I can get by with it.” ?

1. negligent

2. indecisive

3. spontaneous

4. reckless

5. purposeless

Q2) The reason why living on the edge has become popular is because of the

a) constant need for something different.

b) population being much younger.

c) exhausting effort to make changes.

d) strong tendency to stay within our limits.

Q3) The phrase “potentially harmful circumstances” refers to circumstances that can

(a) certainly be dangerous.

(b) be fairly dangerous.

(c) be possibly dangerous.

(d) seldom be dangerous.

Q4). Choose the option that correctly states the two meanings of ‘outlook’, as used in the passage.

1. A person’s evaluation of life

2. A person’s experiences in life

3. A person’s point of view towards life

4. A person’s regrets in life

5. A person’s general attitude to life

a) (1) and (4)

b) (2) and (3)

c) (3) and (5)

d) (4) and (5)

Q5) Choose the option that best captures the central idea of the passage from the given quotes.

1. It’s all about quality of life and finding a happy balance between work and friends.

2. To go beyond is as wrong as to fall short.

3. Life is like riding a bicycle. To keep your balance, you must keep moving.

4. Balance is not something you find. It’s something you create.

a) Option (1)

b) Option (2)

c) Option (3)

d) Option (4)

Q6) The author explains the importance of discipline and boundaries in our lives using the example of

a) road accidents.

b) traffic rules.

c) lines on the highway.

d) safe driving.

Q7) The author attempts to __________________ the readers through this write-up.

b) question

c) offer aid to

d) offer advice to

Q8) What is the message conveyed in the last paragraph of the passage?

a) Love what you do.

b) Love yourself to love others.

c) Be the best version of yourself.

d) Be yourself

Q9) Which of the following will be the most appropriate title for the passage?

a) Much too soon

b) Enough is enough

c) How much is too much?

d) Have enough to do?

Q10) The author uses colloquial words such as “yeah” and “Woo-hoo!”. Which of the following is NOT a colloquial word?

Q11) Select the option that makes the correct use of “unsustainable”, as used in the passage, to fill in the blank space.

a) In the long run, the ______ officials followed emergency procedures.

b) Emergency procedures were _______ by the officials.

c) Officials reported an ________ set of events during the emergency.

d) Officials admit that the emergency system is _______ in the longer run.

Q12) What does the author mean when he says, “to get our lives in order”?

a) To resume our lives.

b) To organize our lives.

c) To rebuild our lives.

d) To control our lives.

Ans 1) (c) 1 and 4

Ans 2) (a) constant need for something different.

Ans 3) (c) be possibly dangerous

Ans 4) (c) (3) and (5)

Ans 5) (d) Option (4)

Ans 6) (c) lines on the highway.

Ans 7) (d) offer advice to

Ans 8) (c) Be yourself

Ans 9) (c) How much is too much?

Ans 10) (d) stress

Ans 11) (d) Officials admit that the emergency system is __________ in the longer run.

Ans 12) (b) To organize our lives.

Unseen Passage for Class 10 English: Case-Based Factual Passage

Read the unseen passage for the Class 10 English exam given below and answer the questions that follow:

Cataract is the major cause of blindness, which is also caused by damage to the cornea. It occurs more often in old age. As one starts growing old, the lens of the eye hardens, loses its transparency and becomes opaque. It obstructs the light rays from entering the eye.

The onset of cataract blurs the vision. Sometimes, the cataract patient sees multiple images instead of a single object image. Because of the gradual development of cataract, the afflicted person loses his/her vision, and the world becomes dark to him/her.

The development of cataract is a complex process. However, the following factors can be attributed to its formation. Cataract generally develops in old age but sometimes, children are born with cataract because of hereditary defect. Eye injuries too, can cause cataracts.

People exposed to sun rays for longer periods develop cataract earlier than others. Researchers opine that the smoke inhaled while smoking carries substances internally damaging the eyes.

Ultraviolet radiation, invisible to the human eye, is linked to skin cancer. The victim loses vision and the world becomes dark to him.

On the basis of your understanding of the above passage, answer any eight of the following questions: 1×8

(a) What is the major cause of blindness?

(c) Why does the lens of the eye become opaque in old age?

(f) How does cataract affect its victim ultimately?

(d) How does one detect cataract in the early stages?

(e) Give any two factors which are responsible for the formation of cataract.

(i) Find the word from para 1 that means ‘that through which light cannot enter’.

(g) Which type of cancer is caused by ultraviolet radiation?

(b) When does cataract generally occur?

(h) How is smoking responsible for the development of cataract?

(a) cataract

(c) lens of the eye hardens and loses its transparency.

(f) may lead to complete loss of vision

(d) vision becomes blurred/sees multiple images in place of one. (anyone)

(e) eye injury / long exposure to sun rays / hereditary factors (any two)

(g) skin cancer

(b) old age

(h) smoke when inhaled, carries a substance that damages the eye and causes internal harm to the eye.

Students can also practise the unseen passages questions by solving the CBSE Class 10 English Previous Year Papers .

Unseen Passage for Class 10 with Answers PDF

Read the passage given below:

Then all the windows of the grey wooden house (Miss Hilton used to live here. She expired last week) were thrown open, a thing I had never seen before.

At the end of the day, a sign was nailed on the mango tree: FOR SALE.

Nobody in the street knew Miss Hilton. While she lived, her front gate was always locked, and no one ever saw her leave or saw anybody go in. So, even if you wanted to, you couldn’t feel sorry and say that you missed Miss Hilton.

When I think of her house, I see just two colours. Grey and green. The green of the mango tree, the grey of the house and the grey of the high iron fence prevented you from getting at the mangoes.

If your cricket ball fell in Miss Hilton’s courtyard, you never got it back. It wasn’t the mango season when Miss Hilton died. But we got back about ten or twelve of our cricket balls.

The house was sold, and we were prepared to dislike the new owners even before they came. I think we were a little worried. Already we had one resident on the street who kept on complaining about us to our parents. He complained that we played cricket on the pavement, and if we were not playing cricket, he complained that we were making too much noise anyway.

One afternoon when I came back from school Pal said, ‘‘Is a man and a woman. She pretty, but he ugly like hell.’’ I didn’t see much. The front gate was open, but the windows were shut again. I heard a dog barking in an angry way.

One thing was settled pretty quickly. Whoever these people were, they would never be the sort of people to complain that we were making noise and disturbing their sleep.

A lot of noise came from the house that night. The radio was going at full volume until midnight, when the radio station closed down. The dog was barking, and the man was shouting. I didn’t hear the woman.

On the basis of your understanding of the above passage, complete the statements that follow: 1×8

(a) Nobody went into Miss Hilton’s house because her front _________.

(b) Her house had only two colours, (i) ______, and (ii) _______.

(c) The high iron fence did not let the boys get __________.

(d) They never got it back if their __________ fell into her courtyard.

(e) The boys were ready to dislike the ____________.

(f) One resident of the street always ___________.

(g) The new owners of Miss Hilton’s house were (i) _______ and (ii) _______.

(h) The man was shouting, the dog was barking, only __________.

a) gate/door was always locked

b) grey, green

c) at the mangoes/mangoes

d) cricket ball/balls

e) new owners even before they came/ new owners

f) complained about them to their parents/kept on complaining

g) pretty woman, ugly man/man and woman

h) the woman has not heard/woman was silent /unheard / no voice of the woman

We hope students must have found this page on CBSE Unseen Passage for Class 10 helpful for their studies. For more study material related to the CBSE Class 10 board exam, keep visiting BYJU’S. Also, download the BYJU’S App for interactive study videos.

case study passage class 10 english

  • Share Share

Register with BYJU'S & Download Free PDFs

Register with byju's & watch live videos.

close

Counselling

Self Studies

  • Andhra Pradesh
  • Chhattisgarh
  • West Bengal
  • Madhya Pradesh
  • Maharashtra
  • Jammu & Kashmir
  • NCERT Books 2022-23
  • NCERT Solutions
  • NCERT Notes
  • NCERT Exemplar Books
  • NCERT Exemplar Solution
  • States UT Book
  • School Kits & Lab Manual
  • NCERT Books 2021-22
  • NCERT Books 2020-21
  • NCERT Book 2019-2020
  • NCERT Book 2015-2016
  • RD Sharma Solution
  • TS Grewal Solution
  • TR Jain Solution
  • Selina Solution
  • Frank Solution
  • ML Aggarwal Solution
  • Lakhmir Singh and Manjit Kaur Solution
  • I.E.Irodov solutions
  • ICSE - Goyal Brothers Park
  • ICSE - Dorothy M. Noronhe
  • Sandeep Garg Textbook Solution
  • Micheal Vaz Solution
  • S.S. Krotov Solution
  • Evergreen Science
  • KC Sinha Solution
  • ICSE - ISC Jayanti Sengupta, Oxford
  • ICSE Focus on History
  • ICSE GeoGraphy Voyage
  • ICSE Hindi Solution
  • ICSE Treasure Trove Solution
  • Thomas & Finney Solution
  • SL Loney Solution
  • SB Mathur Solution
  • P Bahadur Solution
  • Narendra Awasthi Solution
  • MS Chauhan Solution
  • LA Sena Solution
  • Integral Calculus Amit Agarwal Solution
  • IA Maron Solution
  • Hall & Knight Solution
  • Errorless Solution
  • Pradeep's KL Gogia Solution
  • OP Tandon Solutions
  • Sample Papers
  • Previous Year Question Paper
  • Value Based Questions
  • CBSE Syllabus
  • CBSE MCQs PDF
  • Assertion & Reason
  • New Revision Notes
  • Revision Notes
  • HOTS Question
  • Marks Wise Question
  • Toppers Answer Sheets
  • Exam Paper Aalysis
  • Concept Map
  • CBSE Text Book
  • Additional Practice Questions
  • Vocational Book
  • CBSE - Concept
  • KVS NCERT CBSE Worksheets
  • Formula Class Wise
  • Formula Chapter Wise
  • JEE Crash Course
  • JEE Previous Year Paper
  • Important Info
  • JEE Mock Test
  • JEE Sample Papers
  • SRM-JEEE Mock Test
  • VITEEE Mock Test
  • BITSAT Mock Test
  • Manipal Engineering Mock Test
  • AP EAMCET Previous Year Paper
  • COMEDK Previous Year Paper
  • GUJCET Previous Year Paper
  • KCET Previous Year Paper
  • KEAM Previous Year Paper
  • Manipal Previous Year Paper
  • MHT CET Previous Year Paper
  • WBJEE Previous Year Paper
  • AMU Previous Year Paper
  • TS EAMCET Previous Year Paper
  • SRM-JEEE Previous Year Paper
  • VITEEE Previous Year Paper
  • BITSAT Previous Year Paper
  • UPSEE Previous Year Paper
  • CGPET Previous Year Paper
  • CUSAT Previous Year Paper
  • AEEE Previous Year Paper
  • Crash Course
  • Previous Year Paper
  • NCERT Based Short Notes
  • NCERT Based Tests
  • NEET Sample Paper
  • Previous Year Papers
  • Quantitative Aptitude
  • Numerical Aptitude Data Interpretation
  • General Knowledge
  • Mathematics
  • Agriculture
  • Accountancy
  • Business Studies
  • Political science
  • Enviromental Studies
  • Mass Media Communication
  • Teaching Aptitude
  • NAVODAYA VIDYALAYA
  • SAINIK SCHOOL (AISSEE)
  • Mechanical Engineering
  • Electrical Engineering
  • Electronics & Communication Engineering
  • Civil Engineering
  • Computer Science Engineering
  • CBSE Board News
  • Scholarship Olympiad
  • School Admissions
  • Entrance Exams
  • All Board Updates
  • Miscellaneous
  • State Wise Books
  • Engineering Exam

Case Study Class 10 English Questions and Answers (Download PDF)

Free pdf download.

SHARING IS CARING If our Website helped you a little, then kindly spread our voice using Social Networks. Spread our word to your readers, friends, teachers, students & all those close ones who deserve to know what you know now.

Case Study Class 10 English

If you are looking for the CBSE Case Study class 10 English in PDF, then you are in the right place. CBSE 10th Class Case Study for the English Subject is available here on this website. These Case studies can help the students to solve the different types of questions that are based on the case study or passage.

CBSE Board will be asking case study questions based on English subjects in the upcoming board exams. Thus, it becomes an essential resource to study. 

The Case Study Class 10 English Questions cover a wide range of chapters from the subject. Students willing to score good marks in their board exams can use it to practice questions during the exam preparation. The questions are highly interactive and it allows students to use their thoughts and skills to solve the given Case study questions.

Download Class 10 English Case Study Questions and Answers PDF (Passage Based)

Download links of class 10 English Case Study questions and answers pdf is given on this website. Students can download them for free of cost because it is going to help them to practice a variety of questions from the exam perspective.

Case Study questions class 10 English include all chapters wise questions. A few passages are given in the case study PDF of English. Students can download them to read and solve the relevant questions that are given in the passage.

Students are advised to access Case Study questions class 10 English CBSE chapter wise PDF and learn how to easily solve questions. For gaining the basic knowledge students can refer to the NCERT Class 10th Textbooks. After gaining the basic information students can easily solve the Case Study class 10 English questions.

How to Solve Case Study Based Questions Class 10 English?

In order to solve the Case Study Based Questions Class 10 English students are needed to observe or analyse the given information or data. Students willing to solve Case Study Based Questions are required to read the passage carefully and then solve them. 

While solving the class 10 English Case Study questions, the ideal way is to highlight the key information or given data. Because, later it will ease them to write the final answers. 

Case Study class 10 English consists of 4 to 5 questions that should be answered in MCQ manner. While answering the MCQs of Case Study, students are required to read the paragraph as they can get some clue in between related to the topics discussed.

Also, before solving the Case study type questions it is ideal to use the CBSE Syllabus to brush up the previous learnings.

Features Of Class 10 English Case Study Questions And Answers Pdf

Students referring to the Class 10 English Case Study Questions And Answers Pdf from Selfstudys will find these features:-

  • Accurate answers of all the Case-based questions given in the PDF.
  • Case Study class 10 English solutions are prepared by subject experts referring to the CBSE Syllabus of class 10.
  • Free to download in Portable Document Format (PDF) so that students can study without having access to the internet.

Benefits of Using CBSE Class 10 English Case Study Questions and Answers

Since, CBSE Class 10 English Case Study Questions and Answers are prepared by our English experts referring to the CBSE Class 10 English Syllabus , it provided benefits in various way:-

  • Case study class 10 English helps in exam preparation since, CBSE Class 10 Question Papers contain case-based questions.
  • It allows students to utilise their learning to solve real life problems.
  • Solving case study questions class 10 English helps students in developing their observation skills.
  • Those students who solve Case Study Class 10 English on a regular basis become extremely good at answering normal formula based English questions.
  • By using class 10 English Case Study questions and answers pdf, students focus more on Selfstudys instead of wasting their valuable time.
  • With the help of given solutions students learn to solve all Case Study questions class 10 English CBSE chapter wise pdf regardless of its difficulty level.

Manipal MET 2024 Application (Released) (All Pages)

  • NCERT Solutions for Class 12 Maths
  • NCERT Solutions for Class 10 Maths
  • CBSE Syllabus 2023-24
  • Social Media Channels
  • Login Customize Your Notification Preferences

case study passage class 10 english

  • Second click on the toggle icon

case study passage class 10 english

Provide prime members with unlimited access to all study materials in PDF format.

Allow prime members to attempt MCQ tests multiple times to enhance their learning and understanding.

Provide prime users with access to exclusive PDF study materials that are not available to regular users.

case study passage class 10 english

Level Up Academy

CBSE MCQ for Class 10 English Case Based Factual Passage Free PDF

Featured image website

The CBSE MCQ for Class 10 English Case Based Factual Passage  are provided below, in detailed and free to download PDF format. The solutions are latest , comprehensive , confidence inspiring , with easy to understand explanation . To download CBSE MCQ for Class 10 English Grammar PDF for Free, just click ‘ Download pdf ’ button.

CBSE MCQ for Class 10 English Case Based Factual Passage PDF

Checkout our mcqs for other topics.

  • Discursive Passage MCQ Questions
  • Case based Factual passage MCQ Questions
  • Letter to the Editor MCQ Questions
  • Letter of Complaint MCQ Questions
  • Tenses MCQ Questions
  • Subject-Verb Concord MCQ Questions
  • Modals MCQ Questions
  • Determiners MCQ Questions
  • Reported Speech MCQ Questions

How should I study for my upcoming exams?

First, learn to sit for at least 2 hours at a stretch

Solve every question of NCERT by hand, without looking at the solution.

Solve NCERT Exemplar (if available)

Sit through chapter wise FULLY INVIGILATED TESTS

Practice MCQ Questions (Very Important)

Practice Assertion Reason & Case Study Based Questions

Sit through FULLY INVIGILATED TESTS involving MCQs. Assertion reason & Case Study Based Questions

After Completing everything mentioned above, Sit for atleast 6 full syllabus TESTS.

Contact Form

Privacy Policy

  • Class 6 Maths
  • Class 6 Science
  • Class 6 Social Science
  • Class 6 English
  • Class 7 Maths
  • Class 7 Science
  • Class 7 Social Science
  • Class 7 English
  • Class 8 Maths
  • Class 8 Science
  • Class 8 Social Science
  • Class 8 English
  • Class 9 Maths
  • Class 9 Science
  • Class 9 Social Science
  • Class 9 English
  • Class 10 Maths
  • Class 10 Science
  • Class 10 Social Science
  • Class 10 English
  • Class 11 Maths
  • Class 11 Computer Science (Python)
  • Class 11 English
  • Class 12 Maths
  • Class 12 English
  • Class 12 Economics
  • Class 12 Accountancy
  • Class 12 Physics
  • Class 12 Chemistry
  • Class 12 Biology
  • Class 12 Computer Science (Python)
  • Class 12 Physical Education
  • GST and Accounting Course
  • Excel Course
  • Tally Course
  • Finance and CMA Data Course
  • Payroll Course

Interesting

  • Learn English
  • Learn Excel
  • Learn Tally
  • Learn GST (Goods and Services Tax)
  • Learn Accounting and Finance
  • GST Tax Invoice Format
  • Accounts Tax Practical
  • Tally Ledger List
  • GSTR 2A - JSON to Excel

Are you in school ? Do you love Teachoo?

We would love to talk to you! Please fill this form so that we can contact you

Case Based Factual Passages (with Past Year Questions)

  • Discursive Passages (with Past Year Questions)

Question 2 - Case Based Factual Passages (with Past Year Questions) - Reading Comprehension - English Class 10

Last updated at April 16, 2024 by Teachoo

This question asked in [CBSE Outside Delhi Set-3, 2019]

Read the following passage carefully: 

(1) keep your watch accurate. for some people, moving up the time on their watch will help them get up earlier.  for others, they will remember that the time on the watch is wrong and will disregard it altogether. it may be  helpful to set your watch just two minutes ahead instead of five or ten. , (2)keep a clock, phone, computer or anything that displays time in each room of your house. one of the easiest  ways to run late is simply by not realising that the time is passing as quickly as it is. , (3)set all your clocks and watches to the same time. don’t be an optimist. things usually take longer than what  you’d expect, even without major delays. if you have a dinner appointment at 7:30 p.m., don’t think you can  work till 7 p.m., then take a bath, dress and reach on time. realistically, calculate the time you will take at each  step and then add 10 minutes more to allow for unexpected delays, or you cannot get to your job done in time. , (4)wake up when you are supposed to wake up. don’t hit the snooze button, keep on lying in bed, and watch  tv at the very start of your day. may be you can try even setting your clock 10 minutes earlier than you need.  if you have difficulty with this, move your alarm clock to somewhere away from your bed; that way, you will  have to get up to turn it off. commit yourself to being 15 minutes early for everything. if you have to reach  your place of work at 8:00, don’t even tell yourself this. just tell yourself (and everyone else who listens — but  don’t annoy them or make them think that they are late or early) “i have to be at work at 7:45.” if you do this,  you will be on time even with little unforeseen delays. you will be on time even with a traffic jam..

Clock.jpg

Question 2 ( i )

Keep a______  in each room. , (a) clock   , (b) phone , (c) computer   , (d) any one of the above.

So, the correct answer is (D)

Question 2 (ii)

It is good to commit yourself ______ for  everything. , (a) to be late   , (b) to delay , (c) to being 15 minutes early , (d) to search to the last moment.

So, the correct answer is (C)

Question 2 (iii)

To be punctual we should  ______. , (a) hit the snooze button of the alarm clock  (b) get up at the right time , (c) start watching tv in the morning , (d) keep on lying in bed.

So, the correct answer is (B)

Question 2 (iv)

It is a good habit to  ______. , (a) reach your work place in time. , (b) calculate the time before starting any work. , (c) keep lying in bed after the alarm clock buzzes. , (d) both (a) and (b)., question 2 (v), according to the passage the watch should be  set  _________  ahead. , (a) five minutes, (b) ten minutes , (c) two minutes, (d) fifteen minutes, question 2 (vi), the writer suggests to keep the watch  ____. , (a) ahead of time  , (b) accurate , (c) inaccurate  , (d) none of these, question 2 (vii), the given chart shows that students should  get up ____________. , (a) before 5 a.m. , (b) after 5 a.m. , (c) between 5 a.m. to 6 a.m. , (d) at 7 a.m., question 2 (viii), a student spends maximum time in  _____ during the day time. , (a) taking meals, (b) taking rest , (d) sleeping, question 2 (ix), a student  ____________  devote time to  studying on his own. , (a) can  , (b) should not , (c) should  , question 2 (x), proper time ___________ helps everyone lead  a stress free life. , (a) organisation  , (b) management , (c) planner  , (d) devotion, question 2 (xi), which one of the following is not a time management tip , (a) prioritize your work. , (b) set time limit. , (c) remove important tasks from ‘to-do’ list. , (d) plan ahead., question 2 (xii), the word which means the same as ‘correct  in all details’ in para 1 is  ___________. , (a) watch  , (b) disregard , (c) ahead  , (d) accurate.

Maninder Singh's photo - Co-founder, Teachoo

Maninder Singh

CA Maninder Singh is a Chartered Accountant for the past 14 years and a teacher from the past 18 years. He teaches Science, Economics, Accounting and English at Teachoo

Hi, it looks like you're using AdBlock :(

Please login to view more pages. it's free :), solve all your doubts with teachoo black.

cbsencertsolutions

CBSE NCERT Solutions

NCERT and CBSE Solutions for free

Unseen Passage For Class 10 English With Answers

Please refer to Unseen Passage for Class 10 with Answers provided below. Unseen passages and Comprehensions are really important in Class 10 English exams. Students are expected to read through the entire passage and then answer various MCQ questions. We have provided here the best collection of Class 10 Unseen Passages with solves questions which the students can use to practice on a daily basis to get better marks in Class 10 English exams.

Class 10 English Comprehensions with Answers

Unseen Passage: As ‘Dirty’ Factories Thrive, Toxins Spread Across

Industrial pollution is not restricted to the peripheries but is silently causing damage inside the city. On the Edge, a study by an NGO, has identified 18 potential pollution hotspots in the capital. Unsafe, and often unorganized, methods of e-waste dismantling, dyeing, lead acid battery recycling, cathode-ray tube (CRT) dismantling and other processes have given rise to health and environmental concerns, says the Toxics Link study. Delhi has more than 1.2 lakh industries and 29 industrial estates, necessitating a watch over acid and lead fumes, as well as groundwater and air pollution. The researchers assessed 51 industrial areas on five parameters–industrial processes, chemicals used, emissions, disposal technique and occupational health hazard.

They found that 18 of them were flouting the norms. Prem Nagar in Mandoli houses 110 lead acid battery recycling units. There is no understanding of pollution control, claims the report. “Most units in this area rely on coal-to-fuel crude furnaces and recover lead in a crude manner. While recycling, battery acid is dumped on the ground, waste pile or into a waterbody. As lead plates are melted, lead ash settles in the surroundings, collects on clothing or is inhaled by workers,” it says.A large amount of waste at Seelampur, where e- aste is recycled, comes from the US and Europe. Workshops emit lead fumes and the melted lead solder often finds its way to nearby drains. “The working conditions are appalling with poorly ventilated rooms and no safety measures,” says the report about Moti Nagar, which recycles CFL devices. The landfills at Ghazipur, Bhalswa and Okhla are responsible for toxic leachate contamination of air and groundwater.

The Najafgarh drain basin, which includes Wazirpur, Naraina, Anand Parbat and Okhla industrial areas, is the largest surface drain joining Yamuna and contributes to over 50% of the wastewater discharged into the river. Wazirpur is one of the worst managed areas with 1,200 small units, a majority of which are involved in steel pickling. The report also highlights the need to address toxic smoke and dust from open dumping, burning and spilling of chemicals. “In 2011, MCD was supposed to close down around 22,000 units, but not much seems to have been done”, said Satish Sinha of Toxics Link.

Questions & Answers

Questions. Is industrial pollution restricted to the peripheries in Delhi? Ans. No, the industrial pollution has gone down deeper into the city of Delhi.

Questions. How many potential pollution hotspots have been identified by an NGO? Ans. An NGO has identified 18 potential pollution hotspots in the capital.

Questions. Name three things that have given rise to health and environmental concerns. Ans. Unsafe methods of the e-waste dismantling, dyeing and lead acid battery recycling have given rise to health and environmental concerns.

Questions. What were the parameters of industrial areas assessed by the researchers? Ans. The researchers assessed 51 industrial areas on the given parameters — industrial processes, chemicals used, emissions, disposal technique and occupational health hazard.

Questions. Where does a large amount of waste at Seelampur come from? Ans. A large amount of waste at Seelampur where e-waste is recycled, comes from the US and Europe.

Questions. What harm do the landfills at Ghazipur, Bhalswa and Okhla do to the environment? Ans. The landfills at Ghazipur, Bhalswa and Okhla contaminate air and ground water making it toxic.

Questions. Name the problems that should be addressed immediately as recommended by the report. Ans. The report highlighted the need to address toxic smoke and dust from open dumping, burning and spilling of chemicals.

Questions. What contributes to 50% of the wastewater discharged into the river? Ans. The Najafgarh drain and the industrial areas contribute to over 50% of the wastewater discharged into the river.

Unseen Passage with questions and answers for Class 10

Unseen Passage: US Pips Russia As Top Arms Supplier To India

The US has overtaken Russia as the largest arms supplier to India in the last three years. But in terms of overall numbers, Russia still remains the biggest weapons exporter to India, having notched military sales worth over $40 billion since the first MiG-21 jets and T-55 tanks in the 1960s to the present-day Sukhoi-30MKI fighters and T-90S main-battle tanks. Russia, of course, is now running scared of the aggressive inroads being made by the US into the money-spinning Indian arms market, as if the continuing challenge from France and Israel was not enough. India, incidentally, has even turned to Israel to upgrade some of its Russian-origin equipment. 

With IAF having inducted over 200 of the 272 Sukhoi-30MKI fighters ordered for over $12 billion, Russia is eager to quickly seal the final R&D contract with India to jointly develop the stealth fifth-generation fighter aircraft. If India indeed moves to induct over 200 such “swing-role” stealth fighters in the next decade, it will spend close to $35 billion on them.

Unseen Passage For Class 10 English With Answers

But that is in the future. On Tuesday, in a written reply to Rajya Sabha, defence minister Arun Jaitley said India spent Rs 83,458 crore on importing weapons over the last three years. The US stood first in this list with Rs 32,615 crore, followed by Russia (Rs 25,364 crore), France (Rs 12,047 crore) and Israel (Rs 3,389 crore). The US has bagged Indian defence deals worth almost $10 billion since 2007-2008.

The deals include ones for 12 C-130J ‘Super Hercules’ aircraft ($2 billion), eight P-8I long-range maritime patrol aircraft ($2.1 billion) and 10 C-17 Globemaster-III giant strategic airlift aircraft ($4.1 billion), as earlier reported by TOI. India is also now close to inking another two deals with the US, worth over $2.5 billion, for 22 Apache attack helicopters and 15 Chinook heavy-lift choppers. Both these helicopters outgunned their Russian rivals, Mi-28 Havocs and Mi-26s, in the sweepstakes.

The Apache deal will get bigger in the coming years since the Army also wants 39 such attack helicopters after IAF inducts the first 22. US defence secretary Chuck Hagel also made a renewed pitch for co-production of another dozen arms projects during his visit here last week. They range from Javelin anti-tank guided missiles, MH-60 Romeo multi-role helicopters, “big data” and cyber-security to magnetic catapults for aircraft carriers, spy drones, mine-scattering systems and 127mm warship guns.

India has consciously tried to diversify its arms imports since the 1999 Kargil conflict due to Russia’s propensity to not adhere to delivery schedules, jack up costs midway through execution of contracts, create hurdles in transfer of technology and unreliable spares supply. The long-delayed delivery of aircraft carrier INS Vikramaditya (Admiral Gorshkov) as well as the huge jump in its refit cost to $2.33 billion was just the most prominent example of it.

Questions & Answer

Questions . How has the Us overtaken Russia? Ans. The US has overtaken Russia in the supply of arms to India.

Questions. Why is Russia scared? Ans. Russia is scared of the aggressive inroads made by the US in sales of arms to India.

Questions. Why has India turned to Israel? Ans. India has turned to Israel to upgrade some of its Russian-origin equipments.

Questions. How much money will India invest to develop the stealth fifth generation fighters in the next decade? Ans. India will spend close to $ 35 billion in the purchase of 200 stealth fighters in the next decade.

Questions. Name four countries that have topped in the sales of weapons to India. Ans. The US, Russia, France and Israel are the four countries that have topped in the sales of weapons to India.

Questions. What did the US defence secretary say about arms project in India? Ans. US defence secretary made a renewed pitch for co-production of arms in another dozen projects.

Questions. Why is India consciously trying to diversify its arms imports? Ans. India is consciously trying to diversify its arms imports as it doesn’t want to depend on any one country, mainly Russia, in this regard.

Questions. What is INS Vikramaditya? Ans. INS Vikramaditya is an aircraft carrier of the Indian Navy.

English Unseen Passage for Class 10 pdf with answers

Read the passage given below and answer the questions that follow:

Driving etiquette is a civilized driving behavior, comprising observance of rules and propriety. Certain statutory obligations have to be observed. While driving a vehicle, it is absolutely essential that all documents connected with the ownership of the vehicle, driving license, pollution – free certificate in original etc. are kept in the car. They have to be produced on demand by the competent authority. When a person driving a vehicle is still a learner, an ‘L’ broad should be prominently displayed.

To caution pedestrians or a slow moving vehicle in front of us, sounding the horn is essential. We are bound to do it legally as well but we should not honk unnecessarily, creating sound pollution.

Basic human dignity is more important than strict observance of rules. At crossroads, those who come from our right always have precedence. We should give hand signals for those who follow us, particularly when we slow down, take a sudden turn or overtake a vehicle. Never overtake a vehicle from the wrong side even if there is space. When another driver makes a mistake, try to correct him if time permits, instead of abusing him or threatening him with dire consequences. When you make a mistake, admit it gracefully and apologize for it if possible. When we cause an accident, instead of escaping we should find out from the victim whether we caused any injury and whether any medical assistance is required. If necessary, we should take the injured person to the hospital and inform the police.

The sign of good driving is to create a feeling of safety among other passengers. Abrupt braking, smoking or speaking over the mobile phone makes them insecure. Drunken driving is illegal and immoral. Under the influence of liquor the driver does not know what he is doing. When you stop at a traffic signal, stop the engine as otherwise it leads to atmospheric pollution.

Powerful lights cause harm to those who come from the opposite side. Dipping and dimming lights should be a habit with us. Similarly, playing the music system at high volume is irritating to others. We must be sensitive to others’ needs. Keeping to our lane is not only convenient and safe for us but is also a legal compulsion. Finally, politeness on the roads does not cost us anything, it only makes driving a pleasure. So let us all be polite even as we observe the rules of the road.

On the basis of your reading of the passage, answer the following questions in about 30-40 words each

Question. What is driving etiquette? Ans. Driving etiquette is a civilized vehicle driving behavior comprising observance of rules and propriety. Certain statutory obligations need to be observed while driving a vehicle,

Question. Write some examples of etiquette to be followed while driving. Ans. Some etiquette to be followed while driving are – (a) At crossroads, those who come from our right should always have precedence. (b) We should give hand signals for those who follow us, particularly when we slow down, take a sudden turn or overtake a vehicle. (c) When another driver makes a mistake, try to correct him if time permits, instead of abusing him.

Question. What is the sign of good driving? Ans. The sign of good driving is to create a feeling of safety among the other passengers. Abrupt braking, smoking or speaking over the mobile phone makes them insecure.

Question. Which documents should be kept in the vehicle while driving? Ans. While driving a vehicle, the important documents to be kept inside are your driving license, pollution free certificate in original and any other documents connected with the ownership of the vehicle.

Tick the correct option:

Questions. Find a word from para 1 which is a synonym of ‘protocol’. (a) Etiquette (b) Competent (c) Obligations (d) Behavior

Questions. Find a word from the passage which is synonym of ‘mandatory’. (a) Essential (b) Possible (c) Statutory (d) Inform

Questions. Find a word from para 1 which is the synonym of ‘concerned’. (a) Insecure (b) Competent (c) Immoral (d) Legal

Questions. Find a word from the passage which means ‘sudden and unexpected’. (a) Abrupt (b) Essential (c) Injury (d) Assistance

Comprehension Passages for Class 10

Unseen Passage: Skilling India: The Way Ahead

India has over 35,000 institutions of higher education, a number that is one of the largest in the world.The central and state universities together still comprise a substantial percentage of them. However, over the next decade or two, this is likely to shift towards private institutions as more and more Indians decide to get higher education. India today is a rapidly changing country, which is markedly different from what it was 20 years ago when it started on a new path to liberalisation. With a large number of companies in the other hemisphere outsourcing jobs to India, the country has virtually become the world’s back-office.

This has also spurred the demand for quality graduates in multiple disciplines. With the number of middle class Indians swelling, the requirements of careers changing and a new breed of young Indians leading the clamour for world-class education at home, India’s education sector is also changing rapidly. The Gross Enrolment Ratio or GER in higher education in India is still a little less than 15 per cent.

This too varies from region to region. With 60 per cent of this huge population under the age of 25, the demand for higher education is set to grow. The government is responding to these requirements by establishing more AIIMS and IITS across states, but given the state’s limited resources, there remains a certain limit to government’s spending on education. In these circumstances, the rapidly increasing demands of Indians on the education front can be met only when the private sector invests heavily in the field.

And this is something it is already doing. According to a McKinsey report on Indian demography, 590 million people will live in Indian cities by 2030, almost twice the current US population. There will be 91 million urban homes by 2030 in India, as against 20 million today. A large number of rural residents would be shifting base to urban centers, giving up their age-old family profession of farming and cultivation in favour of new avenues for their children. These create a never-before seen demand for higher education in India. This is where the role of the private sector will and is proving to be crucial. It is not just the numbers that are increasing.

Another major shift is in the demand for quality. More globally-exposed Indians are today asking why a greater number of Indian educational institutions cannot provide world-class education, comparable to prestigious foreign institutions Consequently, not only are more and more private universities and colleges emerging everywhere, but also there is an increasing consciousness among ‘education providers’ that quality education is what the young Indians are looking for. Private universities are also breaking new paradigms in education by offering wide options of learning to students and developing new mechanisms of learning.

The teaching techniques and the quality of learning have also been transformed. Indian institutions today are also working to cater to a growing international clientele of students that are looking for new pastures to get an education. Some prestigious institutions today also attract foreign students to the country, with their state-of-the-art infrastructure and world-class education. It is time the government recognises the role of private sector education in building a sound future for India.

Questions & Answer

Questions. Does India have a high number of institutions of higher education? Ans. Yes, India has over 35,000 institutions of higher education.

Questions. Where will more and more Indians decide to get higher education in the coming decades? Ans. In the coming decades more and more Indians will look towards private institutions for higher education.

Questions. How has India virtually become the world’s back office? Ans. India has virtually become the world’s back office as foreign companies are outsourcing jobs to India.

Questions. What are the requirements of careers changing in India? Ans. With the number of educated middle class Indians swelling, the requirements of careers are changing in India.

Questions. How is the government responding to the requirements of higher education? Ans. The government is responding to the requirements of higher education by establishing more AIIMS and IITs across states.

Questions. What will be the condition of Indian cities by 2030? Ans. By 2030, almost 590 million people will live in cities in India, almost twice the current population of the United States of America.

Questions. What is the thing in higher education that the young Indians are looking for? Ans. Young Indians are looking for the quality education or the world-class education in India.

Questions. What role can the private sector education play in the future? Ans. The private sector education can play a leading role in building a sound future of India.

Solved Unseen Passages for Class 10

Unseen Passage: 98 Countries Have Abolished Death Penalty

Written history has several mentions of capital punishment. The legal principle lex talionis — an eye for an eye, a tooth for a tooth, a life for a life — was mentioned in the 18th century BCE code of Babylonian king Hammurabi. Many ancient societies had similar

codes to justify judicial executions. The 7th century BCE’s Draconian Code of Athens, the 5th century BCE Roman Law and Kautilya’s Arthasastra, all had provisions of death sentence. These executions were done by several methods like crucifixion, drowning, beating to death, stoning, burning alive, impalement, hanging and so on.

According to Amnesty International’s 2014 report on death penalties, in 2013, at least 778 executions were reported in 22 countries, 96 more than in 2012. With at least 369 executions in the year, Iran leads the list. It is followed by Iraq (169+), Saudi Arabia (79+), USA (39) and Somalia, where at least 34 judicial executions took place. Other countries that reported more than 10 executions were Sudan and Yemen. The global executions figure does not include China which is believed to have been executed thousands of convicts. China classifies death penalty as a state secret. The report also shows that at least 1,925 people were sentenced to death in 57 countries in 2013. As of 2013, there are 23,392 death row inmates globally. The methods of execution range from beheading, hanging, lethal injection to shooting.

Amnesty International reports that as of the end of 2013, more than two-thirds of all countries have abolished death penalty in law or in practice. There are 98 countries which have abolished it for all crimes. Most of these are in Western Europe and the Americas. Seven countries, including Brazil, Chile and Kazakhstan have abolished it for ordinary crimes. In these countries, death penalty can only be given for exceptional crimes such as crime committed under military law or under exceptional circumstances. Another 35 countries are categorized as abolitionist in practice. These retain the death penalty for ordinary crimes, but there have been no executions in the past 10 years. The death sentence is retained by the legal system of 58 countries. More than half of the world’s population lives in these countries.

They say that it is possible for innocent people to get executed because of unfair and discriminatory application of the death penalty. Studies across the world have shown that in most cases the person sentenced to death is from an economically and socially backward section of society, indicating the inability to hire good lawyers to contest their cases. Many studies have suggested that there is no evidence to show that capital punishment has any effect on murder rates. It is also argued that the sentence is a denial of human rights and sends a wrong message — that killing is acceptable under certain circumstances.

Questions & Answers

Questions. What did the code of Babylonian king Hammurabi say about death penalty? Ans. The code of Hammurabi believed in the concept — an eye for an eye, a tooth for a tooth, a life for life.

Questions. Name any three methods of executing the death penalty to the condemned persons. Ans. Crucifixion, burning alive and hanging were the three main methods of executing the death penalty to the condemned persons.

Questions. How many people were executed in 2013 in different countries of the world? Ans. According to Amnesty Internationals’ 2014 report, at least 778 people were executed in 22 countries in 2013.

Questions. Why does the Amnesty Internationals’ 2014 report not include China in it? Ans. The Amnesty Internationals’ 2014 report doesn’t include China as it classifies death penalty as a state secret.

Questions. How many countries have abolished the death sentence? Ans. There are 98 countries in the world which have abolished the death penalty for all crimes.

Questions. How many countries retain the death sentence in their legal system? Ans. The legal system of 58 countries still retain the death sentence for the condemned persons.

Questions. What do opponents of capital punishment say? Ans. They argue that the death sentence is a denial of human rights to live.

Questions Is there any evidence to show that capital punishment has any effect on murder rates? Ans. There is no concrete evidence to show that capital punishment has any appreciable effect on murder rates.

Class 10 Solved Unseen Passage

My Nation, My Prayers

So many religious and ethnic groups, so much cultural diversity… over 350 languages; more than 1600 dialects; nearly 650 different tribes. A different food habit every few kilometers… and yet, one country!

With Independence, we just didn’t become free from British rule; we regained the freedom to be what we are, to live the way we want to. And that freedom has been the hallmark of this great region.It’s not a coincidence that India has never invaded a country in her long history. It’s her unwritten mission statement to be a giver, not a taker. She gave the concept of zero to the world; the game of chess, algebra, trigonometry and calculus; she gave the world its first university, the earliest school of medicine, and she gave four religions to the world. She nurtures hundreds of mosques, churches, temples and gurdwaras, to name a few; she gave asylum to more than 300,000 refugees who fled religious and political persecution. The list is endless and unparalleled. But all this will pale in front of one gift she has given to the world: PRAYER.

Take for example the most popular Gayatri Mantra. It doesn’t invoke God for any small individual comfort. Instead, it calls out for inspiration and guidance for our intellect. No polytheism, no segregation, no discrimination. Just a genuine cry for righteousness! And that has been core of all common and popular prayers of this nation. A longing to move from untruth to truth; from darkness to light and from death to immortality is the spirit of “Asato ma sad gamaya…” Where else can we find a more inclusive wish for wellbeing than the meaning of “Sarve bhavantu sukhinah…”? This elementary prayer wishes happiness, goodness, and freedom from misery and pain for all.

There is even a prayer seeking not to hate each other. Add to it such simple but profound prayers like “Lokah samastah sukhino bhavantu” (May peace and happiness prevail) and Satyam param dhimahi (Let truth, divinity and knowledge shine through me). They unite the entire creation into a bond of divine blessing. Another one-liner that is mind boggling for its exclusiveness came to my knowledge when I heard Sri Sri Ravi Shankar inspire a huge gathering to say the prayer “Annadata shukhbava” before eating. It was different from the more traditional mealtime prayers like the “Brahma arpanam…” But when explained how this wish for happiness covers the entire food chain from the farmer to the miller to the trader to the cook to the waitress… I was awestruck.

It made me wonder why we haven’t made such deep prayers a part of our national consciousness. Who can have objections to praying for such divine attributes? They nowhere limit whom you should pray to. Allah, Jesus, Krishna, Ganesh or Shiva don’t seem to matter. Then, why are our children not being taught the essence of these rayers? Those who protest such prayers in the name of secularism not only expose their ignorance of their meaning, but also do the biggest disservice to the ideals of secularism. If everybody is blessed with these attributes, the essence of secularism will be a fait accompli.

It’s time to turn to the last word of the Rig Veda, another gem from the Indic region. This word is the ultimate essence of unity. It’s a commitment, a call to move together, not just at the physical level but at the levels of thoughts, feelings and consciousness. Sanghachadwam! Let’s progress together!

Question & Answer

Question . Why has India never invaded a country in her long history? Ans. India has never invaded any country in her long history as it has her unwritten statement. It has always been a ‘giver’ and not a ‘taker’.

Question . What does the Gayatri Mantra invoke? Ans. The ‘Gayatri Mantra’ doesn’t invoke God for small individual gains. It is a genuine cry for righteousness and well-being of mankind.

Question . How does the prayer “Annadata Shukhbava” cover the entire food-chain? Ans. The prayer “Annadata Shukhbava” is a wish for happiness. It covers the entire food chain from the farmer to the miller, from the trader to the cook and the waitress.

Question . How is the last word of the Rig Veda the ultimate essence? Ans. The last word of the Rig Veda is the ultimate essence of unity at both the physical as well as the levels of thoughts, feelings and consciousness. It invokes progressing together.

Question . Forms of spoken languages are called: (a) languages (b) dialects (c) alphabets (d) letters

Question . Synonym for the word ‘harassment’ in the passage is: (a) persecute (b) persecution (c) torture (d) torturing

persecution

Question . The correct noun form of the word ‘segregate’ is: (a) segregated (b) segregating (c) segregation (d) segregate

segregation

Question . The expression ‘fait accompli’ means: (a) as already decided by fate (b) fate (c) fortune (d) fate will decide

as already decided by fate

Case based Unseen Passage for Class 10

Unseen Passage: Revision Of Green Laws May Hit Delhi

The ministry of environment and forests has asked for the public’s comments on reviewing five crucial environmental laws, including the Air and Water (prevention and control of pollution) Acts—any amendment to which will impact the city massively.

Delhi Pollution Control Committee (DPCC), the pollution watchdog that implements these acts in Delhi, will not comment. DPCC officials claimed MoEF hasn’t asked them to. “We are not sure if we are supposed to express our views. We may comment once the draft new law is ready. The state governments can’t do much when a committee to review the laws has already been set up,” said a senior official.

Sanjiv Kumar, environment secretary, Delhi, also said he has no idea if state governments are to make suggestions. The committee, however, said on MoEF’s website that it “desires” to engage in consultation with state governments.

Meanwhile, environmentalists are concerned they may not be able to articulate how these Acts can be strengthened given the ministry’s 1,000 character (a little over six text messages) limit. Many are worried the review has been commissioned to dilute environmental laws and penalties on polluters, especially industries.

Rahul Choudhury, advocate and NGT Bar Association member said, “There is no scope to dilute these laws because they are already very weak.” He cited the example of state pollution control boards that continue to renew no objection certificates to industries not conforming to norms. “There is no provision in the acts to review past conduct by pollution boards and revoke NOCs. The head of the board is usually a political appointee, often with no knowledge of environmental issues. Such loopholes make things easy for industries,” he said.

Comparing the country’s pollution control boards with the US Environmental Protection Agency, Choudhury said, “They have far more stringent parameters. In the last few years, more than 80% industries in Ghaziabad and Noida have not been complying with the Air Act”.

C R Babu, DU professor emeritus and chairman of the state-level expert appraisal committee, and Anumita Roychowdhury, head of Centre for Science and Environment’s clean air programme, said emissions from automobiles need to be brought under the Air Act from the motor vehicle (MV) Act. As of now, vehicular emissions are under MV Act.

“I am quite certain that this process has been started to weaken the acts but this will not help business in the long run. We will lose out tremendously on natural capital and ecological services. Can you imagine what’s going to happen if air and water pollution control laws are relaxed further in Delhi? People can’t survive in the city if that happens,” said Babu

Questions . Why has the ministry of environment and forests asked for public comments on environmental laws? Ans. The ministry of environment and forests has asked for public comments on environmental laws to review them rationally and amending Air and Water Acts, if needed, in the changing circumstances.

Questions . Why are environmentalists worried? Ans. Environmentalists don’t have genuine faith in the intention of the ministry of environment and forests. They are worried that the review has been commissioned only to dilute the laws regarding the prevention of pollution.

Questions . How do India’s pollution control boards compare with their counterparts in the US? Ans. In the US, the US Environmental Agency has far more stringent parametres when compared to their counterparts in India. In the last few years, 80% of industries in Ghaziabad and Noida have not been complying with their Act.

Questions . What will happen if air and water pollution control laws are relaxed in Delhi?. Ans.  It will have a disastrous effect on the environment if air and water pollution control laws are relaxed in Delhi. People can’t survive in the city if that happens.

Question. The word ‘articulate’ means (a) speaking (b) writing clearly (c) meditating (d) expressing clearly

Question. Synonym for the word ‘harsh’ in the passage is: (a) hard (b) difficult (c) stringent (d) tough

Question. The noun form of the word ‘pollute’ is: (a) polluting (b) pollution (c) polluted (d) none of these

expressing clearly 2. (c) stringent

Question. The opposite of ‘strengthening’ in the passage is: (a) weakening (b) diluting (c) removing (d) eliminating

Unseen Passage for Class 10 with questions and answers

Unseen Passage: Man Vs Nature

In 1999, sitting in the Srinagar Development Authority office with a team of planners to prepare the city’s 2000-21 master plan, I had not thought that it would stand gravely defied within its lifetime. Did the planners foresee Jhelum swelling up to breach the embankments, and water engulfing the whole city? Probably not.

Perhaps we were not so far-sighted, and driven only by the history of disasters in the city rather than their future possibilities. This is not a handicap exclusive to those of us who drew up what we thought was a forward-looking master plan for Srinagar.

Veteran planner G M Pampori was leading the team based on his experience of preparing the first master plan of the city (1971-91). At 78 years, he appeared impatient and twitchy, as the exercise had started late – almost a decade late. The last master plan had lapsed in 1991. For almost 10 years we lived without a master plan in one of the most rapidly urbanising cities, and Srinagar in all probability was not an exception. The city grew at a fast rate, water bodies shrank, unauthorised colonies came up. Residents certainly took their land, nature, vegetation and water bodies for granted.

Stand on top of Shankaracharya hill in Srinagar overlooking the city, and you can tell that the centre of Srinagar city is Dal Lake. The city shares a unique relationship with water. Water bodies have played a huge role in its expansion and development. Geological evidence proves that Kashmir was once a vast lake, and this is also part of the many narratives on Kashmir, most famously Rajatarangini by Kalhana.

Meandering Vitasta — ancient name of river Jhelum — was the genesis of Srinagar city, which served as the main artery of transportation and as the nerve centre of its social and cultural life. People’s daily life revolved around the river and the numerous water channels linked to it. Today, old-age monuments situated on its banks are clustered with buildings of the modern city.

Dal, Nagin, Anchar lakes, river Jhelum, wetlands like Khushalsar, Hokarsar and many other water bodies make the city a picturesque sight, besides continuing to provide it vital means of sustenance. They have borne silent witness to ruthless modernisation, villages turning into towns, towns turning into cities. Modern concrete structures squeezed out natural bodies, and master plans were repeatedly violated.

The city grew oblivious to its surroundings and expanded without showing any mercy to its water bodies and natural habitat. Encroachment on Dal Lake reduced its size to 15% of the original, shrinking it from 75 sq km to around 12 sq km.

The natural hydrology of this region connects its water bodies through small channels, to provide natural flow and even outflow of water within them. Rapid urbanisation and growth have cut off these connections between the water bodies and increased pollution in them. This has led to choking of several lakes — including Dal Lake — which earlier formed a natural flood lung of Jhelum, and took in reverse flows when it flooded. A flood spill channel was also constructed in the early 20th century to take the strain of water in Jhelum when it passed through the city, but it hardly worked.

The early 20th century also witnessed the beginning of a continuous, ongoing process of migration from the inner, older core to city suburbs. New residential colonies came up and

Srinagar got its first motorable roads, leading to a decline of its traditional system of canal transportation. As a result, in the 1970s the famous Nallah Mar Canal (built in the 15th century) was filled in and a road-widening scheme was launched along its bank, cutting through much of the historical fibre of the city. This scheme also spelled ecological doom for the Brarinambal and Khushalsar water lagoons.

In the 15th century, when Sultan Zain ul Abidin (commonly known as Bud Shah) was building the Nallah Mar Canal as a main artery of communication between the old city in Srinagar and the villages near Dal Lake, was he ahead of his time in understanding and sensitively promoting the natural linkages of water bodies?

Questions & Answer 

Questions . How and why were the city planners of Srinagar Development Authority caught unprepared by the unprecedented disaster that struck it recently? Ans. The city planners of Srinagar were caught unaware by the unprecedented floods in September 2014. They were not foresighted and were driven only by the history of disasters in the past rather than their future possibilities

Questions . Name some of the causes that led to such a sorry state of affairs in Srinagar. Ans. Unplanned urbanisation, shrinkage of water bodies, unauthorised colonies, encroachments on Dal Lake were the reasons that led to such a sorry state of affairs in the state. The Dal Lake shrank from 75 sq km to around 12 sq km.

Questions . How have rapid urbanisation and growth affected the natural hydrology of the region? Ans. Srinagar continued expanding without showing any mercy to its water bodies and natural habitat. Encroachments, unplanned constructions and unauthorised colonies only worsened the situation and destroyed the hydrology of the region.

Questions . How was Sultan Zain ul Abidin ahead of his time? Ans. Sultan Zain ul Abidin built the Nallah Mar Canal as the main artery of communication between the city of Srinagar and the villages near Dal Lake. He understood the sensitivity of promoting the linkages of water bodies and so was far ahead of his time.

Questions . Raised banks of a river are called: (a) bunds (b) dams (c) embankments (d) banks

embankments

Questions . The word related to the ‘earth’ in the passage means: (a) earthly (b) worldly (c) geological (d) geology

Questions . Synonym for the word ‘hub’ in the passage is: (a) artery (b) centre (c) main (d) central

Questions . Choose the most appropriate word for ‘natural water bodies and water’ in the passage is: (a) water (b) hydrology (c) precipitation (d) none of these 

Unseen Passage for Class 10 with answers

Read the passage given below and answer the questions / complete the sentences that follow :

In India, as perhaps in many other countries of the world, trees have been held in veneration since ancient times. Our rishis of yore retired to forests to meditate and to do penance amidst trees “far from the madding crowd”. Our forefathers loved trees and forests so much so that, they even deified and worshipped them as abodes of Vrikshadevata and Vanadevata. Many of them were not cut without good reason. In ancient India, groves of trees and flower gardens were reared with loving care. In ancient Sanskrit literature, there are many references of young maidens kicking the Ashoka tree to make it flower. But in fact, the kick could be administered by anyone. The method seems to work, though nobody really knows why? Some say the kick interrupts the flow of water upwards, triggering a defence mechanism which causes the tree to flower. In western countries people beat their walnut trees to encourage them to bear fruit. It is believed that a good beating ensures a good crop of walnuts. Like the Ashoka, the Walnut too is a tall tree. Perhaps all tall trees benefit occasionally from a beating sound. In parts of Uttar Pradesh, they resort to another strategy to get mango trees to flower. If a tree has not given fruit in a particular year, the owner, accompanied by three or four friends, walks menacingly towards it with an axe and loudly announces his intention of cutting down the tree. His friends persuade him not to do so. When he has gone they berate the tree for not giving fruit and advise it to give plenty of fruit the next time if it wants to live. The method is said to give good results. Question. Why did Rishis retire to forests? Answer. Rishis retired to forests to meditate and to do penance

Question. Why are walnut trees beaten in Western countries? Answer. Walnut trees are beaten in Western countries to encourage them to bear fruits.

Question. In Uttar Pradesh mango trees are caused to flower by __________. Answer. advising them

Question. According to some people kicking __________ the upward flow of water. Answer. interrupts

Question. One similarity between Ashoka and Walnut trees is that both are (a) thick trees (b) strong trees (c) weak trees (d) tall trees

Question. The word ‘menacingly’ in the passage means (a) penance (b) threateningly (c) encourage (d) amidst

Question. Vrikshadevata and Vanadevata are referred to : (a) The Sky (b) The Sun (c) The Earth (d) The Trees

Question. In ancient times, young maidens used to kick which tree? (a) The Ashoka (b) The Walnut (c) The Banyan (d) The Neem

Unseen Passage for Class 10 with answers pdf

Read the passage given below and answer the questions that follow :  During our growing up years, we as children were taught–both at home and school—to worship the photos and idols of the Gods of our respective religions. When we grew a little older, we were told to read holy books like the Bhagwad Gita, the Bible and the Quran; we were told that there are a lot of life lessons to be learnt from these holy books. We were then introduced to stories from our mythologies which taught us about ethics and morality—what is good and what is bad. I also learnt to be respectful towards my parents who made my life comfortable with their hard work and love and care, and my teachers who guided me to become a good student and a responsible citizen. Much later in life, I realised that though we learn much from our holy books, there is a lot to learn from our surroundings. This realisation dawned upon me when I learnt to enquire and explore. Everything around us—the sun, the moon, the stars, rain, rivers, stones, rocks, birds, plants and animals—teach us many valuable life lessons. No wonder that besides the scriptures in many cultures, nature is also worshipped. The message that we get is to save our environment and maintain ecological balance. People are taught to live in harmony with nature and recognise that there is God in all aspects of nature. Nature is a great teacher. A river never stops flowing. If it finds an obstacle in its way in the form of a heavy rock, the river water fights to remove it from its path or finds an alternative path to move ahead. This teaches us to be progressive in life, and keep the fighting spirit alive. Snakes are worshipped as they eat insects in the field that can hurt our crops, thus protecting the grains for us. In fact, whatever we worship is our helper and makes our lives easy for us. There are many such examples in nature, but we are not ready to learn a lesson. Overcome with greed, we are destroying nature. As a result, we face natural disasters like drought, flood and landslides. We don’t know that nature is angry with us. However, it is never too late to learn. If we learn to respect nature, the quality of our life will improve.

Question. What message do we get when we worship nature? Answer. By worshipping nature, we get the message to save our environment and maintain ecological balance. People are taught to live in harmony with nature and recognise that there is God in all aspects of nature.

Question. How does a river face an obstacle that comes in its way ? Answer. A river finds an obstacle in its way in the form of a heavy rock, the river water fights to remove it from its path or finds an alternative path to move ahead.

Question. What are we taught in our childhood and growing years ? Answer.  In our childhood and growing up years, we are taught to worship the photos and idols of the gods of our respective religions and also read the holy books like the Bhagwad Gita, the Bible and the Quran.

Question. Why should we respect our parents and teachers ? Answer. We should respect our parents as they make our lives comfortable with their hard work, love and care; and teachers guide us to make us good students and responsible citizens.

Question. What does a flowing river teach us? Answer. A flowing river teaches us to be progressive in life and keep the fighting spirit alive.

Question. harmony (a) beauty (b) friendship (c) discomfort (d) honesty

Question. ethics (a) conduct (b) deed (c) action (d) moral philosophy

Question. guided (a) answered (b) advised (c) fought (d) polished

Question. explore (a) search (b) frequent (c) describe (d) request

Question. valuable (a) proper (b) desirable (c) available (d) useful

Unseen Passage with multiple choice questions for Class 10

Read the passage given below and answer the questions that follow : 

THE FACE OF VILLAGE INDIA.

Thirteen years ago, when the then American President Bill Clinton drove down to Rampur Mani Haran village in western UP, he had come there to open a women’s polytechnic, funded by a prominent NRI. Today, the Bill Clinton school stands bright and shiny on the same campus, among low slung building that, house libraries and class rooms. According to the School President Raj Kamal Saxena, there are 565 students studying under CBSE of whom 234 are girls. Besides the local folk, the affluent families across the social spectrum sent their children to this institution. The success story of the school in the area has given rise to a spate of educational institutions along the road from Shamli to Saharanpur. All of them promise to unlock exciting career options especially for the landed class of the area. These schools carry trendy names and have sprung up in the years after the 42nd President of the US came calling. Sitting amidst sugar cane fields and mango orchards they present a picture of society trying to change. Despite such a positive social factor visible in the area, there is little evidence of all round changes in the area. The roads are battered and dusty. The dream of an all weather road remains a dream. Steady power supply is unthinkable and traditional industries are now on a decline. Added to these are the problems of governance such as law and order situation. According to locals, people in the region are looking for a change. The driving force behind this urge for change are the youth. Most of the young men who have returned to the area have been educated in Delhi and have returned to work in their home town and for them the development of the region overrides all other concerns. Among the semi agrarian middle classes, the call for a change is not through the region’s facilities, but through the ballot box. They are eager for a change in the very government of the state.

Question. What are the infrastructure drawbacks in the area? Answer. Unsteady power, battered roads, decline of industries are the infrastructure drawbacks in the area.

Question. What changes are the local youth and middle classes looking for ? Answer. They want the region to be developed. 

Question. What is the origin of the Bill Clinton School? What is its condition today ? Answer. It was started by Bill Clinton. It has children from affluent families as well as local folk.

Question. How has the presence of this school affected the area ? Answer. It has given rise to a spate of educational institutions.

Question. Find the word from the passage that means the same as ‘damaged’. (a) bargained (b) battered (c) benefited (d) borrowed

Question. What does the word ‘spate’ mean ? (a) occur (b) outpouring (c) orient (d) oblivious

Question. ‘Agrarian’ means ___________ (a) one who is born in Agra. (b) one who belongs to Agra. (c) one who works in Agra. (d) one who respects Agra.

Question. Find the word from the passage that means the same as ‘well off’. (a) affable (b) affluent (c) afford (d) affiliate

Question. Give the antonym of ‘new’. (a) turbulent (b) traditional (c) tardy (d) tame

Read the following passage carefully. 

A FOR ALLERGY

(1) Like bad news and common cold, allergies can pop up when least expected. I suddenly developed an allergy to crocin (paracetamol) some years ago after having it all my life to treat everything, from headache to fever to toothache. (2) A stuffed or drippy nose, frequent sneezing, an itchy throat, rashes, sinus, ear pain,difficulty in breathing, stomach cramps, itchiness, red or watery eyes are some of the common symptoms of an allergic reaction. Pollen, dust, polluted outdoor air, and indoor pollutants, such as dust mites, animal dander, cigarette smoke, and mould are among the common environmental pollutants, while other triggers include medicines, paint,and chemicals in cleaners and cosmetics, such as hair colour and skin creams. Among foods, eight allergens account for almost 90% of food allergies: milk, soy, wheat, egg,peanut, tree nuts, fish, and shellfish. (3) Most of us wrongly believe that people with allergies are either born with them or develop them in early childhood. An allergy can develop at any time in your life and its prevalence among adults is rising. While most people who develop allergies as adults have experienced some allergic reaction—either to the same or an unrelated trigger before—a few have no history of sensitivity. (4) In an acute immune reaction, the allergy trigger may be one, but the symptom is usually caused by a combination of factors. Stress, a sterile environment that prevents the body from developing immunity and lifestyle-induced changes in the body’s hormonal balance are thought to be some triggers. This has been borne out by clinical evidence that women are more likely to develop allergies at puberty, after pregnancy, and at menopause, all pointing to hormonal causes. (5) Avoiding the allergy trigger and taking anti-allergy medication as soon as you can, after exposure to an allergen is the best possible protection. Since pollen levels generally peak in the morning, people with airway sensitivity and asthma should postpone outdoor exercises to later in the day or stick to exercising indoors, as deeper and more rapid breathing induced by aerobic exercise causes more pollen and dust being inhaled, which can wreck your airways and lungs. Since air pollutants tend to cling to clothes and hair,changing your clothes when you come home or washing your hair before going to bed, lowers exposure.

On the basis of your reading of the given passage, choose the correct option. 

Question. Which of the following are the common symptoms of an allergic reaction? (a) Itchy throat and frequent sneezing (b) Muscle pain (c) Dryness (d) Hair fall

Question. Most of us wrongly believe that: (a) allergies can pop up when least expected (b) people develop allergies as adults (c) allergies among adults are rising (d) allergies are either born with them or develop them in early childhood

Question. What is the best possible protection? (a) Avoiding the allergy trigger and taking anti-allergy medication. (b) Rapid breathing induced by aerobic exercise. (c) Washing your hair before going to bed. (d) Pointing to hormonal causes.

Question. Based on your reading of the passage, choose the incorrect statement from the following. (a) An allergy can develop at any time in your life. (b) The allergy trigger is usually caused by a combination of factors. (c) Among foods, eight allergens account for almost 90% of food allergies. (d) Taking anti-allergy medication as soon as you can, after exposure to an allergen is the best possible protection.

Question. Which of the following best explains the phrase ‘account for’, as used in the passage? (a) Pass off (b) Make up (c) Write up (d) Look up

Read the following passage carefully.

HAPPINESS THEN AND NOW

(1) Happiness lies within the mind of an individual. No amount of external wealth may be helpful in making him happy. Our forefathers had lifestyles based mainly on the concept of ‘simple living and high thinking’. Excessive material wealth did not mean much to them. (2) The structural framework of our forefathers’ families was different from those of ours.The bond of love which they shared cannot be easily found today. The family provided an emotional cushioning effect against tension and stress. However, in today’s nuclear family, we are detached from feelings towards our kith and kin. This may finally destroy the family psyche. (3) The joint family system provided a proper environment for the child to grow up. The values of respect, tolerance, responsibility, and integrity were internalised in the child.In the long run, they became better human beings, compared to those in the present generation. Our forefathers felt happiest, if their children became honest human beings. But today, we are happy only if we attain our coveted material ends and are ready to resort to any means to attain that end. (4) Our forefathers had a vision to make India the best. To attain their ambitions, they were ready to make all sorts of personal sacrifices. On the other hand, today people are ready to migrate to the west, to enjoy a comfortable life. Often, they become successful in foreign lands. But, in the process, they become alienated from their motherland. Also, distance from their ailing parents is a worrying factor and keeps them perturbed. It is not easy for them to return, as their children will not be able to adjust to the Indian environment and way of life. Thus, this is a crisis and a frame of mind worse than their forefathers. (5) In our generation, tradition and modernity have intermingled to form a special system.We are happier than our forefathers in being able to lessen evils like the rigid caste system, untouchability, and child marriage, but we have failed to totally eradicate them. In fact, electoral politics has made use of the caste system, through the issuance of party tickets on the basis of caste, community, and religion.

On the basis of your reading of the given passage, choose the correct option.

Question. How are our forefathers different from us? (a) They had lifestyles based mainly on the concept of ‘simple living and high thinking’. (b) They had different structural framework of families. (c) They made all sorts of personal sacrifices to attain their ambitions. (d) All of these

Question. What impact does a joint family system have on a child? (a) It helps a child to become a better human being. (b) It internalizes the values of respect, tolerance, responsibility and integrity in a child. (c) Both (a) and (b) (d) Neither (a) nor (b)

Question. In our generation, _____________ and ____________ have intermingled to form a special system. (a) untouchability, intolerance (b) tradition, modernity (c) religion, caste (d) politics, corruption

Question. Based on your reading of the passage, choose the incorrect statement from the following. (a) Our youngsters had a vision to make India the best. (b) Happiness lies within the mind of an individual. (c) The family provided an emotional cushioning effect against tension and stress. (d) Our forefathers had lifestyles based mainly on the concept of simple living and high thinking.

Question. Choose the option that correctly states the two meanings of ‘cushioning’, as used in the passage. 1. Sharpening 2. Shielding 3. Intensifying 4. Softening 5. Irritating (a) (1) and (5) (b) (2) and (4) (c) (3) and (5) (d) (1) and (3)

Unseen Passage for Class 10

THE IDITAROD TRAIL

(1) In the early 1920s, settlers came to Alaska looking for gold. They travelled by boat to the coastal towns of Seward and Knik, and from there, by land, into the gold fields. The trail they used to travel inland is known today as the Iditarod Trail, one of the National Historic Trails designated by the Congress of the United States. The Iditarod Trail quickly became a major thoroughfare in Alaska, as the mail and supplies were carried across this Trail. People also used it to get from place to place, including the priests, ministers, and judges who had to travel between villages. In winter, the settlers’ only means of travel down this trail was via dog sleds. (2) Once the gold rush ended, many gold-seekers went back to where they had come from,and suddenly, there was much less travel on the Iditarod Trail. The introduction of the airplane in the late 1920s meant dog teams were no longer the standard mode of transportation, and of course, with the airplane carrying the mail and supplies, there was less need for land travel in general. The final blow to the use of the dog teams was the appearance of snowmobiles. (3) By the mid-1960s, most Alaskans didn’t even know the Iditarod Trail existed, or that dog teams had played a crucial role in Alaska’s early settlements. Dorothy G. Page, a self-made historian, recognised that only a few people knew about the former use of sled dogs as working animals and about the Iditarod Trail’s role in Alaska’s colourful history. To raise awareness about this aspect of Alaskan history, she came up with the idea to have a dog sled race over the Iditarod Trail. She presented her idea to an enthusiastic musher, as dog sled drivers are known, named Joe Redington Sr. Soon Page and the Redington were working together to promote the idea of the Iditarod race, and in 1967, the first Iditarod Trail Sled Dog Race became a reality.

Question. ‘suddenly, there was much less travel on the Iditarod Trail.’ Why? (a) Because of the introduction of airplane (b) Because many gold seekers went back to where they had come from, once the gold rush ended (c) Because only few people knew about the Iditarod Trail (d) Because mails and supplies were carried via dog sleds

Question. What was the final blow to the use of the dog teams? (a) The appearance of railways (b) The appearance of airplanes (c) The appearance of mobiles (d) The appearance of snowmobiles

Question. Which of the following statement is NOT TRUE about the Iditarod Trail? (a) People used the trail to get from place to place. (b) The Iditarod Trail is one of the National Historic Trails designated by the Congress of United States. (c) In winters, the trail was not used much by the settlers. (d) The trail became a major thoroughfare in Alaska.

Question. The first Iditarod Trail Sled Dog Race became a reality in: (a) 1920 (b) 1967 (c) 1897 (d) 1960

Question. Why did Dorothy G. Page come up with the idea to have a dog sled race over the Iditarod Trail? (a) To use sled dogs as working animals (b) To encourage dog sled drivers (c) To raise awareness about the Alaskan history (d) All of these

Case based factual Passage for Class 10

Read the following passage carefully. 

Natural calamities generally bring death and disaster on a large scale. Tsunami waves,earthquakes, floods, droughts and epidemics have taken a heavy toll of human lives,particularly the children. The children who survive these disasters suffer the most. They are condemned to lead a life full of misery, insecurity, want and fear. Only the concerted efforts of the Government and the agencies involved in disaster management can help them lead normal lives again.

The natural calamities have tragically affected the lives of about 15000 children in recent years in India. The tsunami waves and earthquakes created havoc on a large scale.About 5430 children became homeless due to the tsunami and 5438 were the victims of the earthquakes. Floods displaced 2602 and droughts affected 2035 children. Epidemics affected 1313 children. Out of these figures, 1093 children live constantly in general fear; 110 children have withdrawn socially; 20 children have become deaf and 16 of them can’t speak any more. More than 450 children can’t sleep at night while 1164 can’t concentrate on anything. The tragedy is that 21 of them have become blind, 43 asthmatic and 276 are still in shock. 566 suffer from frequent headache and stomach ache. About 850 children fear water and sound and 117 have refused to go to school. Disaster management programmes have not proved up to the mark and up to the task. The Government and the concerned agencies should provide comprehensive aids and help to the survivors, particularly the children. They must provide nutritious food, clothes, shelters and medical aids to them. All efforts should be made to wipe out the scars left by these disasters.

Based on your understanding of the passage, answer any six out of the eight questions by choosing the correct option.

Question. Based on your understanding of the passage, choose the option that lists the statements that are TRUE. 1. Some people are hugely benefited by a natural calamity. 2. Natural calamities can have a life altering impact on people. 3. Natural calamities cannot have any adverse impact on environment. 4. Fortunately, there is no economy losses due to any natural calamities. (a) 1 and 2 (b) 2 and 5 (c) 1 and 3 (d) 2 and 4

Question. What can make the lives of the children, affected with various disasters, normal? (a) The concerted efforts of the government. (b) The concerted efforts of the agencies involved in disaster management. (c) Both (a) and (b) (d) Neither (a) nor (b)

Question. As per the histogram, the most disastrous natural calamity for children is: (a) Earthquake (b) Tsunami (c) Flood (d) Drought

Question. The number of children suffering from frequent headache and stomach ache is: (a) 839 (b) 1,164 (c) 110 (d) 566

Question. Who should be responsible for providing the disaster affected children with nutritious food, clothes, shelters and medical aids? (a) The Government (b) The concerned agencies (c) Both (a) and (b) (d) Neither (a) nor (b)

Question. Which of these are the examples of natural calamities? (a) Floods (b) Tsunamis (c) Earthquakes (d) All of these

Question. Who among the following are the worst sufferers of natural calamities? (a) Elderly people (b) Youth (c) Children (d) Adults

Question. Natural calamities generally cause: (a) epidemics (b) death and disaster on a large scale (c) earthquakes (d) floods

Short Unseen Passage Class 10 with questions and answers

1. Overpowering prey is a challenge for creatures that do not have limbs. Some species like Russell’s viper inject poison. Some others opt for an alternative non-chemical method — rat snakes, for instance, catch and push their prey against the ground,while pythons use their muscle power to crush their prey to death. But snakes can’t be neatly divided into poisonous and non-poisonous categories.

2. Even species I listed as non-poisonous aren’t completely free of poison. The common Sand Boa, for instance, produces secretions particularly poisonous to birds. So the species doesn’t take any chance – it crushes its prey and injects poison as an extra step. Do vipers need poison powerful enough to kill hundreds of rats with just one drop? After all, they eat only one or two at a time. While hunting animals try their worst to kill most efficiently, their prey use any trick to avoid becoming a meal, such as developing immunity to poison. For instance, Californian ground squirrels are resistan to Northern Pacific rattlesnake poison.

3. Competition with prey is not the only thing driving snakes to evolve more and more deadly poison. Snakes also struggle to avoid becoming prey themselves. Some snake killers have partial immunity to poison. Famously, mongooses are highly resistant to cobra poison, and with their speed and agility, kill snakes fearlessly. It would be the death of cobras as a species if they didn’t evolve a more deadly poison to stop mongooses.Poison has another important role. It’s an extreme meat softener; specific enzymes break up the insides of the prey. Normally, a reptile depends on the sun’s warm rays to aid digestion.

4. But I wonder if we cannot use venom in our favour. In remote parts of India, local hospitality often involves leather-tough meat. I chew and chew until my jaws ache. If I spit it out or refuse, our hosts would be offended, I swallow like a python stuffing a deer down its throat and hope I don’t choke. If only I had poison.

Based on your understanding of the passage, answer any eight out of the ten questions by choosing the correct option.

Question. “But snakes can’t be neatly divided into poisonous and non-poisonous categories.” Choose the term which best matches the statement. (a) Over-confidence (b) Certainty (c) Dilemma (d) Ignorance

Question. Which of the characteristics is apt about the writer in the context: “If I spit it out or refuse, our hosts would be offended”? (a) Reckless (b) Compelled (c) Patient (d) Negligent

Question. Which one of these statements is NOT TRUE about mongoose? (a) A mongoose runs faster than a snake. (b) A mongoose is killed by a cobra. (c) A mongoose cannot kill a cobra. (d) A mongoose can kill any snake.

Question. Hunting animals try their worst to kill their prey. What does their prey do to avoid becoming their meal? (a) It develops strength to fight with them. (b) It develops immunity to poison/venom. (c) Both (a) and (b) (d) Neither (a) nor (b)

Question. Snakes strive to evolve a more deadly poison, so that: (a) their prey have no chance to remain alive. (b) they can stop mongooses. (c) Both (a) and (b) (d) Neither (a) nor (b)

Question. Some snake killer’s have partial immunity to: (a) dangerous bacteria (b) dangerous virus (c) poisonous air (d) poison

Question. The narrator chews and chews the leather-tough meat until his jaws ache, for fear of: (a) his hosts (b) his guests (c) pythons (d) choking

Question. Choose the option that correctly states the meanings of ‘offended’, as used in para 4. 1. disgusting 2. upset 3. wrong 4. displeased 5. unlawful (a) 1 and 2 (b) 2 and 4 (c) 3 and 5 (d) 4 and 5

Question. Select the option that makes the correct use of ‘driving’, as used in para 3. (a) Don’t talk while driving your car. (b) He is driving his sheep into a field. (c) Hunger is driving him to speak lie. (d) He is driving his team efficiently.

Discursive Passage for Class 10

Read the passage and answer the questions that follow:

1 The youth is a dynamo1, an ocean, an inexhaustible reservoir2 of energy. But this energy cannot be kept in prison. Its basic nature is to flow, to express itself. The youth energy, on the basis of the nature of its expression, can be divided into four categories.

2 The vast majority of the youth today are with the establishment3, whose formula of life is learn, earn, burn, and enjoy. It means learn to operate the modern devices and employ them to earn the maximum amount of wealth to the point of burning the natural resources of the earth, as well as yourself out, and then enjoy your own funeral. This category of youth is intelligent, skilful and hard-working, but it lacks insight4 and foresight5. They are self-indulgent and any sense of moral code of conduct is alien6 to their nature. Neither are they able to see in depth, to find out whether there is a deeper meaning and purpose to their human life, nor have they the capacity to look beyond the tips of their nose to find out the consequences of their way and approach, where it is leading them to. They are the ends unto themselves and enjoyment is the motto of their life.

3 The second category of youth, in nature and approach, is the same, but as it is less privileged and less qualified and skilled, it has lesser opportunities for earning and enjoying. Such youth may be incited to be against the establishment. This opposition takes various forms. When it is well organised and systemic7, it may take the form of political opposition and even go to the extent of expressing itself in unjust ways. When the opposition is not so intense and organised, it remains content with giving verbal expression to its resentment periodically. The youth of the above two categories need to be shown the correct path to positively channelize their energy.

4 The third section of youth is a sober and thoughtful class of people, which objectively observes and studies the phenomenon8 of development of the world. These youth find that man, in his insatiable thirst for consumption, has become blind and lost the sense of distinction between milk and blood. Today man, in his mad rush for exploitation, is sucking the blood of Mother Earth; leading to their destruction and is thereby digging his own grave. This responsible category of young people is looking for an alternative mode of development based on co-operation between man and man. This development based on mutual love, friendship and harmony is not only sustainable, but leading to endless prosperity mutually. To bring about his natural revolution from death-movement to life-movement is the aim of this group.

5 The fourth and most vital group of youth which is going to usher humanity into the third millennium9 and act as the pioneer10 for the future development of planetary life is engaged in evolving a new way of life and releasing a new principle of global consciousness through a fundamental research in the science of life. The science of life is a new branch of knowledge which takes the whole man into account without dividing him into subjective and objective halves of spirituality and physicality and does not treat him either as a refined (thinking) animal or an ethereal11 entity, having its base in some other non-physical world. It, rather, recognises man as a basic unit of conscious life which has got immense, practically inexhaustible, possibilities and potentialities for evolution, development and growth. As per the Vedic formula, man is the microcosmos12 and his fullest flowering and enfoldment lies in his identification with the cosmos.

Answer each of the questions given below by choosing the most appropriate option :

Question. Which trait is lacking in the youth that is dexterous with the modern devices ? (a) Intelligence. (b) Perseverance. (c) Foresight. (d) Skilfulness.

Question. Which category of youth supports the view that man’s growth lies in his identification with cosmos ? (a) The youth that will take us to the third millennium. (b) That which supports sustainable development. (c) Youth that opposes the establishment. (d) Youth that is self-indulgent and unscrupulous.

Question. What form can the opposition to establishmant take ? (a) Political opposition. (b) A verbal expression of rescutment. (c) Both (a) and (b). (d) None of the three.

Question. Sober and thoughtful youth think that man ………….. (a) has become blind. (b) cannot differentiate between milk and blood. (c) is sucking the blood of Mother Earth. (d) all of the three.

Question. What is the most vital group of youth doing ? (a) It is sucking the blood of Mother Earth. (b) It is evolving a new way of life. (c) It has become blind to the realities. (d) It has become non-violent.

Answer the following questions briefly :

Question. How has the writer praised the youth ? Answer :  Praising him highly, he has called the youth as a ‘dynamo’, an ocean and an inexhaustible store of energy.

Question. What is the motto of the majority of the youth do today ? Answer :  A great majority of the youth is with the establishment and their motto of life is — learn, earn, burn and enjoy.

Question. What is the most prominent quality associated with the modern man ? Answer :  The man today has an insatiable thirst for consumption and he is thoughtlessly exploiting the available resouress.

Question. Explain the attitude of modern man towards the Mother Earth.  Answer :  The modern man, in his mad rush, has gone to the extent of sucking the blood of Mother Earth.

Question. How are the less privileged youth at a disadvantage ? Answer :  Being less qualified and skilled, they have lesser opportunities for earning and enjoying.

Question. What has been said about the sober and thoughtful youth ? Answer :  This section of youth objectively observes as well as studies the phenomenon of development of the world.

Question. Pick out the words/phrases from the passage which are similar in meaning to the following : (a) serious and sensible (Para 4) (b) the universe seen as a well-ordered whole (Para 5) Answer :  (a) sober. (b) cosmos.

1 India has never subscribed to the doctrine1 of militarism and war in her history. Here war was never treated as an ideal. It was only tolerated as unavoidable and inevitable, and all attempts were made to check it and bring it under control. In spite of the frequency of wars in ancient India, in spite of highly developed military organization, techniques of war and imperialism, and in spite of the open justification of war as national policy, the heart of India loved pacifism2 as an ideal capable of realization. India’s symbolic role was that of a peacemaker and it sincerely pinned its faith on the principle of ‘Live and let live’. At least philosophically, India’s intelligence supported the cause of peace not only in national affairs, but in international affairs also. All the great seers of the yore3 visualized the unity of life, permeating all beings, animate or inanimate, which ruled out killing and suicidal wars.

2 This doctrine of philosophical pacifism was practised by ancient Aryans is, no doubt, a question of controversial4 nature. Certainly, the great Indian teachers and savants stuck to this doctrine tenaciously5 and in their personal life they translated it into practice and preached it to masses and even to princes of military classes.

3 Another culture of those times, the existence of which has been proved by the excavations of Mohan-jo-Daro, also enunciated6 the doctrine of pacifism and friendship to all. Strangely enough, the Indus Valley civilization has revealed no fortification and very few weapons.

4 Ahimsa or the doctrine of non-violence in thought, speech and action assumed a gigantic7 importance in the Buddhist and Jain period. By a constant practice of this virtue, man becomes unassailable by even wild beasts, who forgot their ferocity the moment they entered the circumference of his magnetic influence. The monks and nuns of the churches were apostles of peace, who reached every nook and corner of the world and delivered the message of love to war-weary humanity. The greatest votary8 was the royal monk Ashoka, who in reality was responsible for transforming Ahimsa as an act of personal virtue, to Ahimsa as an act of national virtue.

5 Many a historian recounting the causes of the downfall of the Mauryas, hold the pacific policy of Ashoka which had eschewed9 the aggressive militarism of his predecessors, responsible for an early decay of the military strength of the state and its consequent disintegration, leading to the rise of Sungas, Kanvas and Andhras. But in reality, the fault lies with the weak successors of Ashoka, who could not wield the weapon of non-violence with a skill and efficiency which required the strength of a spiritual giant like Ashoka. They failed due to their subjective weakness : pacifism itself was no cause of their failure.

6 Besides the foregoing philosophical and religious school of thought, even many political authorities gave their unqualified support to the cause of pacifism. They recognized the right of rivals to exist, not mainly as ememies, but as collaborators10 in the building of a civilization operation. Thus, for centuries, in the pre-Mauryan India, scores of small independent republics existed and flourished without coming in clash with each other.

7 With regard to Kautilya, the much maligned11 militarist and the so-called Machiavelli of India, he thinks that the object of diplomacy is to avoid war.

8 The Mahabharata observes in this connection, “A wise man should be content with what can be obtained by the expedients12 of conciliation, gift and dissention.” It denounces the warring world of men by comparing it to a dog-kennel. “First there comes the wagging of tails, then turning of one round to other, then the show of teeth, then the roaring and then comes the commencement of the fights. It is the same with men; there is no difference whatever.” Yajnavalkya adds : ‘War is the last expedient to be used when all others have failed.” Likewise, Sri Krishna whose Bhagwad-Gita has been styled by some as ‘a song of the battle’, should not be considered out-and-out militarist. When all the three expedients were exhausted, then alone the fourth was resorted to. 

9 All possible avenues of peace such as negotiation, conciliation through conference, meditation and so on, were explored before the war was resorted to. This proves that the heart of ancient India was sound and it longed for peace, although war also was not treated as an anathema13, which was to be avoided as far as possible.

Answer each of the questions by choosing the most appropriate option :

Question. Aryans preached and practised this to the masses. (a) Non-violence. (b) Freedom of speech and action. (c) Philosophical pacifism. (d) Practice of military organization.

Question. Mahabharata compares the warring world with ………. (a) wise men. (b) dog-kennel. (c) song of the battle. (d) militarist.

Question. The heart of India loved ………. (a) a highly developed military organization. (b) techniques of wars and imperialism. (c) loans. (d) pacifism.

Question. Principle of ‘Love and let live’ means ……….. (a) imperialism. (b) militarism. (c) frequency of wars among nations. (d) role of peacemakers.

Question. Unearthing Mohan-jo-Daro reinforced the following of pacifism. (a) There was no fortification or many weapons. (b) They delivered the message of love. (c) They were apostles of peace. (d) They thought that the object of diplomacy is to avoid war.

Question. How did the Aryans practise the doctrine of Pacifism ? Answer :  They stuck to this highly lauded doctrine by translating it into actual practice and preaching it to masses and even to princes.

Question. Why should Bhagvad-Gita not be considered as ‘A song of the battle’ ? Answer :  It is because it denounces the warring world by comparing it to a dog-kennel. The great scripture considers war as the last resort when all other options have failed.

Question. How was war treated in India ? Answer :  War was never treated as an ideal in India. It was only tolerated when all other attempts had failed to check it.

Question. What is Ahimsa ? Answer :  Ahimsa is the doctrine of non-violence in thought, speech and action.

Question. Describe India’s preparedness for war in spite of their belief in Pacifism. Answer :  Though India believed in Pacifism, the country was always prepared for war. It had a highly developed military organisation.

Question. Pick out the words/phrases from the passage which are similar in meaning to the following : (a) to express in definite and clear terms (Para 3) (b) the beginning (Para 8) Answer :  (a) enunciate. (b) commencement.

Question. What is the meaning of co-existence with rivals ? Answer :  It means that Indians recognize the right of rivals to exist, not as enemies, but as collaborators in the development of culture.

unseen passage for class 10 with answers pdf

Related Posts

Unseen Passage for Class 5 in English

Unseen Passage for Class 5 in English

Unseen Passage For Class 8 English With Answers

Unseen Passage For Class 8 English With Answers

unseen passage for class 9

Unseen Passage For Class 9 English With Answers

NCERT Books and Solutions for all classes

NCERT Books and Solutions for all classes

Unseen Passage for Class 10 English with Answers

Unseen passage for class 10.

Unseen Passage: Difficult But Worth It

With high-visibility film releases and continued discounts on all brands, Delhiites usually have a lot to do on their much awaited August 15 holiday. But as some kite lovers tell us, even with all the options available to them, switching off from those options and coming back to kites and socializing is worth it. Amit Chadha, who runs his own business in Kamla Nagar, tells us, “The excitement of flying kites is not the same as it used to be when we were kids. Tab toh kuch aur hota hi nahi tha karne ko, sirf terrace kite wars hote they. Now, cancelling movie and travel plans to fly kites with friends seems like a bad idea to most, but my group of friends and I still make it happen. We think it’s worth switching off from the usual rigmarole on one day and doing something relaxed and fun like this. Every year, we get a lot of friends who see our updates online, and then join us by the evening. This year also, we have a kite flying get-together at my place on the 15th.” Saurabh Tiwari, an MBA student staying with his family in Paschim Vihar in West Delhi, adds, “I’ve been away from home for the last two I-Days, but now that I am back, I want to do what we’ve always done on this day — get a music system on the roof, some snacks, lots of friends – and fly kites. I have made a WhatsApp group and I’m convincing my friends to get together and come over to our old flat’s terrace. August 15 used to be a day-long kite-flying party for us, and I really want to do the same thing now, even though it doesn’t happen so much in other parts of the city these days.”

Passing on the tradition

Getting over the whole August 15 kite flying concept is most difficult for children of the 80s and the 90s, who have grown up participating in society functions where they indulged in kite battles. And for them, getting their kids and the next generation excited about the same is a very “important challenge”. Juhi Malpani Bhatt, a professor of architecture, who stays with her family in Dwarka, tells us, “When we were kids, we would get excited about kites weeks in advance. I really missed all that when I moved out of Delhi for a short while. But now, I want both my kids to feel the same way about kites. We’ve been getting them kites for the past couple of years, and thanks to their school functions and our efforts, they’ve developed a liking for it and that makes me happy and nostalgic.” Vinod Taneja, who works with a bank in Green Park, is still trying to get his kids out of their rooms and onto the terrace. He says, “For us, kite flying came so naturally. We were never taught how to do it, bas dekhi dikhai ho jaati thi. I am trying to get my son to be involved in the whole August 15 kite flying thing now. But usko TV se hata ke upar leke jaana hi ek struggle hai. Plus, how do I teach him? This year again, we’ve got kites and manjhas and have invited friends over. I really want him to feel the way we felt about this day. So let’s see what happens.”

Questions & Answer

Questions. What do some kite lovers tell about their option? Ans. Some kite lovers would like to use the option of flying kites on August 15.

Questions. What does Amit Chadha say about the excitement of flying kites? Ans. Amit Chadha feels that the excitement of flying kites is not the same as it used to be when he was a kid.

Questions. Do people in general like cancelling movies and travel plans to fly kites? Ans. No, people in general would not like cancelling movies and travel plans to fly kites.

Questions. How does Saurabh Tiwari enjoy flying kites on the roof? Ans. Saurabh Tiwari will take some snacks, a music system and lots of friends on the roof to enjoy kite flying on the Independence Day.

Questions. Is getting over the whole August 15 kite flying concept easy for children of the 80s and 90s? Ans. No, the children of 80s and 90s were brought up in the tradition of flying kites on the Independence Day.

Questions. What makes Juhi happy and nostalgic? Ans. Flying kites on the Independence Day with her kids makes Juhi happy and nostalgic.

Questions. How does Vinod Taneja struggle with his son regarding kite flying? Ans. Vinod Taneja struggles with his son as the child is glued to the television and has no interest in flying kites.

Questions. What does Vinod Taneja want his son to feel? Ans. Vinod Taneja wants his son to feel the way he felt about the Independence Day himself when he was a kid.

Discursive Passage for Class 10

Unseen Passage: transfer of power in India

Two-thirds of a century into India’s independence, two aspects of the country’s political evolution are noteworthy. The first is the institutionalization of the periodic transfer of power, peacefully and predictably, recently evident in NDA’s victory earlier this year. But the journey has been much rockier with regard to another critical question: how to direct that power for the broader public good. 

While popular commentary on political power focuses on its misuse for private gain or corruption, there has been less attention on the limited ability of political power to translate intentions into outcomes. 

The history of independent India is replete with government programmes, ranging from state-owned enterprises to multiple poverty programmes, where political power did have good intentions, but where outcomes have left much to be desired. Critics have put the onus on misaligned incentives and a craven political-bureaucratic nexus. 

These factors have their roots in a distinctive feature of India’s political evolution: namely the weakness of the Indian state, hobbled as much by lack of competence as by corruption. Historically the state in India has always been weak and this changed only modestly after Independence. Yes, the state expanded massively; and yes the social composition of the functionaries of the Indian state has changed markedly. 

Size and social legitimacy undoubtedly have built state ‘strength’ — the negative power of the Indian state to thwart is certainly manifest. But positive power — the power to do something, to execute programmes and provide basic public goods that are the bread and butter of a state’s responsibilities to its citizens — is still a far cry. 

Why strong states develop in some societies and not in others is a complex historical question. One argument is that a strong state can only be built on a firm foundation of nationhood which itself is still a work-in-progress in India. Another view is that warfare laid the foundations of the modern nation state especially in Europe and East Asia. 

Historian Charles Tilly famously argued that states make war and war makes states, a reference to the rise of the modern European state after centuries of warfare among hundreds of polities and kingdoms. The ability to wage war successfully requires states to create viable systems of taxation, mobilization and coordination — and only those states that can, survive. But these attributes are also critical for any modern state to deliver public goods and services

Questions. How has the transfer of power in India taken place? Ans. The transfer of power in India has taken place quite peacefully and in a democratic manner.

Questions. Why has the journey been much rockier regarding the second critical question? Ans.  Regarding translating good intentions into solid result, the journey has been quite rockier for the government so far.

Questions. Why has there been less attention on the implementation of schemes? Ans. There has been more stress on talking than, giving attention to the implementation, of schemes by the government.

Questions. What were the outcomes of the good intentions of the government? Ans. The outcomes of the good intentions of the government were not very encouraging. 

Questions. Whom do critics hold responsible for this poor show? Ans. Misaligned incentives and the nexus between politicians and bureaucrats have been held responsible for the poor show.

Questions. What has still remained a far cry? Ans. Providing relief to the masses and implementing the welfare programmes has remained a far cry so far.

Questions. What is another view regarding laying the foundations of the modern nation state? Ans. Another view is that warfare laid the foundations of the modern nation state in Europe and East Asia.

Questions. What has the historian Charles Tilly argued? Ans. Historian Charles Tilly argues that states make war and war makes states.

Unseen Passage with multiple choice questions for Class 10

Unseen Passage: Speeding Up India’s Journey By Bullet trains

                                                                                    By: Vijay Kumar Dutt, Indian Railways High speed in Indian Railways is at present limited to 150 KMPH. However, in many other countries the speed of Railways is of the order of 200 KM per hour (KMPH). In France, Japan, Germany, China, Spain and South Korea high speed traction above 280 KMPH has been introduced. India and Japan have signed a memorandum of understanding (MoU) on 12th December, 2015 on cooperation and assistance in the Mumbai–Ahmedabad High Speed Rail (HSR) Project referred to by many as ‘Bullet Train Project’. Japan has offered an assistance of over Rs. 79,000 crore for the project. The loan is for a period of 50 years with a moratorium of 15 years, at an interest rate of 0.1 per cent.

The project is a 508-kilometre Railway line costing a total of Rs. 97,636 crore, to be implemented in a period of seven years. It has been agreed that for the Mumbai – Ahmedabad HSR Project. Japan’s Shinkansen Technology, known for its speeds reliability and safety, will be adopted. Transfer of technology and “Make in India” will be essential part of this assistance package. Japan will also assist India in training of personnel for HSR.’ The two countries have also entered into two comprehensive technological cooperation agreements on 11th December 2015, for modernization and up gradation of Indian railways. These agreements have been signed during the official visit to India of His Excellency Mr. Shinzo Abe, the Prime Minister of Japan, during December 11-13, 2015.

On the basis of your reading of the passage, answer the questions: 

A) Name the countries where high speed traction above 280 KMPH has been introduced.

High speed traction above 280 KMPH has been introduced in France, Japan, Germany, China, Spain and South Korea

B) Which project is known as “Bullet Train Project”?

The MoU signed between India and Japan on cooperation and assistance in the Mumbai–Ahmedabad High Speed Rail (HSR) Project is known as “Bullet Train Project”

C) How much time and money is estimated for the Project?

The time of seven years and an amount of about Rs 97,636 crore is estimated for the project

D) Which technology will be adopted for the Project?

Shinkansen Technology will be adopted for the Project.

E) What agreements were signed during 11th December to 13th December2015?

The agreements signed during 11 th December to 13 th December 2015 were modernization and upgradation of Indian railways.

F) What is Shinkansen Technology famous for?

Shinkansen Technology is famous for its speeds reliability and safety

G) How has Japan assisted India financially for the Project?

Japan has offered an assistance of over Rs. 79,000 crore for the project. The loan is for a period of 50 years with a moratorium of 15 years, at an interest rate of 0.1 per cent.

H) In paragraph No. 2 the synonym of ‘essential’ is : (a) unwanted (b) inessential (c) unnecessary (d) fundamental

The word is ‘fundamental’.

Unseen Passage for Class 10 with answers pdf

Unseen Passage: 1986 Asian Games at Seoul

It was in the 1986 Asian Games at Seoul when Kartar Singh beat Pakistan’s Shahid Pervaiz Butt in 100kg category to claim the yellow metal. It was the last time an Indian grappler had won gold in the Asian Games.

Twenty eight years hence, the wrestling contingent will be looking to break the jinx. The wrestling squad -7 freestyle, 7 Greco-Roman and 4 women will be without two-time Olympic medallist Sushil Kumar who pulled out due to a shoulder injury and lack of preparation.

In his absence, London Olympics bronze medallist and Glasgow Commonwealth Games gold winner Yogeshwar Dutt will spearhead the Indian challenge in Incheon.

“We have trained very well. Sushil ki kami khalegi (The squad will miss Sushil’s presence). But we are still expecting around four to five medals in freestyle and this includes some gold medals too. In wrestling, an Asian Games medal is like winning the World Championship,” a confident Yogeshwar said.

India’s performance dropped from six medals (1 silver and 5 bronze) in the 2006 Asiad in Doha to only three bronze medals in 2010. But this time round, the squad promises to surpass expectations.

Young Amit Kumar, who won gold in the Glasgow Commonwealth Games and bronze in the World Championship, is one of the leading contenders for gold in the 57kg category. World No. 1 Hassan Rahimi of Iran is likely to pose a tough challenge for the Indian, who is ranked second in the world.

World No. 5 Narasingh Yadav will fill in for Sushil in the 74kg category while World Junior medallist Praveen Rana (70kg), who also won a gold in Colorado Springs, US, early this year, will look to impress. The 70kg category is being introduced for the first time after the International wrestling federation (FILA) introduced new rules and tweaked weight categories late last year in an attempt to make the sport more exciting.

Glasgow CWG silver medallist Bajrang Punia (61kg), bronze medallist Pawan Kumar (86kg) and silver medallist Satayavrat Kadiyan (97kg) complete the line-up in the freestyle category.

There will be no participation in the 125kg category. The Indians are likely to face a stiff challenge from South Korea, Mongolia, Iraq, Japan and Kazakhstan.

“This team is better than the team that participated in Guangzhou in 2010. Yogeshwar and Amit are contenders for gold while Bajrang, Praveen and Narsingh have the capability to contribute to India’s medal haul,” coach Yashvir summed it up.

In the Greco-Roman, India, traditionally a force at the Asian level, are likely to return with a rich medal haul. The squad includes five-time Commonwealth champion and Asian Games bronze medalist Ravinder, Asian champion Krishan Kumar Yadav, World Championship bronze winner Sandeep Yadav, Asian Championship bronze medalist Manoj Kumar and, Arjuna Awardee Dharmender Dalal.

Questions & Answer

Questions. When did an Indian grappler win gold in the Asian games last time? Ans. It was in 1986 Asian games at Seoul when the Indian wrestler Kartar Singh won the gold medal last.

Questions. What has the wrestling contingent been looking forward to during these 28 years? Ans. During these 28 years the wrestling contingent has been looking forward to break the jinx.

Questions. Name the Olympic medalist who has pulled out of the games due to a shoulder injury. Ans. Sushil Kumar, the two time Olympic medallist, has pulled out of the Asian Games 2014 due to a shoulder injury.

Questions. Who will spearhead the Indian challenge in wrestling in Incheon? Ans. Yogeshwar Dutt, who won the bronze at the London Olympics will spearhead the Indian challenge in wrestling in Incheon.

Questions. How is winning an Asian games medal in wrestling like? Ans. Winning an Asian Games medal in wrestling is like winning the world championship.

Questions. How did India perform in 2010 in New Delhi? Ans. India’s performance dropped from six medals in Doha to only three bronzes in 2010 Asian Games.

Questions. Who will fill in for Sushil in the 74 kg category at the Asian games? Ans. Nar singh Yadav, World No. 5, will fill in for Sushil in the 74 kg category.

Questions. Name the countries that will throw a stiff challenge to Indian wrestlers in Incheon. Ans. South Korea, Mongolia, Iraq, Japan and Kazakhstan will throw a stiff challenge to Indian wrestlers in Incheon.

Short Unseen Passage Class 10 with questions and answers

Unseen Passage: Confusing Signals

Prime Minister Narendra Modi’s comment during his recent visit to Leh that Pakistan had lost the ability to fight a conventional war and was hence engaging in a proxy war by aiding and abetting terrorists does nothing to move the needle forward on improving India-Pakistan relations. The comment – made to Indian soldiers and officers – fails to take cognisance of the fact that the Nawaz Sharif government in Islamabad isn’t the sole or even main actor in determining Pakistan’s India policy.

It’s hardly a coincidence that hours after Modi’s comment Pakistani troops violated the ceasefire along the border. Again predictably, Islamabad’s foreign office responded to Modi’s charge by stating that Pakistan itself was a victim of terrorism and that New Delhi would do well not to engage in blame games. All of this takes away from the positive momentum that had been generated when Modi invited SAARC leaders to attend his swearing-in ceremony in May. That Sharif had made the trip to New Delhi had given rise to hopes that Modi is capable of thinking out of the box on Pakistan. But putting the screws on Islamabad ahead of the upcoming meeting of foreign secretaries will only strengthen hawks in the Pakistani establishment.

Moreover, the Sharif government is under siege with two separate protest marches to Islamabad – one led by opposition leader Imran Khan and the other by Canada-based religious scholar Tahir-ul Qadri – planned for today, Pakistan’s independence day. Both sets of protesters want Sharif to go and are rumoured to be supported by Pakistan’s military-intelligence complex. If the latter is true, it would mean that Pakistan’s security establishment is using the protests to send out a clear message to the civilian dispensation that it retains a veto on key issues.

Instead of doing the familiar talks-no talks routine with Pakistan, there is an unconventional path the Modi government can adopt to resolve the various irreconciliables it faces. If the Pakistan army is the main actor in determining India policy, a way must be found to engage the Pakistani armed forces themselves, whether through formal or informal channels, and to assuage their anxieties vis-a-vis India. UPA was hardly capable of this since it generally stuck to the tried and tested path. But Modi has shown a capacity to innovate and surprise, as in the case of his invitation to Sharif. More of such innovation will be needed for progress in India-Pakistan ties.

Question & Answer

Question. Why is Pakistan indulging in a proxy war? Ans. Pakistan has lost the ability to fight a conventional war with India. Hence, it is engaging India in a proxy war by aiding and abetting terrorists.

Question. What hope had Sharif’s trip to Delhi during Modi’s swearing-in-ceremony generated? Did it materialise? Ans. Sharif’s trip to New Delhi to attend Modi’s swearing-in-ceremony generated a hope that the relations between India and Pakistan would improve in future. The continued ceasefire violations have belied all such expectations.

Question. How is the Pakistani security establishment using the protests against the Sharif’s government? Ans. The Pakistani security establishment is using the recent protests led by Imran Khan and Qadri to strengthen its position. It wants to give a clear message to the government that it retains a veto on key issues.

Question. How can Modi deal with Pakistan in the new situation? Ans. Modi has already shown a capacity to innovate and surprise in such complex matters. He can adopt the right strategy to deal with Pakistan in the changed circumstances

Question. The most appropriate synonym for the word ‘traditional’ in the passage is: (a) conventional (b) convention (c) convent (d) innovative

conventional

Question. Choose the right option from those given below to give the noun form of the word ‘responded’ in the passage: (a) responsible (b) response (c) responsive (d) responding

Question. Those who believe in wars are called: (a) fighters (b) jingoes (c) hawks (d) violents

Question. Making sudden changes according to situation is called: (a) invention (b) innovation (c) innovate (d) innovated

Case based factual Passage for Class 10

Unseen Passage: In School, But Are They

Across the country, there is a simmering unease with the education that our 315 million students are getting. Everybody wants education, but most are dissatisfied with it. The biggest issue is this: will it help make a better life? But there is also the feeling, often confirmed, that students are not really learning much.

Several surveys of how well students are learning have shown dismal results.According to the ASER 2013 survey report, 60% of Class 3 students surveyed couldn’t read a Class 1 text. This is up from 53% in 2009. This doesn’t improve in higher classes –53% of Class 5 students couldn’t read a Class 2 text, up from 47% in 2009. A higher proportion is unable to deal with subtraction and division.

Although she doesn’t give much credence to these surveys, Anita Rampal, professor of elementary and social education at Delhi University’s Central Institute of Education, agrees that the schooling system is not delivering. There are three key factors behind a successful schooling system, according to her: building of knowledge and critical faculties, good facilities and environment in school, and an equitable system where all kinds of children learn together.

“In India, we’re lagging in all three and that is why students are not learning to their full potential,” rues Rampal.

Lessons in schools are often information driven, with the teacher giving information that students are expected to soak up and reproduce in the poorly designed examinations, she explains. Classrooms are dull, teachers just stuff information into students and the exam-centric approach finishes off any possibility of ‘learning’.

Contrary to popular perception, children drop out of school most often because they are not getting anything from it, says Meena Shrinivasan, an award winning children’s books author and educational consultant.

“Either the language used in school is too foreign to them and they are treated like inferior species, or the matter being taught is irrelevant, or the absence of toilets for girls makes it impossible to continue, or the teacher is harsh and beats children for not understanding or performing, or it is all just so boring and burdensome that it is just more fun to drop out,” she says. The most vulnerable students, dalits, tribals and girls quit school the first. A recent survey of nearly 1.52 million schools by NUEPA reveals a startling picture of facilities in schools. Over 41% schools do not have a playground, 43% don’t have electricity connection, 76% don’t have computers. Although more than three quarters of the schools had a library, 82% did not have a librarian to look after the books and guide the children. Worldwide, research shows that one of the most reliable predictors of success in later grades is good reading ability in early grades, which comes from good teaching and from a print-rich environment, says Shrinivasan. “Most children in this country come from homes where recreational reading is not a priority or even a possibility, and so they depend on school for their books. Most schools tend to choose some preachy morally uplifting books that no one wants to read, and these too are not easily accessible to children,” she stresses. Teachers who enjoy books and can share this passion with children, and know how to teach reading, and a plentiful supply of age-appropriate interesting fiction and non-fiction are what children need more than any other educational input, Shrinivasan says.

But the condition of teachers is such that 28% teachers in primary schools are not even professionally qualified according to official statistics. In some states the situation is even worse. In the eight north-eastern states, just 36% teachers are qualified on an average. In Bihar, Bengal and J&K about 3 out of 5 teachers are not duly qualified to teach primary students.

Whole generations of children —-India’s future – are going through this broken education system, somehow managing to get past exams, or dropping out by the wayside. It is not difficult to imagine what their, and the country’s future is likely to be if things are not improved drastically.

Question & Answer 

Question. Why is there a simmering unease with the education that 315 million students are getting today? Ans. Students want good education but most of them are dissatisfied with it. There is a simmering unease among them that they are not really learning much in such a set up.

Question. What are the possible three key factors behind a successful schooling? Ans. There are three possible key factors behind a successful schooling system. They are:(i) building of knowledge and critical faculties(ii) good facilities and environment(iii) an equitable system for all students

Question. What are the factors that let a large number of students drop out of school in the middle? Ans. Language problem, ill treatment, irrelevant subject matter, absence of toilets for girls are some of the factors that let students drop out of their schools in the middle.

Question. What is the general condition of teachers in most of our primary schools? Ans. About 28% of teachers in primary schools are not even professionally qualified. In the North-eastern states and in Bihar, Bengal and J&K, 3 out of 5 teachers are not duly qualified to teach primary students.

Question . The most appropriate option for the word ‘seething’ is: (a) simmering (b) restless (c) violent (d) anger

Question . Synonym for the word ‘kinds’ of persons is: (a) variety (b) disparities (c) species (d) races

Question . The opposite of the word ‘relevant’ is: (a) irrelevant (b) irrelevance (c) relevance (d) irreverent

Question . The noun form of ‘recreational’ is: (a) recreating (b) recreating (c) recreation (d) recreationing

Unseen Passage for Class 10 with answers

Unseen Passage: Reality Check By Toi Reveals Overflowing Garbage Dumps And Debris

Will the city be free from garbage and dirt by August 15? A week-long sanitation and cleanliness drive was launched with much fanfare by the administration a few days back, but a reality check reveals that it has remained sporadic. While the civic agencies are making tall claims publicly, the enormity of the task rules out making any significant difference by the end of the exercise.

TOI took a round of the city. Overflowing garbage dumps and waste and debris along the roads were a common sight though the corporations claim to be removing 30% more of garbage daily as part of the drive.

Hardly 500m away from the north and south corporations’ headquarters, Civic Centre, on Jawahar Lal Nehru Marg, at Turkman Gate, one found a huge garbage dump with the waste spilling on to the road. “It is a shame that the corporation has failed to clean the area visible from its own office. In the past one month, hardly anyone has been seen cleaning the area,” said Ramesh Babbar, a local trader.

Across the city, on CV Raman Road, outside New Friends Colony, there is a huge garbage pile. The residents have complained to the corporation several times since during peak traffic hours, this stretch becomes a bottleneck. “Never are the bins placed on the road clean. From the dhalao, garbage spills on to the road,” complained Madhu Arora, a resident living just across the road.

Similarly, in Lajpat Nagar-I’s B-block, just outside a government school, garbage has remained dumped for days. Several complaints have been made but to no avail. “From the school principal to parents, everyone has complained about the heap of garbage outside the school. It is very difficult for the girls to pass through this stretch. The stench is overpowering, especially during monsoon, and diseases are rampant in the area. The lane is narrow and the parked cars and garbage ensure that it remains blocked,” said area councillor Abhishek Dutt.

Across the river, near Laxmi Nagar’s V3S mall, the story is no different. There is a dhalao virtually in the middle of the road and the garbage spills out, making the space even narrower. With massive gridlocks during peak traffic hours, it is a nightmare for commuters. “With shops and a mall nearby, there are cars parked on the road, and the dhalao only adds to the chaos. It’s a breeding ground for mosquitoes, and for pedestrians, it’s a nightmare. But the corporation isn’t doing anything despite several complaints,” said BS Vohra of East Delhi RWA’s Federation.

But corporation officials are patting themselves on the back. A rally of school children was organized by North Delhi Municipal Corporation for City Zone on Tuesday to create awareness about sanitation, health and hygiene. “From every zone we are removing around 150 metric tonnes more than usual. On an average, Delhi produces 7000 metric tonnes of waste everyday, but for the past one week, we are removing more than 9000 metric tonnes,” said Mukesh Yadav, south corporation’s spokesperson.

Questions. What has been the outcome of a week-long sanitation and cleanliness drive that was launched by the administration with so much fanfare? Ans. The campaign of freeing the city from garbage and dirt by August 15, started by the administration has been a great flop. The campaign failed to make any significant improvement in the prevailing situation.

Questions. What does the reality check by TOI reveal? Ans. The reality check by TOI reveals that overflowing garbage dumps, waste and debris lying along the roads are still a common sight in the city. The tall claims of the administration have been proved totally false.

Questions. What is the situation just outside a government school in Lajpat Nagar-I’s B-Block? Ans. Garbage remains dumped for days outside a government school in Lajpat Nagar- I’s B-Block. It gives a foul stench. Diseases are rampant in the area.

Questions. Why are corporation officials patting themselves on the back? Ans. The Corporation officials are patting themselves on their back just for nothing. Their campaign of creating awareness about sanitation, health and hygiene has not achieved its desired aims. Dirt and garbage still stink the city.

Questions. The word used in the passage for ‘rubbish’ is: (a) garbage (b) dirt (c) dust (d) litters

Questions. Choose the appropriate noun form of the word ‘enormous’ in the passage: (a) enorm (b) enormity (c) anomaly (d) none of these

Questions. A ‘bad smell’ is called: (a) odour (b) stench (c) gas (d) scent

Questions. The correct synonym for the word ‘widespread’ in the passage is: (a) rampant (b) enormous (c) universal (d) common

Unseen Passage for Class 10 with questions and answers pdf

Read the following passage carefully.

TIGER SURVIVAL

(1) Now is the time to save the magnificent cats, before they vanish from the earth forever. There is a little time left to preserve these animals, so immediate action is required. The  latest predictions state that tigers will be extinct in the wild by the year 2025. That is less than six short years, from now; less than six years to save a creature that has been around far longer than us!

(2) I cannot imagine, nor would I wish, living in a world without tigers, without these magnificent creatures living freely in the forests.

(3) The power and beauty of tigers is indisputable. They are an integral part of the ecosystem as they are a major link in the food chain. Tigers have been bestowed magical and supernatural properties by many societies, and have also been revered as Gods. All living creatures of mother earth depend on one another for survival. We, as human beings, are brothers and sisters to every living being, from the plants to the animals and to each other.

(4) Tigers are often killed in the most barbaric and unbelievably cruel ways. Crimes like the killing of tigers must be stopped right way. We must do whatever we can to stop the killing of this magnificent creature. We can write to our government and the governments of countries with tiger populations and urge them to protect the tiger. Penalties for killing tigers and other protected wildlife need to be strict and deterring, and enforced with equal might to discourage the poaching of tigers, and bring to task those who profit from such killings.

Question. What can we do to protect the tigers? (a) We can plant more trees and discourage the practise of deforestation. (b) We can take care of the ecosystem as they are a major link in the food chain. (c) We can urge the government to protect the tiger and pose penalties for killing tigers. (d) We can built zoos to preserve the remaining population of tigers.

Question. What is indisputable? (a) Power of tigers (b) Beauty of tigers (c) Killing of tigers (d) Both (a) and (b)

Question. On what does all living creatures of mother earth depend for survival? (a) Wildlife (b) Natural objects (c) One another (d) Ecosystem

Question. Which of the following statement is NOT TRUE, according to the passage? (a) Tigers have been bestowed magical and supernatural properties by many societies. (b) Tigers are often killed in the most interesting and fascinating ways. (c) The power and beauty of tigers is indisputable. (d) The latest predictions state that tigers will be extinct in the wild by the year 2025.

Question. What are we, human beings, to every living being? A (a) Brothers and Sisters (b) Protectors (c) Enemies (d) Opponents

Unseen Passage for Class 10 with questions and answers

Read the passage given below and answer the questions that follow :

(1) Have you ever failed at something so miserably that the thought of attempting to do it again was the last thing you wanted to do?

(2) If your answer is yes, then you are “not a robot.” Unlike robots, we human beings have feelings, emotions, and dreams. We are all meant to grow and stretch despite our circumstances and our limitations. Flourishing and trying to make our dreams come true is great when life is going our way. But what happens when it’s not? What happens when you fail despite all of your hard work? Do you stay down and accept the defeat or do you get up again and again until you are satisfied? If you have a tendency to preserve and keep going then you have what experts call, grit.

(3) Falling down or failing is one of the most agonizing, embarrassing and scariest human experiences. But it is also one of the most educational, empowering and essential parts of living a successful and fulfilling life. Did you know that perseverance (grit) is one of the seven qualities that have been described as the keys to personal success and betterment in society? The other six are: curiosity, gratitude, optimism, self-control, social intelligence, and zest. Thomas Edison is a model for grit for trying 1,000 plus times to invent the light bulb. If you are reading this with the lights on in your room, you know well he succeeded. When asked why he kept going despite his hundreds of failures, he merely stated that they were not failures. They were hundreds of ways not to create a light bulb. This statement not only revealed his grit but also his optimism for looking at the bright side.

(4) Grit can be learnt to help you become more successful. One of the techniques that help is mindfulness. Mindfulness is a practice that helps the individual stay in the moment by bringing awareness of his or her experience without judgement. This practice has been used to quiet the noise of their fears and doubts. Through the simple practice of mindfulness, individuals have the ability to stop the self-sabotaging downward spiral of hopelessness, despair and frustration.

(5) What did you do to overcome the negative and self-sabotaging feelings of failure? Reflect on what you did, and try to use those same powerful resources to help you today.

Question. In what ways can grit be developed? Answer. Do not accept defeat; effort to overcome limitations; not to stay down.

Question. How does mindfulness help? Answer. Mindfulness is a practice that helps the individual stay in the moment by bringing awareness of his or her experience without judgement. This practice is a technique that many have used to quieten the noise of their fears and doubts.

Question. According to the passage, what are the attributes of a human? Answer. Feelings, emotions, dreams, eagerness to grow and flourish to realize dreams are some of the attributes of a human.

Question. What is perceived as grit? Answer. If you have a tendency to persevere and keep going, then you have what experts call grit.

Question. How is ‘failing’ an educational and empowering part of human life? Answer. Failing teaches how to be perseverant and keep going for one’s goals. Each failure makes one understand what to do and what not to do.

Question. In paragraph 2, ___________ means continue. (a) robots (b) satisfied (c) persevere (d) flourishing

Question. In paragraph 3, the synonym of ‘distressing’ is ___________. (a) embarrassing (b) scariest (c) agonizing (d) failing

Question. While inventing the light bulb, Thomas Edison had failed _____________ (a) 1000 times (b) 10000 times (c) 10000 times (d) 10000 times

Question. Failure is a part of ___________ life. (a) normal (b) common (c) human (d) ordinary

Question. ___________ helps in preventing individuals from going down the lines of despair. (a) Mindfulness (b) Satisfied (c) Persevere (d) Flourishing

Unseen Passage with questions and answers for Class 10

THE ROAD AHEAD

(1) Our opportunities are great, but let me warn you, when power outstrips ability, we will fall on evil days. We should develop competency and ability which would help us utilise the opportunities which are now open to us. From tomorrow morning—from midnight today—we cannot throw the blame on the Britishers. We have to assume the responsibility ourselves for what we do. A free India will be judged by the way in which it will serve the interests of the common man in the matters of food, clothing, shelter, and social activities. Unless we root out corruption in high places and root out every trace of nepotism, love of power, profiteering and black marketing, which have spoiled the good name of this country in recent times, we will not be able to raise the standards of efficiency in administration as well as in the production and distribution of the necessary goods of life. (2) Pandit Jawaharlal Nehru referred to the great contribution which this country will make to the promotion of world peace and the welfare of mankind. The chakra, the Ashoka wheel, which is there in the flag embodies for us a great idea, Ashoka, the greatest of our emperors. Look at the words of H.G. Wells about Ashoka, ‘Highnesses, Magnificences,Excellences, Serenities, Majesties. Among them all, he shines alone a star, Ashoka the greatest of all monarchs.’ He cut into rock his message for the healing of discords. If there are differences, the way in which you can solve them is by promoting concord.Concord is the only way by which we can get rid of differences. There is no other method which is open to us. (3) We are lucky in having our leader, one who is a world citizen, who is essentially a humanist, who possesses a buoyant optimism and robust good sense in spite of the perversity of things and the hostility of human affairs. We see the way in which his department interfered actively and in a timely manner in the Indonesian dispute. It shows that if India gains freedom, that freedom will be used not merely for the wellbeing of India but for Vishva Kalyana, world peace, the welfare of mankind. Extract from a speech by Dr Radhakrishnan

On the basis of your reading of the given passage, choose the correct option. 

Question. A free India will be judged by the way in which it will: (a) utilise the opportunities which are open (b) blame the Britishers (c) serve the interests of the common man (d) raise the standard of efficiency in administration

Question. How can we raise the standards of efficiency in administration as well as in the production and distribution of the necessary goods of life? (a) By rooting out corruption in high places (b) By encouraging love of power (c) By eradicating every trace of nepotism and black marketing (d) Both (a) and (c)

Question. Based on your reading of the passage, choose the incorrect statement from the following. (a) Concord is the only way by which we can get rid of differences. (b) A free India will be judged by the way we will root out corruption in high places. (c) If India gains freedom, the freedom will be used for the welfare of mankind. (d) Profiteering and black marketing have spoiled the good name of this country in recent times.

Question. Which of the following explains the phrase ‘root out’? (a) To find and remove (b) To mock and laugh (c) To fight and win (d) To promote and contribute

Question. ‘We are lucky in having our leader, one who is a world citizen, who is essentially a humanist, who possess a buoyant optimism and robust good sense in spite of the perversity of things and the hostility of human affairs’. Substitute the underlined word with the most appropriate option from the following. (a) Cheerful (b) Dull (c) Cunning (d) Moody

English Unseen Passage for Class 10 pdf with answers

Read the following passage carefully. 

DEMONETISATION 2016

(1) On 10 November 2016, the Indian government decided to demonetise the 500- and 1,000-rupee notes, the two biggest denominations of the Indian currency system. These notes accounted for 86% of the country’s circulating cash. With little warning, India’s Prime Minister, Narendra Modi announced to the citizenry that these notes would be rendered ineffective with immediate effect. People were given time till the end of the year to deposit or exchange them for newly introduced 2,000 and 500-rupee notes. (2) The government’s goal was to combat India’s thriving underground economy on several fronts: eradicate counterfeit currency, fight tax evasion (only 1% of the population pays taxes), eliminate black money accumulated from money laundering and terroristfinancing activities, and to promote a cashless economy. Individuals and entities with huge sums of black money acquired from parallel cash systems were forced to take their largedenomination notes to a bank, and account for them satisfactorily and submit the proof of tax paid. If the individual could not provide the proof of making any tax payments on the cash he/she possessed, a penalty of 200% on the tax due was to be imposed. (3) Demonetisation had a severe impact on the gold market. The extraordinary demand for the yellow metal brought a stiff hike in its cost. However, the government made it mandatory that every buyer had to submit his/her PAN card details for purchases made. (4) Many Indians switched to alternative payment methods. The biggest gainers were mobile wallet companies that offered easy transactions through a large network of partners. Alibaba-backed Paytm saw a sevenfold increase in overall traffic. Customers found the option of prepaid cash cards useful. Other alternatives included mobile payment systems linked to e-commerce businesses like Ola Money, FreeCharge, and Flipkart Wallet.

Question. Customers found the option of ________________ useful. (a) prepaid cash cards (b) submitting pan card details (c) paying tax (d) money laundering

Question. Which of the following statement is NOT TRUE, according to the passage? (a) Demonetisation had a severe impact on the stock market. (b) Alibaba-backed Paytm saw a sevenfold increase in overall traffic. (c) On 10 November 2016, the Indian government decided to demonetise the two biggest denominations of the Indian currency system. (d) People were given time till the end of the year to deposit or exchange the notes.

Question. Why did the government decide to demonetise the 500- and 1,000- rupee notes? (a) To eradicate counterfeit currency (b) To fight tax evasion (c) To eliminate black money (d) All of these

Question. What was the impact of demonetisation on the entities with huge sums of black money? (a) They were forced to take their large-denomination notes to bank. (b) They were asked to submit the proof of tax paid. (c) A penalty of 200% on the black money was to be imposed. (d) Both (a) and (b)

Question. What did the government make mandatory for every buyer to purchase gold? (a) To submit the proof of tax paid (b) To submit the PAN card details (c) To opt for mobile payment systems (d) To submit the proof of citizenship

Read the following passage carefully. 

The UN’s 2017 International Year tells that sustainable tourism is an important tool for development, most importantly in poor communities and countries. Today sustainability — environmental, social, and economic – is increasingly recognised as the benchmark for all tourism business. As noted by the UN World Tourism Organisation, 57% of international tourist arrivals will be in emerging economies, by 2030. The various ‘Tourism Terms’ are defined as follows:

Category  – Ecotourism                Definition – Responsible travel to natural areas that conserves the environment, socially and economically sustains the well-being of local people,and creates knowledge and understanding through interpretation and education of all involved (including staff, travellers, and community residents).

Category  – Ethical Tourism            Definition – Tourism in a destination where ethical issues are the key driver, e.g. social injustice, human rights, animal welfare, or the environment.

Category  – Geotourism                  Definition – Tourism that sustains or enhances the geographical character of a place– its environment, heritage, aesthetics, culture, and well-being of its residents. Category  – Pro-Poor Tourism                        Definition – Tourism that results in increased net benefit for the poor people in a destination. Category  – Responsible Tourism      Definition – Tourism that maximises the benefits to local communities, minimises negative social or environmental impacts, and helps local people conserve fragile cultures and habitats or species. Category  – Sustainable Tourism    Definition – Tourism that leads to the management of all resources in such a way that economic, social, and aesthetic needs can be fulfilled while maintaining cultural integrity, essential ecological processes, biological diversity, and life-support systems.

Based on data collected by a survey by Travel Bureau, the following market profile of an ecotourist was constructed:

Age: 35 – 54 years old, although age varied with activity and other factors such as cost. Gender: 50% female and 50% male, although clear differences based on activity were found. Education: 82% were college graduates, a shift in interest in ecotourism from those who have high levels of education to those with less education was also found, indicating an expansion into mainstream markets. Household composition: No major differences were found between general tourists and experienced ecotourists Party composition: A majority (60%) of experienced ecotourism respondents stated they prefer to travel as a couple, with only 15% stating they preferred to travel with their families, and 13% preferring to travel alone. (** experienced ecotourists = Tourists that had been on at least one “ecotourism” oriented trip.) Trip duration: The largest group of experienced ecotourists—(50%) preferred trips lasting 8-14 days. Expenditure: Experienced ecotourists were willing to spend more than general tourists,the largest group (26%) . Important elements of trip: Experienced ecotourists top three responses were: (a) wilderness setting, (b) wildlife viewing, (c) hiking/trekking. Motivations for taking next trip: Experienced ecotourists top two responses were (a) enjoy scenery/nature, (b) new experiences/places.

Based on your understanding of the passage, answer any six out of the eight questions by choosing the correct option.

Question. In the line “……… recognised as the benchmark”, the word “benchmark” does not refer to: (a) a basis for something. (b) the criterion required. (c) the ability to launch something new. (d) a standard point of reference.

Question. The World Tourism Organisation of the UN, in an observation, shared that: (a) emerging economies of the world will gain 57% of their annual profits from International tourists. (b) countries with upcoming economies shall see maximum tourist footfall from all over the world in the next decade. (c) a large number of international tourists in 2030 will be from developing countries. (d) barely any tourist in the next decade shall travel from an economically strong nation to a weak one.

Question. Choose the option that lists the correct answers for the following: 1. Asha Mathew, an NRI, loves animals and wishes to travel to places that safeguard their rights and inculcate awareness of their rights. What kind of tourist is she? 2. Gurdeep Singh from UK is an environmental scientist and has always chosen to travel to places that are examples of a symbiotic relationship between man and nature. What kind of tourist is he? (a) (1) is an ecotourist and (2) is a geotourist. (b) (1) is an ethical tourist and (2) is a geotourist. (c) (1) is a sustainable tourist and (2) is a pro-poor tourist. (d) (1) is a geotourist and (2) is a responsible tourist.

Question. In the market profile of an ecotourist, the information on gender indicates that: (a) female ecotourists were more than the male ecotourists. (b) the activity preferences were varied in females and males. (c) the choice of things to do on a trip were quite similar for both the genders. (d) male ecotourists were frequent travellers.

Question. According to the survey, one of the most powerful driving forces leading experienced ecotourism to invest in new trips was: (a) setting up work stations in new places. (b) the chance to go camping in the wild. (c) competing with other ecotourists as frequent travellers. (d) the opportunity to travel to new places.

Question. Choose the option that lists statement that is NOT TRUE. (a) Economically backward countries will benefit from sustainable tourism. (b) The tourism business currently recognises sustainability as an important factor. (c) Emerging economies will receive negligible international tourists in the near future. (d) The sustainability factor in tourism is a significant means for development.

Question. The survey clearly showed that the age range of ecotourists: (a) remained the same for the choice of tourist attractions to visit. (b) changed with the monetary requirements for the trip. (c) fluctuated due to male-female ratio. (d) was constant across various features of the trip.

Case based Unseen Passage for Class 10

More than 87,000 healthcare workers have been infected with Covid-19, with just six states — Maharashtra, Karnataka, Tamil Nadu, Delhi, West Bengal and Gujarat — accounting for three-fourths (around 74%) of the case burden and over 86% of the 573 deaths due to the infection, official data showed. Maharashtra alone, with the highest number of over 7.3 lakh confirmed Covid cases so far, accounts for around 28% of the infected healthcare workers and over 50% of the total deaths, according to the data. While Maharashtra, Karnataka and Tamil Nadu had tested over one lakh healthcare workers each till August 28, Karnataka reported only 12,260 infected healthcare workers — almost half the burden in Maharashtra. Tamil Nadu reported 11,169 cases that included doctors, nurses and Asha workers. The three states together accounted for 55% of the total cases among health workers. Risk to frontline workers can jeopardise India’s Covid fight — The three states also reported the highest number of deaths in healthcare professionals,though with a wide gap between Maharashtra and the other two. While Maharashtra reported 292 deaths among healthcare workers, Karnataka and Tamil Nadu had 46 and 49 deaths, respectively.

A large number of infections and even deaths of healthcare workers in particular states is being viewed with concern by officials and public health experts, who say risks to frontline workers can jeopardise India’s fight against the pandemic. The issue, discussed in a review meeting headed by the cabinet secretary on Thursday,saw the Centre cautioning states about the need to protect a crucial resource. The possible factors responsible for high infections, officials said, were lax infection control in hospitals and the need for stringent containment measures in areas where health professionals reside to safeguard them. Despite the high number of cases, the government has received only 143 claims since April under the ` 50 lakh Covid-19 insurance scheme for healthcare workers engaged in Covid mitigation activities. Official sources said the wide gap between the number of deaths and claims could be because all the casualties may not be eligible under the scheme. Besides, the claims are a bit slow in coming as families of the dead take time to apply and do the required paperwork.

‘Solidarity with health workers cannot be met with mere words of encouragement but by concerted efforts to strengthen the health workforce. Safety net for their families should be provided including a term insurance cover of over ` 2 crore, with the government as sole guarantee,’ said Giridhar Babu, epidemiologist at the Public Health Foundation of India. ‘Protecting healthcare workers is of paramount importance to make sure we have a large enough force to take care of patients who need their services.’ said Dr H Sudarshan Ballal, chairman, Manipal Hospitals, who said such workers may be at risk because of a large number of asymptomatic patients and lack of proper use of PPEs. (Source: The Times of India/Health Ministry)

Question. In the line “… risks to frontline workers”, the term ‘frontline workers’ does NOT refer to: (a) healthcare workers (b) police (c) cleanliness workers (d) teachers

Question. Which state of India is on the top in terms of confirmed COVID-19 cases? (a) Karnataka (b) Tamil Nadu (c) Delhi (d) Maharashtra

Question. Based on your understanding of the passage, choose the option that lists the factors responsible for high infection in healthcare professionals. 1. Careless infection control in hospital 2. Negligency by healthcare professionals 3. Lack of stringest containment measure 4. The lack of healthcare professionals (a) 1 and 2 (b) 2 and 4 (c) 1 and 3 (d) 3 and 4

Question. How many healthcare workers, infected with COVID-19, were there in Karnataka till August 2020? (a) 11,169 (b) 12,260 (c) 1,07,100 (d) 15,213

Question. Choose the option that lists statement that is NOT TRUE. (a) Maharashtra was the worst sufferer of Covid-19. (b) Karnataka had less number of Covid-19 cases as compared to Maharashtra and Tamil Nadu. (c) The deaths of healthcare workers are disregarded by officials. (d) Most of the families of deceased healthcare workers received ` 50 lakh under Covid-19 insurance scheme.

Question. ‘Healthcare workers’ refers to: (a) doctors (b) nurses (c) Asha workers (d) All of these

Question. On how many healthcare workers COVID-19 tests have been conducted in Punjab? (a) 1,127 (b) 994 (c) 13,141 (d) 2,029

Question. How many claims has the government received since April 2020 under the ` 50 lakh COVID-19 insurance scheme for healthcare workers engaged in COVID-19 alleviation activities? (a) 49 claims (b) 51 claims (c) 46 claims (d) 143 claims

Class 10 Solved Unseen Passage

CHILD MARRIAGE: AN EVIL

(1) Child marriages are rampant in North India. They continue to blight the lives of people.Children bound by marriage are victims of blind customs and superstitions prevalent in rural areas and certain urban concentrations among the weaker socio-economic groups.Nothing seems to stop this anti-social practice despite the Child Marriage Act passed as early as in 1929, which makes child marriage a grave offence. (2) Why do child marriages take place and what can be done to prevent them from happening? The evil thrives because of illiteracy and other related causes—the most important of which is the anxiety of parents to marry off their daughters at the earliest. In many high-illiteracy states, like Rajasthan, the practice of child marriage is in vogue.Akhha Teej is D-day for the parents of minor girls, since, on that day, the parents seek salvation from the anxiety of girls growing up in their midst. (3) A child marriage is less likely to take place if the parents are literate or at least the father is. He is, then, aware of the legal minimum age for marriage and the health hazards his daughter will face by an early marriage. If the mother, otherwise literate, has been exposed to the importance of family planning, she is also less likely to solemnise her daughter’s wedding before the legal minimum age. (4) Among the other reasons that parents give away young daughters in marriage is the need, felt especially by families with more than one daughter, to keep wedding expenses down. By marrying two daughters simultaneously, parents save on expenses. Parental anxiety about grown-up (14 years and above) daughters going astray, forces the less educated to give away their female children in marriage. (5) The Child Marriage Restraint Act in 19710, raised the minimum age of marriage for girls from 15 to 110 years and for boys from 110 to 21 years. The committee, on the status of women, in its report in 1974, had recommended that all offences under the Child Marriage Restraint Act should be made cognizable and special officers be appointed to enforce the law. (6) The crux of the problem is that the role of a girl-child in traditional rural areas is circumscribed around marriage and motherhood.

Question. Why do parents marry two daughters simultaneously? (a) To support the daughters (b) To avoid health hazards (c) To save on expenses (d) To enforce the law

Question. What is Akhha Teej? (a) The day when the family planning is exposed. (b) The day when parents seek salvation from the anxiety of girls growing up in their midst. (c) The day when the daughter faces health hazards. (d) The day when two daughters marry simultaneously.

Question. Children bound by marriage are victims of: (a) blind customs (b) superstitions (c) both (a) and (b) (d) neither (a) nor (b)

Question. Select the option that makes the correct use of ‘grave’, as used in the passage, to fill in the blank space. (a) Life is a battle from cradle to _____________. (b) In the _____________, the rich and poor lie equal. (c) We realised very quickly that we had made a _____________ mistake. (d) She knelt beside her father’s _____________ to place flowers.

Question. In what circumstance will a child marriage be less likely to take place? (a) If the children are victims of blind customs (b) If the parents are literate (c) If the parents seek salvation (d) If the parents keep wedding expenses down

Read the passage and answer the questions that follow:

1 The word ‘depressed1’ in common usage means sad, frustrated2, fed up, bored or pessimistic3. The mood of a depressed person is much lower at his or her best moments than the mood of the normal person at his or her worst. Depression is a state of mind. It is specifically a mental disorder characterised4 by a lowering of the individual’s vitality5, his mood, desires, hopes, aspirations6 and of his self-esteem7.

2 Depression arising out of environmental8 factors is called reactive depression whereas depression arising out of some biochemical9 changes in the brain is called endogenous10 depression. If depression is mild or moderate11 and if the individual is in touch with his surroundings, it is known as neurotic12 depression. If the individual is severely disturbed and is not able to comprehend13 what is happening around, he can be said to be in a state of psychotic14 depression.

3 Old age is one of the stages of human development, where a person attains15 wisdom, maturity, social and economic stability16 with social recognition and emotional fulfilment. Generally, societies show a great respect and consideration for the aged. In ancient times, old people were considered as the guiding stars in Indian families, since they were symbols of tradition, respect, wisdom and experience. In primitive17, ancient and medieval18 cultures, old persons had a recognized social role. They were of great value because they could impart knowledge and skill to youngsters. The old people were considered as repositories19 of wisdom and tradition and were not perceived20 as problems.

4 At present, social structures and values are undergoing transformation21 from traditional to modern. There is a rapid stride22 in urbanization and industrialization, leading to the breaking up of joint families and property. This has ultimately weakened the traditional families and social position and status of the aged in the family. From time to time, changes in the institutions of marriage and family have diminished23 the control of parents over their children. It has increased the freedom of children and they view the aged as a useless and non-productive entity24. Modernization has eventually led to the degradation of their status and authority. Consequently25, the integrity26 of the family and the existence of the elderly as an integral part of the family are being uprooted. The importance of their functional27 positions thus declines and consequently, their authority and much of the respect and prestige that they enjoyed earlier get faded. These changes generally bring about depression in older people.

5 As the old age advances, events at home may also contribute more to their problems. The ‘empty nest’ feeling arising as a result of the grown-up children leaving the home, daughters departing as a result of wedlock and sons leaving station in pursuit28 of higher education or jobs may make the aged more lonely. The loneliness also arises because of premature loss of spouse29. This would deprive the person of a long-standing emotional bond that had provided plenty of emotional succour30 and security. The loss wherever it might occur in the later years leaves the individual terribly lonely and at the mercy of sons and daughters-in-law. Added to these, the increasing gap and interactional stress and strain in the family may leave the elderly without peace of mind. The elderly, as a result of these developments, feel marginalised31, alienated32 and left out of the mainstream. The foregoing are the common problems faced by most of the elderly. These either directly or indirectly lead to a state of depression and make ageing for many an unwanted and unpleasant event to be abhorred33.

6 Usually, the mild depression which is caused due to environmental factors is temporary. The person reconciles34 within a short time and tries to forget the loss. Kind words and timely support of friends, relatives and members of the family help one recover from depression.

Answer each of the questions given below by choosing the most appropriate option :

Question. Endogenous depression arises out of biochemical changes ……… (a) in the brain. (b) in the heart. (c) in our food articles. (d) in the environment.

Question. In ancient cultures, old people were considered as ………. (a) empty nests. (b) big problems. (c) social structures. (d) repositories of wisdom.

Question. In the old age, a person attains ………. . (a) wisdom (b) maturity (c) economic stability (d) all of the three.

Question. Generally, societies tend to show a great respect to ………. (a) the wealthy people. (b) the epic writers. (c) the aged. (d) the depressed people.

Question. The mild depression usually is ………. (a) temporary. (b) very dangerous. (c) incurable. (d) treated by ultra-modern medicines.

Answer the following questions briefly :

Question. What is reactive depression ? Answer : Depression arising out of environmental factors is called reactive depression.

Question. What was the status of the old people in ancient India ? Answer : They had a status of honour and had a recognised social role.

Question. Why were the old people valued ? Answer : They were valued because they could impart knowledge and skill to the youngsters.

Question. What transformation are the social values undergoing ? Answer : They are undergoing transformation from traditional to modern.

Question. What are the causes for the disintegration of joint family system ? Answer : Rapid urbanization and industrialization are the causes for the disintegration of joint family system

Question. How does one recover from mild depression ? Answer : One recovers from mild depression through kind words and timely support of friends, relatives and members of the family.

Question. Pick out the words/phrases from the passage which are similar in meaning to the following : (a) decreased (Para 4) (b) to feel that they do not belong to a group (Para 5) Answer : (a) diminished. (b) alienated.

1 Today, India looks like it is on course to join the league1 of developed nations. It is beginning to establish a reputation not just as the technology nerve centre2 and back office3 to the world, but also as its production centre. India’s secularism and democracy serve as a role model to other developing countries. There is a great pride in an India that easily integrates with a global economy, yet maintains a unique cultural identity. But what is breathtaking4 is India’s youth. For, despite being an ancient civilization that traces itself to the very dawn of human habitation, India is among the youngest countries in the world. More than half the country is under 25 years of age and more than a third is under 15 years of age.

2 Brought up in the shadow of the rise of India’s service industry boom, this group feels it can be at least as good as, if not better than, anyone else in the world. This confidence has them demonstrating a great propensity5 to consume, throwing away ageing ideas of asceticism6 and thrift7. Even those who do not have enough to consume, today feel that they have the capability and opportunity to do so. The economic activity created by this combination of a growing labour pool and rising consumption demand is enough to propel8 India to a double-digit economic growth for decades. One just has to look at the impact that the baby boomers9 in the US had over decades of economic activity, as measured by equity10 and housing prices. This opportunity also represents the greatest threat to India’s future. If the youth of India are not properly educated and if there are not enough jobs created, India will have forever lost its opportunity. There are danger signs in abundance. 

3 Fifty-three per cent of students in primary schools drop out, one-third of children in class V cannot read, three quarters of schools do not have a functioning toilet, female literacy is only 45 per cent and 80 million children in the age group of 6-14 do not even attend school. India’s IT11 and BPO12 industries are engines of job creation, but they still account for only 0.2 per cent of India’s employment. The country has no choice but to dramatically industrialize and inflate its domestic economy. According to a forecast by the Boston Consulting Group, more than half of India’s unemployed within the next decade could be its educated youth. We cannot allow that to happen. India is stuck in a quagmire13 of labour laws that hinder employment growth, particularly in the manufacturing sector. Inflexible labour laws inhibit entrepreneurship14, so it is quite ironic that laws ostensibly15 designed to protect labour actually discourage employment.

4 Employment creation needs an abundant supply of capital. Controls on foreign investment in India have resulted in China getting five times the foreign direct investment, or an advantage of $200 billion over the past five years. The growing interest in India by global private equity firms augurs16 well as they represent pools of patient and smart capital, but they too face many bureaucratic hurdles. When it comes to domestic capital availability, budget deficits adding up to 10 per cent of the national GDP impede capital availability for investment and infrastructure17. Raising infrastructure spending, coupled with rapid privatization, may not only create employment, but also address the growing gaps in infrastructure. China has eight times the highway miles and has increased its roads this cansignificantly in the past few years while India has only inched along. Freight costs at Indian portsare almost double the worldwide average, just to give two examples.

5 Moreover, like the Lilliputians that kept the giant Gulliver tied down, there are some 30,000 statutes18 in India, of which only a portion are even operational, and these keep the employment creation engine tied down. Since there are no sunset19 provisions in any laws, the regulatory morass20 only grows every year. In the meantime, we as citizens of the world and descendants of India have to make a difference. We have to ensure that India and its youth attain that potential, both through our business pursuits and the support of educational charities, on-the-ground proponents21 of participative democracy as well as other deserving organizations and initiatives. I believe that hope can triumph and that this can be India’s century — not one that will happen as surely as the sun will rise each day, but one that many willing hands will need to create together.

Question. The modern Indian youth has a great propensity ……. (a) to save. (b) to earn. (c) to explore. (d) to consume.

Question. What has helped China in getting far more foreign direct investment than India ? (a) Rapid privatisation. (b) Better infrastructure spending. (c) Greater employment opportunities. (d) Fewer controls and bureaucratic hurdles.

Question. The regulatory morass grows every year due to the absence of ……….. (a) sunset provisions. (b) any regulatory laws. (c) educational charities. (d) bureaucratic controls.

Question. Greater economic activity has been generated in India by ………. (a) growing labour pool. (b) rising consumption. (c) both (a) and (b). (d) neither (a) nor (b).

Question. What did the Lilliputians do ? (a) They made Gulliver their king. (b) They kept Gulliver tied down. (c) They loved Gulliver. (d) They killed him.

Question. What hinders employment growth ? Answer :  The quagmire of labour laws in our country.

Question. How can we ensure that India and its youth attain their full potential ? Answer :  It can be done through our business pursuits and the support of educational charities.

Question. How does India serve as a role model to other countries ? Answer :  India serves as a role model on the strength of its secularism and democracy.

Question. What has been said about India’s IT and BPO industries ? Answer :  It has been said that they have created many jobs, but only for 0.2 per cent of employed people.

Question. What enabled China to get such a huge foreign investment ? Answer :  It was because, as compared to India, the controls on foreign investment were far fewer in China.

Question. What does the writer say hopefully in the end of the passage ? Answer :  He believes that hope will triumph and it may prove to be India’s century.

Question. Pick out the words/phrases from the passage which are similar in meaning to the following : (a) to push forward (Para 2) (b) obstruct (Para 3) Answer :  (a) propel. (b) hinder.

unseen passage for class 10 with answers pdf

Related Posts:

Notes and Questions Chapter 2 Forms of Business Organisation

Related Posts

Unseen passages

Unseen Passages

  • Syllabus 2024-25
  • CBSE Class X SQP 2023-24
  • CBSE Class XII SQP 2023-24
  • Class X SQP 2022-23
  • Class XII SQP 2022-23
  • Request Answers

NCERT Tutorials

Reading Comprehension Passages English Grammar CBSE/ICSE

  • Post last modified: 17 June 2022
  • Post category: School Grammar

Comprehension Passages test the intelligence of students in terms of their reading and comprehending i.e. understanding the passages and ability to answer the questions based on such passages. Students should be able to express their thoughts in short simple and lucid manner. Such exercises take away the need to memorise and show the way to write precisely. It also judges the vocabulary of the readers.

Comprehension Passages CBSE Classes

Types of passages.

Two types of passages are given for comprehension in exam.

(i) Case Based (Factual Passage ) (300-350 words): A factual passage composed information in a direct manner about a particular subject. These passages focus completely on details or facts. It may include instructions, a report or a description. It helps the reader to develop a complete idea of a specific person, place, object or being.

(ii) Discursive Passage  (350-400 words): A discursive passage may include the opinion of a person which are generally argumentative, persuasive and interpretative. It allows students to arrive at a conclusion through reasoning and understanding rather than intuition. It presents a balanced and objective approach towards the subject being discussed.

Types of Questions

In the examination, the students are tested on a variety of questions that test their inferential and understanding skills.

In Case Based Factual Passage

  • 8   Objective   Type Questions  (including Multiple Choice Questions) are asked carrying 1 mark each.

In Discursive Passage

  • 4 Short Answer Type Questions  are asked carrying 2 marks each.
  • 4 Objective Type Questions  (including Multiple Choice Questions) to test vocabulary are asked carrying 1 mark each.

Vocabulary based questions may include word formation, meanings, synonyms and antonyms.

Steps to Attempt Reading Comprehension

  • Read each and every line of the passage carefully. Reading the passage twice is always helpful, as it helps a better understanding and makes it easier for the student to find answers.
  • If the title of the passage is given, read it first, as it gives the central idea of the passage.
  • Underline the difficult words while reading the passage, as you might be tested on these words in the very short answer type questions.
  • Always give emphasis on the beginning and end of the passage. These parts often hold the most important information of the passage.
  • While answering, be sure that you’ve clearly understood the question. Answer must be relevant to the question.
  • Ensure that you answer the question according to the marks it carries.
  • Try to use your own language and frame the answer according to the question.
  • Make sure in the answer that you use the same tense in which the question has been asked.
  • Write the question number on each answer very carefully in the answer sheet to avoid mistakes.

How to Answer Vocabulary Based Questions

These questions test the student’s knowledge of words and their meanings in the context they are written in the passage, as well as the meaning of proverbs and idioms commonly used.

The types of questions asked are:

Word Meaning Based:  If you are not sure of the meaning of the word, you will have to guess it from the context in which it is used. Just find a word or a phrase from the options given that makes the same sense as before.

Synonym Based:  Rewrite the sentence in which the word is used, inserting only one word from the options given so that the sentence makes the same sense from before.

Antonym Based:  Rewrite the sentence in which the word is used, inserting only one word from the options given so that the sentence makes the opposite sense from before.

One Word Substitution Based:  Rewrite the sentence in which the word is used, inserting only one word from the options given so that the sentence makes the same sense as before.

Solved Comprehension Passages

Case based factual passages.

Bullet Train in India

Quite recently India laid the foundation stone for one of its most sought-after projects – running a Bullet Train. It was very well considered as a dream project of the Honourable Prime Minister, Narendra Modi. Entire India felt proud of having its first ever bullet train scheduled to run between Mumbai and Ahmedabad, a distance of 508 km, in about 2 hours 35 minutes. In his own words, “To grow, one needs to expand one’s dreams and decide one’s strength to achieve that. It’s the New India which has to fly high”. “Bullet Train is a project that will provide pace to development. Along with new technology, it will also bring results faster”, he added. According to Achal Khare, the Managing Director of the National High Speed Rail Corporation, the project would be completed by December 2023.

It all began with the Prime Minister’s ambitious dream of having a high-speed train in India that cuts the travel time and yet remains an economical option to go from one city to another. The technology of this High-Speed Rail, also known as HSR, was influenced by Japan, which runs a network of bullet trains in their country on Shikasen technology, making many cities well connected to others. According to reports, Japanese Prime Minister Shinzo Abe offered to provide US$ 12 billion of soft loans to build India’s first Bullet Train. The loan was offered at an interest rate of 0.1 per cent per year with repayment over 50 years and a moratorium for the first 15 years. The Japanese Government will be bearing 80 per cent of the total project cost whereas the increase in cost estimates has to be borne by both India and Japan.

The reason this deal between India and Japan is considered path-breaking is that there is currently no financial institution in India that could provide such a huge funding to be repaid over as long a time as 50 years.

 1. The ………………. for the Bullet Train project was laid recently. a) railway track b) signalling system c) foundation stone d) None of these

2. The bullet train will take about ………………. to run between Mumbai and Ahmedabad. a) 3 hours 25 minutes b) 2 hours 55 minutes c) 2 hours 35 minutes d) None of these

3. According to PM Modi, the two benefits that the bullet train will bring are to provide pace to ………………and faster………. a) travel, trains b) train travel, results c) development, trains d) development, results

4. The project is expected to be ………………… by December 2023. a) completed b) operationalised c) started d) None of these

5. What was the Prime Minister’s ambitious dream?

6. Where is the technology for the bullet train coming from?

7. What are the details of the loan offered for this project by Japan?

8. Why is this deal between India and Japan considered as path-breaking?

1. foundation stone

2. 2 hours 35 minutes

3. development, results

4. completed

5. The Prime Minister’s ambitious dream was to have a high-speed train in India that cuts the travel time and yet remains an economical option to go from one city to another.

6. The technology for the bullet train is coming from the bullet trains that run in Japan based on Shikasen technology.

7. The details of the loan offered for this project by Japan are US$ 12 billions of soft loans at an interest rate of 0.1 per cent per year with repayment over 50 years and a moratorium for the first 15 years.

8. The deal between India and Japan is considered path-breaking because currently no financial institution in India can provide such a huge funding to be repaid over as long a time as 50 years.

Mother Teresa: A Humanitarian

Mother Teresa was a humanitarian. This means she did things to help out other people. Her entire life was devoted to helping the poor, the sick, the needy and the helpless.

Mother Teresa was born in Uskub, Ottoman Empire on 26th August, 1910. This city is now called Skopje. Her birth name was Agnes Gonxha Bojaxhiu. Her father died when she was eight and she was raised by her mother. She joined the Sisters of Loreto at the age of 18 to become a missionary in India. She first had to learn English. So she went to Ireland to learn English at the Loreto Abby.

A year later she started her missionary work in Darjeeling, India. She learnt the local language, Bengali, and taught at the local school. She soon took her first vows as a nun and took the name, Teresa.

When she was 36 years old, she felt the call from God to help the poor of India. She received some basic medical training and then set out to help the sick and the needy. This wasn’t an easy task in 1948 India. She had very little support and, while trying to feed and help the poorest of the poor, she herself was constantly hungry and even had to beg for food.

Soon other women joined her and she formed the Missionaries of Charity. Mother Teresa described the purpose of the Missionaries of Charity as an organisation to take care of “the hungry, the naked, the homeless, the crippled, the blind, the lepers, all those people who fed unwanted, unloved and uncared for throughout society.”

It wasn’t an easy task to build such an organistion and to keep the focus on the poorest people. In 1979, Mother Teresa was awarded the Nobel Peace Prize for work undertaken in the struggle to overcome poverty and distress, which also constitutes a threat to peace. She was beatified in 2003, the first step on the path to sainthood, within the Catholic church. She worked almost up until her death on 5th September, 1997.

1. Mother Teresa was called …………. because her entire life was devoted to helping the poor, the sick, the needy and the helpless. a) philanthropic b) a humanitarian c) a sympathiser d) merciful

2. Mother Teresa was born in Uskub, now known as …………., on 26th August, ………………… a) Skopje, 1899 b) Basra, 1911 c) Skopje, 1910 d) None of these

3. She went to Ireland to learn…………. at the Loreto Abby. a) English b) Irish c) Hindi d) Bengali

4. Agnes Gonxha Bojaxhiu changed her to Teresa when she …………… as a nun. a) left Ireland for India b) joined the Sisters of Loreto c) took her vows d) None of these

5. When did she decide to help the poor in India?

6. What hardships did she face while helping the poor?

7. The purpose of the Missionaries of Charity, according to Mother Teresa, is……………

8. Why was Mother Teresa honoured with the Nobel Peace Prize?

1. b) a humanitarian

2. c) Skopje, 1910

3. a) English

4. c) took her vows

5. She decided to help the poor in India when, at the age of 36, she felt the call from God to help the poor in India.

6. The hardships she faced while helping the poor were that she had very little support and, while trying to feed and help the poorest of the poor, she herself was constantly hungry and even had to beg for food.

7. To work as an organisation to take care of “the hungry, the naked, the homeless, the crippled, the blind, the leapers, all those people who feel unwanted, unloved and uncared for throughout society.”

8. Mother Teresa was honoured with the Nobel Peace Prize for work undertaken in the struggle to overcome poverty and distress, which also constitutes a threat to peace.

Discursive Passages

Happiness is like the sun; it is often hidden by the clouds of thoughts, worries and desires. We have to scatter and dissolve them to experience happiness. You don’t have to create happiness. All you have to is calm your mind, because when there is a quiet mind and inner peace, there is happiness. Happiness is not something far away and unattainable. Happiness does not depend on circumstances, objects or events. It is an inseparable part of our consciousness, of our essence, but hidden and covered from sight by our thoughts, desires and worries. The mind is always in a constant race from one thought to another, from one worry to another. It constantly moves from one object or task to another, never standing still. This restlessness hides the happiness that is within you. It is like a choppy sea that hides the bottom. When the sea gets calm, you can see the bottom. In the same way, when the mind gets quiet, you sense the happiness that is within you.

You cannot see a treasure at the bottom of a stormy and muddy lake although it is there. However, when the wind stops, the water becomes still and mud sinks, you can see the treasure. The treasure is there, whether you see it or not. So is happiness. It is always here, only hidden by thoughts, desires and worries.

You can experience more and more happiness in your life. Only your thoughts stand in your way of experiencing it. Next time you feel happy, stop for a moment and watch the state of your mind. You will be surprised to discover that it is calm, and there are almost no thoughts in your mind. Since the mind is not accustomed to stay in this peaceful state for long, it soon becomes active again, and the sense of happiness disappears.

1. What is common between happiness and the sun?

2. How can we enjoy moments of happiness in our life?

3. How does the restlessness of our mind come in the way of our happiness?

4. How can we detect the treasure at the bottom of a stormy and muddy lake?

5. The word………. in para 1 is the synonym of ‘disperse’.

6. The word ……… in para 1 means the same as ‘one and the same’.

7. The word ‘hidden’ in para 2 is an antonym of …………… a) transparent b) evident c) obvious d) visible

8. Which of the following words in para 3 is a synonym of ‘habituated’? a) surprised b) adapted c) accustomed d) peaceful

1. Happiness is like the sun; it is often hidden by the clouds of thoughts, worries and desires. We have to scatter and dissolve them to experience happiness.

2. Restlessness of our mind comes in the way of our happiness by making the mind disturbed, whereas happiness requires a clam mind and inner peace.

3. The mind is always in a constant race from one thought to another, from one worry to another, never standing still. This restlessness hides the happiness that is within us.

4. One cannot see the treasure at the bottom of a stormy and muddy lake although it is there. However, when the wind stops, the water becomes still and the mud sinks, one can see the treasure.

6. inseparable

7. d) visible

8. c) accustomed

Panther and the Game Animals

1. One would imagine that at the very sight of the panther, deer, antelopes and its other preys would just run for their lives. Nothing of the sort. They all stand their ground and make such a loud noise that the panther is left with no other choice except to leave quietly. I have seen a tinny chital baby standing in the middle of an opening in the forest, stamping its feet on the ground and shooting away a tiger. With the white of its erect tail showing, it kept up its shrill call until the tiger made itself scarce. No tiger in its senses would attempt to catch such an impertinent brat, just as you would not dram of catching an offending crow cawing away in your verandah.

2. While the panther sticks to cover and hugs the edge of the forest, the game animals, on the other hand, like to assemble right out in open vast grazing grounds. Open spaces, which the panther carefully avoids, are what the game animals deliberately seek.

3. It is difficult to describe the pandemonium kicked up by various animals when they spot or suspect a panther around. The chital strikes a shrill note, the kakar emits a deafening bark and the sambar rings a bell. The peacock on its perch, the jungle fowl on the ground, and the monkey treetops, all join in the chorus of condemnation of the panther. They curse the panther in their own inimitable language. The resulting confusion of sounds is so irritating to the sharp ears of the panther that it is left with no other option except to go away.

4. The panther has thus to deal with its ever alert and watchful associates who show no mercy and expect none. It is a fight between finesse and flight, between clever and skilful defence.

5. Contrary to the common belief, the panther never springs upon its prey. It stalks as close to its victim as it can manage, and then makes the final dash by rushing at it at lightning speed.

1. What strategy do animals like deer, antelopes etc adopt to drive away the panther?

2. How do the panther and the game animals (deer, antelopes, etc) react to open spaces?

3. What effect does the loud noise made by birds and animals have on the panther?

4. How does the panther kill its prey?

5. The word…….…. in para 1 means the same as “high-pitched and piercing”.

6. The word…………. in para 2 is the synonym of ‘intentionally’.

7. Which word in para 3 is an antonym of ‘praise’? a) pandemonium b) confusion c) condemnation d) irritating

8. Which of the following words is a synonym of ‘springs’ in para 5? a) flies b) leaps c) originates d) evolves

1. The animals like deer, antelope etc. stand their ground and make such a loud noise that the panther is left with no other choice except to leave quietly.

2. The panther carefully avoids open spaces by sticking to cover at the edge of the forest, whereas the game animals deliberately seek such spaces. They like to assemble in vast open grazing grounds.

3. The loud noise made by the birds and animals when they suspect that a panther is around is so irritating to the sharp ears of the panther that it has no option except to go away.

4. The panther kills its prey by first stalking as close as possible to its victim and then makes the final dash by running at it at lightning speed.

6. deliberately

7. c) condemnation

8. b) leaps

Print Friendly, PDF & Email

want to share! Share this content

  • Opens in a new window

You Might Also Like

Modals: class 10 practice exercises, determiners – grammar notes class 10 cbse, rearrange and form meaningful sentences: english grammar, discursive comprehension passages for practice grammar class 9 and 10, leave a reply cancel reply.

Unseen Passage for Class 10 Factual CBSE With Answers

Looking for an easy way to learn English Grammar? then you are in right place. Here we providing basic  English Grammar  topics like Tenses Verbs, Nouns, etc…

Unseen Passage With Answers for Class 10 CBSE

Sample Unseen Passage for Class 10 CBSE – 1 Read the following passage carefully and answer the questions that follow.

High-altitude climbing is still a very dangerous task in spite of the availability of oxygen masks and other protective equipment, which modern climbers take with them. These, of course, are indispensable accessories of climbing, but more important than these is the stamina of the climber, which ultimately determines the success of his attempt. Throughout his journey, death is his constant companion, which he can keep at a distance only with his superb presence of mind.

He has to tread every inch of the ground with utmost care, for a false step may not only strike him a fatal blow, but also bring disaster to the whole expedition. That is why all expeditions invariably take with them local guides who are experienced climbers and who have a thorough knowledge of the nature of the terrain. Moreover, a huge amount of capital is needed for financing these expeditions, and this is generally provided by governments or rich private organisations.

The primary object of a mountaineering expedition is to get to the top of a high mountain, which, in the past has withstood all attempts to conquer it. But it should not be presumed that the expedition is a complete failure if it does not reach its destination. Sometimes operations are temporarily suspended because of bad weather, loss of some valuable equipment or the sudden death of a very important member of the party.

Every big expedition takes with it men who are interested in botany, biology, geology and various other branches of science, and these men carry with them equipment for recording their observations concerning the weather, the terrain, and different forms of life in higher altitudes. Other scientists, explorers and expeditionists utilise the fruits of their observations. Thus, every unsuccessful expedition contributes to the success of later expeditions.

The British Expedition led by Colonel Hunt would have found their way to Everest much more difficult had not earlier expeditions armed them with useful knowledge about the death-dealing weather which they had to encounter in the vicinity of the summit.

To ordinary people, mountaineering need not be a fearfuljourney in the land of snowstorms, where the brave adventurer is always face to face with death. They can scale less ambitious heights, rest their weary limbs under a quiet shelter and feast their eyes in the distant landscape. In the company of friends they can enjoy an outing near a waterfall or cross into the next valley with haversacks full of provisions dangling from their shoulders. All those who can afford to go to hill station should seek this innocent pleasure, for it can be had without any risk to life or limb.

Answer the following questions briefly:

(i) What determines the success of the climbers? (ii) What accessories does a climber have to be equipped with? (iii) How useful are the local guides in these expeditions? (iv) How does an unsuccessful expedition contribute? (v) How is mountaineering different for ordinary people? (vi) Why are expeditions sometimes suspended? (vii) How should the terrain climbers tread every inch of their climb? (viii) What precautions should the climbers take during their journey? Answer: (i) The stamina of the climbers determines the success of their attempt. (ii) A climber has to be equipped with oxygen masks and other protective equipment. (iii) The local guides are experienced climbers and also have a thorough knowledge of the nature of the terrain. (iv) An unsuccessful expedition contributes to the success of later expeditions. (v) They can scale less ambitious heights, rest their weary limbs, feast their eyes in the distant landscape in the company of their friends. (vi) They are suspended due to bad weather, loss of some valuable equipment or sudden death of a very important member. (vii) Terrain climbers should tread every inch of their climb with utmost care. (viii) They should seek guidance of local guides during their journey.

Sample Unseen Passage for Class 10 CBSE – 2 Read the passage carefully.

We give undue importance to our health and the treatment of diseases. A large number of medicines treat only the symptoms of the disease, and not the root cause. In fact, the cause of many chronic ailments is still being researched. It is here that Yoga therapy comes to our assistance. Yoga emphasizes treatment of the root cause of an ailment. It works in a slow, subtle and miraculous manner. Modern medicine can claim to save a life at a critical stage, but, for complete recovery and regaining of normal health, one must believe in the efficiency of Yoga therapy.

The Yogic way of life includes a code of ethics, regulations, discipline, combined with prayer and meditation. Even a discussion of these subjects helps one relieve mental tensions and change attitudes. Simple Asanas help to stretch and relax the whole body and release tensions. The sincere practice of Yoga postures is beneficial , for the mind and body.

The continued practice of Yoga has a profound effect on the inner dimensions of life. Yoga aims at developing the mental, physical, spiritual and emotional facilities. Other formsof physical exercises, like aerobics, assure only physical well-being. They have little to do with the development of the soul and mind.

Answer the following questions briefly: (i) What do most of the medicines treat? (ii) What does the phrase ‘Chronic ailments’ refer to? (iii) How is yoga different from other forms of treatment? (iv) What does the yogic way of life include? (v) How does ‘Simple Asanas’ help? (vi) How does sincere practice of yoga benefit us? (vii) How does yoga therapy work? (viii) Is yoga better than physical exercises? Why? Answer: (i) Most of the medicines treat only the symptoms and not the root cause. (ii) They are Diseases that are recurrent and last, for a long time. (iii) It emphasises on the treatment of the root cause of an ailment. (iv) The yogic way of life code of ethics, regulations includes discipline, prayer and meditation. (v) Simple asanas help to stretch and relax the whole body and release tensions. (vi) Sincere practice of yoga is beneficial for the mind, body and concentration. (vii) It works in a slow, subtle and miraculous manner. (viii) Yoga aims at developing the physical, mental, spiritual and emotional faculties where as physical exercises only develop the body.

Sample Unseen Passage for Class 10 CBSE – 3 Read the passage carefully.

Just by the use of colours you can balance your diet. All you need to do is to pay a little attention to the food you eat. Nutrition experts strongly recommend adding colours to your diet. Sweets and candy bars are generally colourful, but remember they do not contain natural colours and hence are not healthy. The key solution is a variety of naturally coloured foods. The deeper the colour, the greater the benefits.

Getting more colours in your diet doesn’t mean you have to drastically change your current eating habits. Have a glass of 100% juice in the morning. Keep a mix of dried fruits on hand for a quick snack. Grab an apple or banana on your way out. Include at least two vegetables in your dinner. Get into the habit of starting your dinner with a salad. Eat fruit for dessert. Always add greens to sandwiches.

Most red fruits and vegetables contain antioxidants, which offer protection against ultraviolet rays and cancer, and helps to prevent urinary tract infections and diseases related to the circulatory system. Green vegetables along with being appealing possess excellent antioxidant properties that protect the eyes by keeping the retina in good condition and reduce the risk of cancerous tumours.

The orange and yellow group is rich in beta-carotene, an antioxidant that improves cell communication and thereby helps to stop the spread of cancer. While the blue and purple group adds an element of richness to your plate, influences the pineal gland (the third eye) and the nervous system. The white group contains sulphur compounds that protect the DNA and flavonoids, an antioxidant that protects cell membranes.

Therefore, the more colourful your diet is, the better equipped your immune system would be to cope with diseases.

Complete the following sentences: (i) ___________ can balance one’s diet. (ii) Sweets and candy bars ___________ so they are unhealthy. (iii) One should substitute ___________ for pudding at the end of a meal. (iv) Antioxidants help to ___________. (v) The word ‘drastically’ in Paragraph I means ___________ (vi) Green vegetables reduce the risk of ___________. (vii) Blue aind purple vegetables add ___________ in plate. (viii) Colourful diet helps our ___________ to cope with ___________. Answer: (i) use of colours. (ii) do not contain natural colours. (iii) fruits. (iv) offer protection against ultraviolet rays and cancer and help to prevent urinary tract infection and circulatory system diseases (v) sudden, serious or violent measures. (vi) cancerous tumours and keep the retina in good condition. (vii) an element of richness. (viii) immune system; diseases.

Sample Unseen Passage for Class 10 CBSE – 4 Read the passage carefully.

Spices bring to mind images of tempting cuisine, fascinating voyages and the bitter struggle for independence. Expressions like ‘variety is the spice of life’ and ‘sugar and spice and all that is nice’, explain how spices play upon the literary abilities of men of letters.

To orientals, spices are indeed the soul of their food. In the western world, it evokes dreams of exotic tropical islands, exciting expeditions, and the rise and fall of empires. Columbus went westwards in 1492, from Europe, to find a sea route to the land of spices, but found the New World. Eight years later, Vasco da Gama went round Africa and touched Kozhikode on the south west coast of India. Long before that, Arabs started trading with the Orients through land routes. During the 13th century, Marco Polo began the travelling and experienced attraction of spices. The European conquests and trade arrangement in India and the East Indies also have a lot to do with spices.

Thousands of years ago, great masters of Ayurveda notably Susruta and Charaka discussed in detail the uses of spices for culinary and medicinal purposes. Enterprising soldiers of fortune carried the knowledge of spices to Egypt. There they used spices and aromatic herbs in food, medicine, cosmetics and for embalming. The conquest of Egypt and Asia by Alexander the Great, made spices an article of commercial value in the Mediterranean countries and central and northern Europe. There are reports of pepper being used in the preservation of meat, and to mask the unpleasant odour of stale meat available after the long winter storage.

Hippocrates, the father of modern medicine, and Theophrastus, a Greek scholar and botanist, wrote treatises on medicinal plants, including spices.

Answer the following questions briefly: (i) Why do spices fascinate people? (ii) What does “Variety is the spice of life” mean? (iii) Where did Columbus want to reach? (iv) What is meant by ‘cuisine’? (v) What did Susruta and Charaka discuss in detail? (vi) What made spices an article of commercial value in Mediterranean countries? (vii) Who have written treatises on medicinal plants, including spices? (viii) The word in the passage that means ‘interesting’ is ___________. Answer: (i) They bring to mind images of tempting cuisine, fascinating voyages and the bitter struggle for independence. (ii) It means that life becomes exciting when we adopt to change. (iii) Columbus wanted to reach the land of spices. (iv) Refers to different styles of cooking. (v) They discussed the use of spices for culinary and medical purposes in detail. (vi) The conquest of Egypt and Asia by Alexander the Great made species an article of commercial value in Mediterranean countries. (vii) Hippocrates, the father of modern medicine, and Theophrastus, a Greek scholar and botanist, wrote treatises on medicinal plants, including spices. (viii) fascinating.

Sample Unseen Passage for Class 10 CBSE – 5 Read the passage carefully.

Demonetization refers to an economic policy where a certain currency unit leases to be used as a legal tender. It occurs whenever there is a change in the national currency: The existing form or forms of money is pulled from circulation and retired, to be replaced with new notes or coins. Sometimes, a country completely replaces the old currency with new currency.

On 8 November, 2016, the Indian government decided to demonetize the 500 and 1000 rupee notes, the two biggest denominations in its currency system. These notes accounted for 86% of the country’s circulating cash. With little warning, India’s Prime Minister Narendra Modi announced to the citizenry that these notes would be rendered immediately ineffective with immediate effect. People were given time till the end of the year to deposit or exchange them for newly introduced 2000 rupee and 500 rupee notes.

The government’s goal was to combat India’s thriving underground economy on several fronts: eradicate counterfeit currency, fight tax evasion (only 1% of the population pays taxes), eliminate black money accumulated from money laundering and terrorist¬financing activities, and to promote a cashless economy. Individuals and entities with huge sums of black money Acquired from parallel cash systems were forced to take their large- denomination notes to a bank, and account for them satisfactorily and submit proof if tax paid. If the owner could not provide proof of making any tax payments on the cash, he possessed a penalty of 200% on the tax due was imposed.

Demonetisation had a severe impact on the gold market. The extraordinary demand for the yellow metal, made a stiff hike in the cost. However, the Government made it mandatory that every buyer had to submit his/her PAN card details for purchases made. Erring jewelers were brought to book.

Many Indians switched to alternative payment methods. The biggest gainers \yere mobile wallet companies that offered easy transactions through a large network of partners. Alibaba-backed Paytm saw a sevenfold increase in overall traffic. Prepaid cash cards were another option that the customers found useful. Other alternatives were mobile payment systems linked to e-commerce businesses like Ola Money, FreeCharge, Flipkart Wallet.

Answer the following questions briefly: (i) What is meant by demonetization? (ii) When did the government demonetize the 500-and 1000-rupee notes? (iii) What announcement did Prime Minister Narendra Modi make on Nov. 8? (iv) What did the government want to accomplish by the demonetization? (v) When was a penalty of 200% imposed? (vi) Explain the idiom, ‘brought to book’. (vii) Find a word in the passage which means ‘to provide or supply’. (viii) Which were the alternate payment methods available for Indians post demonetization? Answer: (i) Demonetization refers to an economic policy where a certain currency unit leases to be used as a legal tender. (ii) The government demonetized the 500-and 1000-rupee notes on Nov. 8,2016. (iii) Prime Minister Modi announced that the 500- and 1000- rupee notes would be rendered ineffective immediately. The Indian citizenry were given time till the end of the year to deposit or exchange them for the newly introduced 2000 rupee and 500 rupee notes. (iv) The government wanted to eradicate counterfeit currency, fight tax evasion, eliminate black money amassed from money laundering and terrorist-financing activities, and to promote a cashless economy. (v) If the owner could not provide proof of making any tax payments on the cash he possessed, a penalty of 200% on tax due was imposed. (vi) to be punished officially under the law. (vii) Furnish. (viii) The Indians started using the services of mobile wallet companies (Alibaba-backed Paytm) and prepaid cash cards. The other alternatives include mobile payment systems linked to e-commerce businesses like Ola Money, FreeCharge, Flipkart Wallet.

Sample Unseen Passage for Class 10 CBSE – 6 Read the passage carefully.

English is a West Germanic Language that was first spoken in the early medieval England. It has now acquired the status of being the 3rd most widely Spoken Language in the world.

Therefore, the need to be proficient in the language is becoming necessary for every one who has a dream.

How can one gain proficiency in any language? Most certainly by following the LSRW technique, i.e. listening, speaking, reading and writing. Engage with people who are conversant with the language and assimilate what you hear, one at a time. Do not let adversities dampen your progress. “Rome was not built in a day”. Develop your speaking skills gradually. To acquire mastery in the language, one needs to venture on to next step which is extensive reading.

Moving from simple to complex, according to personal preferences. This will help you learn more words and their meanings. Once your vocabulary is developed, experiment with your writing skills. Soon you will find yourself confident, comfortable and fluent with using the language.

Answer the following questions briefly: (i) Pen down two specialities of the English language? (ii) How can we acquire proficiency in a language? (iii) Explain “Rome was not built in a day”. (iv) How can one build up one’s vocabulary? (v) What can dampen your progress? (vi) What is the strategy to be used to develop reading skills? (vii) Find the synonym of – of the middle ages, (in Para 1) (viii) Find the synonym of Skillfulness, (in Para 2) Answer: (i) English is a West Germanic language that was first spoken in the early medieval England. It has now acquired the status of being the 3rd most widely spoken language in the world. (ii) Most certainly by following the LSRW technique, i.e. listening, speaking, reading and writing. Engage with people who are conversant with the language, and assimilate what you hear, one at a time. (iii) This expression means that no task can be accomplished easily or in a short period of time. (iv) Reading and understanding word meanings will develop one’s vocabulary. (v) Adversities can dampen your spirits. (vi) Moving from simple to complex, according to personal preferences. This will help you learn more words and their meanings. (vii) Medieval (viii) Proficiency

Sample Unseen Passage for Class 10 CBSE – 7 Read the passage carefully.

A Norwegian Mathematician Vejomo-Johansson of the KTH Royal Institute of Stockholm, Sweden developed an interest in a unique concept — the technique of knotting a tie and recorded 17,147 of doing so. It is curiosity on the subject increased after viewing a video on YouTube by a film celebrity, on the technique of knotting a tie adopted by the characters in the Matrix film series and in Merovingian series.

To gain further knowledge in the subject, his research led him to the earlier findings of 85 techniques that were recorded at the University of Cambridge, by the experts Yong Mao and Thomas Fink. They had not considered the fancy styles in their study which accounted for the poor count.

They had considered only the normal, traditional and conventional ways of knotting a tie and had used ordinary everyday language to define their work. Johansson discovered that this common methodology was not used in the Matrix series, that involved several tucks, many folds and edges. He eventually set about rewriting the process by including numerous expressions that explained more elaborately different ways of knotting a tie.

Subsequently, tie wearers have been provided with a variety of options to choose from to complete their office attire.

(i) What is the researcher referring to in the passage? (ii) To which faculty does the researcher belong and what is his name? (iii) Who were the researchers from Cambridge? (iv) Mention the drawbacks of their research. (v) What inspired Johansson to do research? (vi) What special technique was used in the Matrix series? (vii) Identify the word that means special and unusual in para 1. (viii) Identify the word need for ‘describe in a detailed manner,’ in para 3. Answer: (i) He is referring to the technique of knotting a tie. (ii) He is a Mathematician by the name Vejomo Johansson. (iii) the researchers from Cambridge were Yong Mao and Thomas Fink. (iv) They had not considered the fancy styles of knotting a tie. (v) The Cambridge researchers got a poor count, and had used everyday language to define their work. This discovery inspired Johansson to do research. (vi) Matrix series included many tucks, several folds and edges. (vii) Unique (viii) Elaborately.

Sample Unseen Passage for Class 10 CBSE – 8 Read the following passage carefully.

My grandmother’s house is like a chambered sea shell it has many rooms, yet it is not a mansion. Its proportions are small and its design simple. It is a house that has grown organically, according to the needs of its inhabitants. To all of us in the family it is known as la casa de Mama. It is the place of our origin ; the stage for our memories and dreams of Island life.

I remember how in my childhood it sat on stilts ; this was before it had a downstairs-it rested on its perch like a great blue bird-not a flying sort of bird, more like a nesting hen, but with spread wings. Grandfather had built it soon after their marriage. He was a painter and house builder by trade-a poet and meditative man by nature. As each of their eight children were born, new rooms were added. After a few years, the paint didn’t exactly match, nor the materials, so that there was a chronology to it, like the rings of a tree, and Mama could tell you the history of each room in her casa, and thus the genealogy of the family along with it.

Her own room is the hjeart of the house. Though I have seen it recently- and both woman and room have diminished in size, changed by the new perspective of my eyes, now capable of looking over countertops and tall beds-it is not this picture I carry in my memory of Mama’s casa. Instead, I see her room as a queen’s chamber where a small woman loomed large, a throne room with a massive four poster bed in its center, which stood taller than a child’s head.

It was on this bed, where her own children had been born, that the smallest grandchildren were allowed to take naps in the afternoons ; here too was where Mama secluded herself to dispense private advice to her daughters, sitting on the edge of the bed, looking down at whoever sat on the rocker where generations of babies had been sung to sleep. To me she looked like a wise empress right out of the fairy tales I was addicted to reading.

And there was the monstrous wardrobe she kept locked with a little golden key she did not hide. This was a test of her dominion over us ; though my cousins and I wanted a look inside that massive wardrobe more than anything, we never reached for that little key lying on top of her Bible on the dresser. This was also where she placed her earrings and rosary when she took them off at night. God’s word was her security system. This wardrobe was the place where I imagined she kept jewels, satin slippers, and elegant silk, sequined gowns of heartbreaking fineness.

I lusted after those imaginary costumes. I had heard that Mama had been a great beauty in her youth, and the belle of many balls. My cousins had ideas as to what she kept in that wooden vault : its secret could be money (Mama’s did not hand cash to strangers, banks were out of the question, so there were stories that her mattress was stuffed with dollar bills, and that she buried coins in jars in her garden under rose-bushes, or kept them in her untouched wardrobe there might be that legendary gun salvaged from the Spanish American conflict over the Island. We went wild over suspected treasures that we made up simply because children have to fill locked trunks with something wonderful.

(i) What was the la casa de Mama in the family? (ii) What did the changing face of the house reflect? (iii) Briefly explain the structure of Grandmother’s house. (iv) How did the narrator’s Grandmother assert herself? (v) Why did Mama not keep the little golden key hidden? (vi) What had the author heard of Mama as a young woman? (vii) Find the word from the passage – authority/control (para 4) (viii) Find the word from the passage – a string of beads to count prayers (para 4) Answer: (i) To all of us in the family it is known as la casa de Mama. It is the place of our origin; the stage for our memories and dreams of Island life. (ii) In his childhood, it sat on stilts; this was before it had a downstairs—it rested on its perch like a great blue bird-not a flying sort of bird, more like a nesting hen, but with spread wings. As each of their eight children was born, new rooms were added. So that there was a chronology to it, like the rings of a tree. (iii) My grandmother’s house is like a chambered sea shell; it has many rooms, yet it is not a mansion. Its proportions are small and its design simple. It is a house that has grown organically, according to the needs of its inhabitants. (iv) There was the monstrous wardrobe Mama kept locked with a little golden key she did not hide. This was a test of her dominion over us. (v) We never reached for tha) little key lying on the top of her Bible on the dresser. God’s word was her security system. (vi) I had heard that Mama had been a great beauty in her youth, and the belle of many balls. (vii) Dominion (viii) Rosary.

Sample Unseen Passage for Class 10 CBSE – 9 Read the passage carefully.

In the early 1920’s, settlers came to Alaska looking for gold. They travelled by boat to the coastal towns of Seward and Knik, and from there by land into the gold fields. The trail they used to travel inland is known today as the Iditarod Trail, one of the National Historic Trails designated by the Congress of the United States. The Iditarod Trail quickly became a major thoroughfare in Alaska, as the mail and supplies were carried across this Trail. People also used it to get from place to place, including the priests, ministers, and judges who had to travel between villages. In the winter, the settlers’ only means of travel down this trail was via dog sled.

Once the gold rush ended, many gold-seekers went back to where they had come from, and suddenly there was much less travel on the Iditarod Trail. The introduction of the airplane in the late 1920’s meant dog teams were no longer the standard mode of transportation, and of course with the airplane carrying the mail and supplies, there was less need for land travel in general. The final blow to the use of the dog teams was the appearance of snowmobiles.

By the mid 1960’s, most Alaskans didn’t even know the Iditarod Trail existed, or that dog teams had played a crucial role in Alaska’s early settlements. Dorothy G. Page, a self- made historian, recognized how few people knew about the former use of sled dogs as working animals and about the Iditarod Trail’s role in Alaska’s colourful history. To raise awareness about this aspect of Alaskan history, she came up with the idea to have a dog sled race over the Iditarod Trail. She presented her idea to an enthusiastic musher, as dog sled drivers are known, named Joe Redington, Sr. Soon the Pages and the Redingtons were working together to promote the idea of the Iditarod race.

Many people worked to make the first Iditarod Trail Sled Dog Race a reality in 1967. The Aurora Dog Mushers’ Club, along with men from the Adult Camp in Sutton, helped clear years of overgrowth from the first nine miles of the Iditarod Trail. To raise interest in the race, a $25,000 purse was offered, with Joe Redington donating one acre of his land to help raise the funds. The short race, approximately 27 miles long, was put on a second time in 1969.

After these first two successful races, the goal was to lengthen the race a little further to the ghost town of Iditarod by 1973. However in 1972, the U.S. Army reopened the trail as a winter exercise, and so in 1973, the decision was made to take the race all the way to the city of Nome- over 1,000 miles. There were many who believed it could not be done and that it was crazy to send a bunch of mushers out into the vast, uninhabited Alaskan wilderness. But the race went on! Twenty-two mushers finished that year, and to date over 400 people have completed it.

(i) Mention two ways by which the Iditarod trail was used. (ii) How was the dog team replaced? (iii) Who created an awareness of Alaskan history? (iv) What was the role of the US army? (v) Who is a musher? . (vi) What is the role of the US army in the Iditarod Trail Sled Dog Race? (vii) Give the meaning of following word: Designated (viii) Through fare (Para 1) Answer: (i) The Iditarod Trail quickly became a major throughfare in Alaska, as the mail and supplies were carried across this. Trail. People also used it to get from place to place. (ii) The introduction of the airplane in the late 1920’s meant dog teams were no longer the standard mode of transportation. The final blow to the use of the dog teams was the appearance of snowmobiles. (iii) Dorothy G. Page, a self-made historian, raised awareness about the Iditarod trail role of Alaskan history. She came up with the idea to have a dog sled race over the Iditarod Trail. (iv) In 1972, the U.S. Army reopened the trail as a winter exercise, and so in 1973, the decision was made to take the race all the way to the city of Nome over 1,000 miles. (v) A dog sled driver. (vi) Cleared the entire trail of about 1,000 miles all the way to Nome. (vii) Chosen/selected. (viii) Main road.

Sample Unseen Passage for Class 10 CBSE – 10 Read the passage carefully.

1. After water, tea is the most popular beverage in the world. Its popularity has survived thousands of years and has played an important role in many cultures. It is enjoyed both hot and cold, as a refreshing drink, as part of a ceremony, or as a tonic for improved health.

2. The drink of Asia for hundreds of years, tea is believed to have been brought to Europe by the Dutch. Today, from remote Ladakh in India to Buckingham Palace in. London, tea is synonymous with cheer. It is rightly said that there will be no agreement on a perfect cup of Jea. Though for tea drinkers the brew is addictive, the preferred method of preparation and taste differ from person to person and region to region. From traditional black teas, to the newer, and extraordinarily healthy white teas, and recognisable flavoured teas such as Earl Grey, to exotic blends such as Rooibos Love, there is a flavour and a blend for everyone. Today many varieties of tea and tea brands are available in the market. An innovation is the tea-bag that is easy, quick and less messy than traditional ways of brewing tea. Green tea is popular in China and the Far East.

3. In Japan, the tea ceremony is a traditional way of greeting guests and is a social occasion. Unlike the tea we are familiar with, green tea is not drunk with sugar or milk. It is an olive-coloured liquid served in porcelain cups. In Morocco, green tea is infused with freshly plucked mint.

4. Some scientists believe tea prevents tooth decay because it is a rich source of fluoride.

5. Tea is also a folk remedy for stomach upsets, flu and diarrhoea. Tea is also said to have antioxidants that fight cancer and also has anti-ageing properties ‘that fight the free radicals in our bodies. Research suggests that drinking tea reduces the risk of heart disease and cancer and lowers cholesterol levels in the blood.

6. A welcome thought for inveterate tea drinkers. Tea is the new apple-a-day to keep the doctor away.

(i) Who introduced tea to Europe? (ii) What does the line “Tea is synonymous with cheer” mean? (iii) What is the new and convenient way of making tea? (iv) Where is the tea ceremony a way of greeting guests? (v) How are the fluoride useful in tea? (vi) Whom do you call ‘an inveterate tea drinker’? (vii) What are the benefits of drinking tea? (viii) Where is ‘Green tea’ popular? Answer: (i) The Dutch introduced tea to Europe. (ii) A cup of tea makes people healthy and happy. (iii) New and convenient way of making tea is by using tea bags. (iv) In Japan tea ceremony is a way of greeting guests. (v) Fluoride in tea prevents tooth decay. (vi) An inveterate tea drinker is one who uses tea as a beverage habitually or is a habitual tea drinker. (vii) It is a remedy for stomach upsets, flu and diarrhoea. It is an antioxidant that fights cancer and also has anti-ageing properties that fight the free radicals in our bodies. (viii) Green tea is popular in China.

Sample Unseen Passage for Class 10 CBSE – 11 Read the passage carefully.

Education of a child starts from the family and mother is the first teacher. But, the irony in India is that although the deity of education is a female i.e. Goddess Saraswati according to the Hinduism, innumerable number of women are illiterate. They do not remain uneducated by their own wish but they are forbidden from receiving education because of the patriarchal system in our society. Right from the early Vedic period people have been celebrating the birth of a son, but in those days daughters born into a family were not neglected but were educated properly.

However, the scene changed during the later Vedic Age and the daughters were considered a social burden. Only the girls belonging to upper class families enjoyed the right of education and got proper nourishment. In medieval period the conditions deteriorated for the females and even in royal families. Girl could not get the same status as the boys. In Muslim households they were taught at their homes while Hindu girls enjoyed the privilege of getting primary education along with the boys in schools. The prevalence of child marriage was excessively practised.

However, in the nineteenth century many social reformers like Raja Ram Mohan Roy, Sir Syed Ahmed Khan, Annie Besant, M. G. Ranade, Joytiba Phule, Swami Dayanand Saraswati etc. came forward for the emancipation of women in India. Especially Raja Ram Mohan Roy advocated female education.

Since then there has been tremendous progress in every field but unfortunately still the girls are neglected. In most of the families the birth of a girl child is not desired and if accepted they are thought inferior to boys and their education is not considered important because it seems a wastage of money to most of the parents. They think it unreasonable because afterwards they would be compelled to spend a heavy amount on their dowry.

So the female literacy rate is unsatisfactory and it has a direct impact upon the overall development of a nation and its population growth. If India wants to be one of the developed nations it must concentrate on female education because if we educate a man, we educate an individual but if we educate a woman we educate an entire family. Again the root cause of all problems facing the women is lack of education.

If all women are educated, then all problems like female infanticide, dowry, female suicides, domestic battering, malnutrition of women, child marriage and other related atrocities would vanish from India. Education provides an essential qualification to fulfill certain economic, political and cultural functions and improves women’s socio economic status. It brings reduction in inequalities. If their standard of living is improved, it will indirectly uplift the level of society. If they are financially strong they will take proper care of their children and provide them with good education.

(i) Why were women not allowed to receive education? (ii) Why was it considered unnecessary? (iii) What are the advantages of educating a girl? (iv) What are the problem that women face in India? (v) What is the irony about education in India? (vi) How can education help women? (vii) Find the words from the passage which means the same as declined (para 2) (viii) Find the words from the passage which means the same as cruelties (para 3) Answer: (i) They do not remain uneducated by their own wish but they are forbidden from receiving education because of the patriarchal system in our society. (ii) They think it unreasonable because afterwards they would be compelled to spend a heavy amount on their dowry. (iii) If we educate a man, we educate an individual but if we educate a woman we educate an entire family. (iv) Women in India face problems like female infanticide, dowry, female suicides, domestic battering, malnutrition of women, child marriage and other related atrocities. (v) the irony in India is that although the deity of education is a female i.e. Goddess Saraswati according to the Hinduism, innumerable number of women are illiterate. (vi) Education of women can do away with the problems faced by women and also help in the development of the country. (vii) Deteriorated. (viii) Atrocities.

Sample Unseen Passage for Class 10 CBSE – 12 Read the poem given below and answer the questions that follow.

On the door step of the temple Under the hot sun I see her everyday. Her hands and feet are Withered like limbs of a dead tree.

Her eyes tired of searching For rays of hope Are sunk deep like caves In a rock. Neither they receive light Nor give it.

Her three yard saree Has torn in thirty places Her body has absorbed. Sixty years of dirt.

Her knotty matted hair Is the abode of worm and lice Here and there some black teeth, Stand as guards in her open mouth. The lines on her face. Have spread far and deep. Like the crow’s feet.

Raising her shaking hands With hope in her eyes She looks at me And an electric current joins together our two hearts Time goes back fifteen years. Her face changes, her eyes twinkle When she smiles her teeth dazzle In that one magic moment, I I see the image of my dead mother

(i) Where does the poet see her? (ii) What has happened to her hands and feet? (iii) What do her eyes search for? (iv) How does she look at the poet? (v) What does he say about her dress and body? (vi) Describe her teeth and mouth. (vii) What happens when she looks at the poet? (viii) What is the magic that follows? Answer: (i) The poet sees her on the door step of the temple under the hot sun. (ii) Her hands and feet are withered like limbs of a dead tree. (iii) Her eyes were searching for rays of hope. (iv) She looks at the poet with hope in her eyes. (v) He says that she is wearing a very dirty and tattered saree. (vi) Her teeth have become black in colour and they stand as guards in her open mouth. (vii) When she looks at the pqet, she experiences an electric current passing through her heart and her mind flashes back in time. (viii) The poet sees the image of his dead mother in that magic moment.

Sample Unseen Passage for Class 10 CBSE – 13 Read the following passage carefully:

The time is now to save these magnificent cats, before they vanish from the earth forever. There is little time left to preserve these animals; immediate action is required. The latest predictions state that tigers will be extinct in the wild by the year 2015. That is less than 04 short years from now. That’s less than 04 years to save a creature that has been around for longer than us.

I cannot imagine, nor would I wish to live in a world without tigers… without these magnificent creatures in our forests living free – without captive populations carrying on the gene pool. The power and beauty of these animals is indisputable. Tigers are an integral part of the ecosystem, as they are a major link in the food chain. Tigers have been bestowed magical and supernatural properties by many societies, and have also been revered as gods. All living creatures of mother earth depend on one another for survival. We as human beings are brothers and sisters to every living being, from the plants to the – animals to each other.

Crimes like these – the killing of tigers – must be stopped. They are often killed in the most barbaric, the most unbelievably cruel ways. I ask you- the reader – if learning of things like this moves you at all, then take that energy and do what you can to stop the killing of these magnificent animals. Write to your government, write to the governments of tiger countries and urge them to protect tigers. Penalties for killing tigers, and other protected wildlife need to be strongly enforced – and strong enough as to be a complete deterrent to the poaching of tigers, and also deter those who profit from such killings.

Extremist groups would ban all exotic animals from captivity because of small numbers of individuals that mistreat their animals, and unfortunately the whole zoo “industry” and caring private owners get painted with the same brush. The vast majority of zoos, big and small, treat their animals well, feed them well and give them proper environment and enrichment. The numbers of bad homes are small, but they do exist. The Animal Rights Organizations should make sure they target only the bad homes and leave those providing a good, happy and healthy home for animals alone.

They don’t realize that banning all exotics from captivity could spell the end for many species of animals. Species whose numbers in the wild are extremely low, such as the Clouded Leopard, Siberian tiger, South China tiger and so forth. Zoos, sanctuaries and responsible, knowledgeable and caring private owners are rapidly becoming the last havens to preserve ever – dwindling species of animals. These places may well, in the not so far future, hold the last remaining tigers – gems for the future who should be nurtured and protected.

Caring for a tiger is an immense responsibility. Not just to the single tiger in your care, but to his or her species, and all tigers. Simply to own a tiger and keep him or her in your backyard as a pet is not enough, nor is it acceptable to keep this animal to yourself. You, the owner, owe it to all tigers – wild and those held in captivity – and to the future generations of these animals to educate the public about tigers – and help to change people’s opinions about wildlife preservation. Education is key to developing sensitivity to and awareness of our environment. Our children hold the hope for the future of the tiger. Heightened awareness of environmental issues will help assure our children grow up to respect and care for our global environment.

Think Globally – Act Locally!

(i) What is the need of the hour? (ii) How can we save tigers from extinction? (iii) Why should penalties be imposed? (iv) What are tigers known for? (v) How can you say that the tigers are on the verge of extinction? (vi) What role can common people play in the conservation of tigers? (vii) Find the word from the passage which means the same as hindrance (para 3) (viii) Find the word from the passage which means the same as refuge (para 5) Answer: (i) It is the need of the hour to take immediate action to save magnificent cats. (ii) Write to the governments of countries and urge them to protect tigers. (iii) They are a deterrent to the poaching of animals. (iv) Power and beauty. (v) The latest predictions state that the tigers could get extinct by the year 2015 that is less than 04 short years now. (vi) They can educate the public about tigers and change people’s opinion about wild life preservation. (vii) Deterrent (viii) Haven

Sample Unseen Passage for Class 10 CBSE – 14 Read the following passage carefully:

It’s not “cool” to be fat, but that has not prevented an obesity epidemic from occurring among America’s youth. Childhood obesity increased from 5 percent in 1964 to about 13 percent in 1994. Today, it is about 20 percent – and rising. Excessive time spent watching television, using the computer, and playing video games is partly to blame for this escalating rate. Children, on an average, spend up to five to six hours a day involved in these sedentary activities. Perhaps it wouldn’t matter if they were sufficiently active at other times, but most of them aren’t.

To make matters worse, children are bombarded with well – crafted TV ads from fast – food chains and other purveyors of high – fat, high – sugar meals and snacks. A recent study reported that two-to-six-year-olds who watch television are more likely to choose food products advertised on TV than children who do not watch such commercials. These highly effective advertising campaigns, combined with a physically inactive lifestyle, have produced a generation of kids who are at high risk for obesity-associated medical conditions.

The major health threat is the early development of Type 2 diabetes (adult onset), particularly in children with a family history of disease. Doctors are reporting a surge in young adolescents developing Type 2 diabetes – which can lead to heart disease, high blood pressure, kidney disease, stroke, limb amputations, and blindness. People who develop diabetes in adolescence face a diminished quality of life and shortened life span, particularly if the disease progresses untreated. It’s a scary prospect for our children but, in many cases, obesity and diabetes are preventable.

When children are spending most of their free time sitting in front of televisions and computers, they are not outside running, jumping or engaging in team sports that would keep their weight down. Parents need to set limits on the time their children are engaged in passive activities. Pediatricians recommend restricting children to one to two hours per day on TV and computers combined – though older children may need additional time for learning activities.

Parental involvement remains the most important key to our children’s healthy diets. Programs to educate parents about nutrition are essential. Fast foods should be consumed only in moderation. Caregivers, who are often busy and harried, must avoid the temptation to whisk their kids into fast-food restaurants or to pick up fast food for dinner at home. Changing eating habits and lifestyles is not easy, but the health benefit for our children is a wonderful payoff for parents willing to take on the task.

Answer the following questions briefly.

(i) What is the main cause of obesity? (ii) Who does Type 2 diabetes normally affect and what does it lead to? (iii) What should the parents do to check obesity? (iv) What do pediatricians suggest? (v) How are advertisements responsible for an obesity-associated medical problem? (vi) How does diabetes affect adolescents? (vii) Find the word from the passage which means as Inactive (para 1) (viii) Find the word from the passage which means as Declined (para 3) Answer: (i) Sedentary life style. (ii) With a family history of the disease. It leads to heart disease, high blood pressure, kidney disease, blindness, etc. (iii) Set limits on the time their children are engaged in passive activities. (iv) Paediatricians suggest that the most important factor to improve our children’s diet is to take healthy and balanced food. (v) Children who watch advertisement on Television are more likely to choose advertised fast food products which lead to obesity associated medical problems. (vi) They face a diminished quality of life and shortened life span. It can lead to many diseases also. (vii) Sedentary (viii) Diminished.

Sample Unseen Passage for Class 10 CBSE – 15 Read the following poem carefully:

The Fallen Leaves -Robert Service

Why should I be the first to fall Of all the leaves on this old tree? Though sadly soon I know that all Will lose their hold and follow me. While my birth brothers bravely blow, Why should I be the first to go? Why should I be the last to cling Of all the leaves on this bleak bough? I’ve fluttered since the fire of spring And I am worn and withered now. I would escape the Winter gale. And sleep soft silvered by a Snail. When swoop the legions of the snow To pitch their tents in roaring weather We fallen leaves will lie below And rot rejoicingly together; And from our rich and dark decay Will laugh our brothers of the May.

Answer the following questions on the basis of your reading of the above poem:

(i) Who is complaining? (ii) Which season is being referred to here? (iii) What is the complaint of the last leaf? (iv) From when have the leaves been fluttering? (v) What does the snail do? Which season is mentioned? (vi) How do the fallen leaves react now? (vii) Why would they escape the Winter gale? (viii) Explain the last 2 lines. Answer: (i) The 1st leaf to fall off the tree is complaining. (ii) Autumn is the season mentioned in the poem. (iii) The last leaf does not like being the last leaf on the bough. (iv) They’ve been fluttering from the onset of spring. (v) The snail sleeps beside the fallen leaves, looking like silver because of the snow on its shell. (vi) They lie below the snow and rot rejoicingly together. (vii) They would escape the Winter gale as they are already fallen and covered by the snow. (viii) From the manure that is created by the rotten leaves, new plants will be born in May with the spring.

Sample Unseen Passage for Class 10 CBSE – 16 Read the poem carefully:

A treacherous monster is the Shark He never makes the least remark.

And when he sees you on the sand, He doesn’t seem to want to land.

He watches you take off your clothes, And not the least excitement shows.

His eyes do not grow bright or roll, He has astounding self-control.

He waits till you are quite undrest, And seems to take no interest And when towards the sea you leap, He looks as if he were asleep.

But when you once get in his range, His whole demeanour seems to change.

He throws his body right about, And his true character comes out.

It’s no use crying or appealing, He seems to lose all decent feeling.

After this warning you will wish To keep clear of this treacherous fish.

(i) Why does the shark not make any remark? (ii) How do you know he has self-control? (iii) How does his demeanor change? (iv) How does he appear when you leap into the sea? (v) How does he react when you come into his range? (vi) Can you appeal to him? (vii) What is the meaning of demeanor on the basis of stanza? (viii) What is the meaning of treacherous on the basis of stanza? Answer: (i) Because he is a treacherous Monster. (ii) His eyes do not roll or become bright when you take off your clothes. (iii) When you get into his range, his demeanour changes. (iv) He appears to be asleep. (v) He will throw his body right about. (vi) Once he gets into his true character, there is no use of crying or appealing to him. (vii) Behaviour (viii) Deceit

Sample Unseen Passage for Class 10 CBSE – 17 Read the following passage carefully.

1. Most people can remember a phone number for up to thirty seconds. When this short amount of time remove of, however, the numbers are erased from the memory. How did the information get there in the first place? Information that makes its way to the short-term memory (STM) does so via the sensory storage area. The brain has a filter which only allows stimuli that is of immediate interest to pass on to the STM, also known as the working memory.

2. There is much debate about the capacity and duration of the short- term memory. The most accepted theory comes from George A. Miller, a cognitive psychologist who suggested that humans can remember approximately seven chunks of information. A chunk is defined as a meaningful unit of information, such as a word or name rather than just a letter or number. Modern theorists suggest that one can increase the capacity of the short-term memory by chunking, or classifying similar information together. By organizing information, one can optimize the STM, and improve the chances of a memory being passed on to long-term storage.

3. When making a conscious effort to memorise something, such as information for an exam, many people engage in “rote rehearsal”. By repeating something over and over again, one is able to keep a memory alive. Unfortunately, this type of memory maintenance only; succeeds if there are no interruptions. As soon as a person stops rehearsing the information, it has the tendency to disappear. When a pen and paper are not handy, people often attempt to remember a phone number by repeating it aloud.

If the doorbell rings or the dog barks to come in before a person has the opportunity to make a phone call, he will likely forget the number instantly. Therefore, rote rehearsal is not an efficient way to pass information from the short-term to long¬term memory. A better way is to practise “elaborate rehearsal”. This involves assigning semantic meaning to a piece of information so that it can be filed along with other pre¬existing long-term memories.

4. Encoding information semantically also makes it more retrievable. Retrieving information can be done by recognition or recall. Humans can easily recall memories that are stored in the long-term memory and used often; however, if a memory seems to be forgotten, it may eventually be retrieved by prompting. The more cues a person is given (such as pictures), the more likely a memory can be retrieved. This is why multiple-choice tests are often used for subjects that require a lot of memorisation.

(i) According to the passage, how do memories get transferred to the STM? (ii) Where can memory be stored? (iii) Why does the author mention a dog’s bark? (iv) What is rote rehearsal? Is it useful? (v) How do theorists believe a person can remember more information in a short time? (vi) How can a lost memory be retrieved? (vii) What is ‘remove of rehearsal’? (viii) What is an efficient way to pass information from the short-term to long-term memory and how is it efficient? Answer: (i) They are filtered from the sensory storage area. (ii) Memory can be stored in the short-term memory, long-term memory, and sensory storage area. (iii) It’s an example of a type of interruption. (iv) When making a conscious effort to memorize something, such as information for an exam, many people engage in “rote rehearsal”.. By repeating something over and over again, one is able to keep a memory alive. Unfortunately, this type of memory maintenance only succeeds if there are no interruptions. As soon as a person stops rehearsing the information, it has the tendency to disappear. (v) It can be done by organizing the information. (vi) A lost memory can be retrieved by prompting. The more cues a person is given (such as pictures), the more likely a – memory can be retrieved. (vii) It is assigning semantic meaning to a piece of information so that it can be filed along with other pre-existing long¬term memories. (viii) ‘Elaborate rehearsal’ is an efficient way to pass information from the short-term to long-term memory. This involves assigning semantic meaning to a piece of information so that it can be filed along with other pre-existing long-term memories.

Sample Unseen Passage for Class 10 CBSE – 18 Read the following passage carefully.

1. Walter Elias “Walt” Disney was born on December 5, 1901, in Hermosa, Illinois. He lived most of his childhood in Marceline, Missouri, where he began drawing, painting and selling pictures to neighbours and family friends. Disney attended McKinley High School in Chicago, where he took drawing and photography classes and was a contributing cartoonist for the school paper. At night, he took courses at the Chicago Art Institute.

2. When Disney was 16, he dropped out of school to join the army but was rejected for being underage. Instead, he joined the Red Cross and was sent to France for a year to drive an ambulance. When Disney returned from France in 1919, he moved back to Kansas City to pursue a career as a newspaper artist. His brother Roy got him a job at the Pesmen-Rubin Art Studio, where he met cartoonist Ubbe Eert Iwwerks, better known as Ub Iwerks. From there, Disney worked at the Kansas City Film Ad Company, where he made commercials based on cutout animation. Around this time, Disney began experimenting with a camera, doing hand-drawn cel animation, and decided to open his own animation business. From the ad company, he recruited Fred Harman as his first employee.

3. Walt and Harman made a deal with a local Kansas City theater to screen their ‘ cartoons, which they called Laugh-O-Grams. The cartoons were hugely popular, and Disney was able to- acquire his own studio, upon which he bestowed the same name. Laugh-O-Gram hired a number of employees, including Harman’s brother Hugh and Iwerks. They did a series of seven-minute fairy tales that combined both live action and animation, which they called Alice in Cartoonland. By 1923, however, the studio had become burdened with debt, and Disney was forced to declare bankruptcy.

4. Disney and his brother, Roy, soon pooled their money and moved to Hollywood. Iwerks also relocated to California, and there the three began the Disney Brothers’ Studio. Their first deal was with New York distributor Margaret Winkler, to distribute their Alice cartoons. They also invented a character called Oswald the Lucky Rabbit, and contracted the shorts at $1,500 each.

(a) What did Walt Disney learn in his childhood? (b) Why was Walt not selected in the army? (c) How did Walt’s brother help him in Kansas? (d) What did Walt learn at Kansas City Film Ad Company? (e) Name the first employee of Ad Company. (f) What deal was made with the Kansas City theatre? (g) Write briefly about Alice in the Cartoonland. (h) What was the deal made with Margaret Winkler? Answer: (a) In his childhood, Walt Disney learnt drawing, painting and selling pictures to neighbours and family friends. (b) Walt was not selected in the army because he was underage. (c) Walt’s brother got him a job at the Pesmen-Rubin Art Studio, where he met cartoonist Ubbe Eert Iwwerks, better known as Ub Iwerks. (d) Walt learnt to make commercials based on cutout animation. (e) Fred Harman was the first employee of the Ad company. (f) The deal was to screen their cartoons, which they called Laugh-O-Grams. (g) Alice in Cartoonland was a series of seven-minute fairy tales that combined both live action and animation. (h) The deal made with Margaret Winkler was to distribute their Alice cartoons.

Sample Unseen Passage for Class 10 CBSE – 19 Read the following passage carefully:

One of the greatest advances in modern technology has been the invention of computers. They are widely used in industries and in universities. Now there is hardly any sphere of human life where computers have not been pressed into service of man. We are heading fast towards the day when a computer will be as much part of man’s daily life as a telephone – or a calculator.

Computers are capable of doing extremely complicated work in all branches of learning. They can solve the most complex mathematical problems or put thousands of unrelated facts in order. These machines can be put to varied uses. For instance, they can provide information on the best way to prevent traffic jams. This whole process by which machines can be used to work for us has been called ‘automation.’ In the future automation may enable human beings to enjoy more leisure than they do today. The coming of automation is bound to have important social consequences.

Some years ago an expert on automation, Sir Leon Bagrit, pointed out that it was a mistake to believe that these machines could ‘think.’ There is no possibility that human beings will be “controlled by machines.” Though computers are capable of learning from their mistakes and improving on their performance, they need detailed instructions from human beings to operate. They can never, as it were, lead independent lives or “rule the world” by making decisions of their own.

Sir Leon said that in future, computers would be developed which would be small enough to carry in the pocket. Ordinary people would then be able to use them to obtain valuable information; Computers could be plugged into a national network and be used like radios. For instance, people going on holiday could be informed about weather conditions. Car drivers can be given alternative routes when there are traffic jams. It will also be possible ‘ to make tiny translating machines. This will enable people who do not share a common language to talk to each other without any difficulty or to read foreign publications.

(a) What is the greatest advancement in modern technology? (b) What complicated works are computers capable of doing? (c) Write one use of computers. (d) Explain automation? (e) Why can’t computers lead independent lives or rule the world? (f) How would computers as translating machines help people? (g) What was the prediction of Sir Leon about computers in future? (h) How can computers help people going on holiday? Answer: (a) The greatest advancement in modern technology is the invention of computers. (b) Computers are capable of doing complicated works like solve mathematical problems and put thousands of unrelated facts in order. (c) Computers can provide information on the best way to prevent traffic jams. They can be used in industries, universities and to carry out complicated work in all branches of learning. (d) Automation is the process by which machines can be used to work for us. (e) Computers need detailed instructions from human beings to operate/ cannot make decisions of their own. Therefore computers cannot lead independent lives or rule the world. (f) Computers as translating machines help people in many ways. They enable people who do not share a common language to talk to each other without any difficulty. They will also help to read foreign publications. (g) Sir Leon predicted that in the future computers would be developed which would be small enough to carry in the pocket. Ordinary people would be able to use these pocket computers to obtain valuable information. Computers could be plugged in to a national network and be used like radios. (h) Computers can help people going on holiday in numerous ways. They can be informed about weather conditions. They can also help car drivers by giving alternative roads when there are traffic jams. They can also make tiny translating machines.

Sample Unseen Passage for Class 10 CBSE – 20 Read the following passage carefully:

I was born in the small but beautiful mountain village of Nakuri near Uttarkashi in Garhwal, with the gurgling, playful Bhagirathi river flowing nearby. My parents were a hard-working and extremely self-contained couple. Even though our family was poor, barely managing the essentials, my father taught us how to live and maintain dignity and self-respect — the most treasured family value till today. At the same time my parents also practised the creed, “Kindness is the essence of all religion.” They were large-hearted, inviting village folk passing by to have tea at our home, and gave grain to the sadhus and pandits who came to the house. This characteristic has been ingrained in me so deeply that I am able to reach out to others and make a difference in their lives — whether it is in my home, in society or at the workplace.

I was the third child in the family — girl, boy, girl, girl and boy in that order — and quite a rebel. I developed a tendency to ask questions and was not satisfied with the customary way of life for a girl-child. When I found my elder brother, Bachchan, encouraging our youngest brother, Raju, to take up mountaineering I thought, why not me ? I found that my brothers were always getting preferential treatment and all opportunities and options were open to them. This made me even more determined to not only do what the boys were doing, but to do it better.

The general thinking of mountain people was that mountaineering as a sport was not for them. They considered themselves to be born mountaineers as they had to go up and down mountain slopes for their daily livelihood and even for routine work. On the other hand, as a student, I would look curiously at foreign backpackers passing by my village and wonder where they were going. I would even invite them to my house and talk to them to learn more about their travels. The full significance of this came to me later when I started working. The foreigners took the trouble to come all the way to the Himalayas in order to educate themselves on social, cultural and scientific aspects of mountaineering, as well as to seek peace in nature’s gigantic scheme of things.

(a) What does the author tell us about the financial condition of her parents? (b) What is the most treasured value of the author’s family? (c) Give an example to show that the author’s parents were very hospitable. (d) What kind of girl was the author? (e) How do you know that the author’s parents discriminated between sons and daughters? (f) Why do the mountain people consider themselves to be born mountaineers? (g) Why would the author invite foreign mountaineers to her house? (h) Why were foreigners drawn to the Himalayas? Answer: (a) The author tells us that the financial condition of the author’s parents was not very good and they struggled to secure even the essentials. (b) The most treasured family value, taught by the author’s father was, how to maintain one’s self-respect and dignity in life. (c) The author’s parents were large-hearted. They were in the habit of inviting village folk home to have tea and giving alms in the form of grains to sadhus and pundits who stopped by their house. (d) The author, third among five children, was a rebel. She had the tendency of asking questions and was not satisfied with the customary way of life that was specified for a girl-child. (e) The author was discriminated against by her parents. Her brothers were being given preferential treatment and all opportunities and options were given only to the boys. (f) The mountain dwellers considered themselves to be born mountaineers because they had to go up and down the mountains for their livelihood and routine work. (g) The author invited foreign travellers to her house to talk to them and learn more about their travels. (h) The foreigners came to the Himalayas to educate themselves on the social, cultural and scientific aspects of mountaineering, as well as to seek peace in nature’s gigantic scheme of things.

Question and Answer forum for K12 Students

Unseen Passage For Class 10 Factual

Unseen Passage For Class 10 Factual CBSE With Answers

Factual passages: convey information in a straightforward and direct manner about a particular subject. Usually, the language and style are simple and clear. Factual passages may give instructions or descriptions or report of an event or a new finding.

Basic  English Grammar  rules can be tricky. In this article, we’ll get you started with the basics of sentence structure, punctuation, parts of speech, and more.

We also providing Extra Questions for Class 10 English Chapter wise.

Type 1 Questions

1. Read the passage given below and answer the questions that follow.

1 Tourists to Jammu and Kashmir have another attraction–a floating post office on the Dal Lake in Srinagar, the first in the country. ‘Floating Post Office, Dal Lake’–claimed to be the only one such post office in the world–is built on an intricately carved maroon houseboat, fastened on the western edge of the Dal Lake.

2 This post office lets you avail of all the regular postal services available in the country while being afloat. The seal used on everything posted from Floating Post Office is unique–along with the date and address, it bears the design of a boatman rowing a shikara on the Dal Lake. The special. feature of this post office is that letters posted from here carry a special design which has the picturesque scenery of Dal Lake and Srinagar city. These pictures reach wherever these letters are posted to and hence promote Kashmir as a tourist destination across the world.

3 This is actually a heritage post office that has existed since British times. It was called Nehru Park Post Office before 2011. But then the chief postmaster John Samuel renamed it as ‘Floating Post Office’.

4 The post office’s houseboat has two small rooms–one serves as the office and the other a small ‘museum that traces the philatelic history of the state postal department. It has a shop that sells postage stamps and other products.

5 But for the locals, Floating Post office is more than an object of fascination. 1–2 crore is deposited per month in Floating Post Office by communities living in and around the Dal Lake. The lake has several islets that are home to more than 50,000 people.

6. The greatest fear is the recurrence of 2014 like floods in which the houseboat had gone for a toss uncontrollably pushed by the flood. Rescue teams had to anchor it using special mechanism in a nearby highland. Then it was brought back on the Dal after the water receded. The biggest boon is that at no time of the year do you need a fan in this post–office!

Unseen Passage With Answers

Attempt the following questions on the basis of the passage you have read:

(a) Jammu and Kashmir has another attraction, the first in the country ………………………. . (i) A floating ATM (ii) A floating bank (iii) A floating post office (iv) A floating museum Answer: (iii)

(b) The greatest fear that the post office has is that ………………………. . (i) it can get drowned (ii) it can be looted (iii) it has less deposits (iv) the floods can ruin it Answer: (iv)

(c) The post office is of no use to the people of Kashmir. (True/False) Answer: False

(d) Chief postmaster John Samuel has renamed the post office. (True/False) Answer: True

(e) The post office is in an intricately carved houseboat. (True/False) Answer: True

(f) The post office’s houseboat has ……………………….. small rooms. Answer: two

(g) The heritage post office has existed since the ……………………….. times. Answer: British

(h) Find the word from the passage which means the same as “attraction”. (para 5) Answer: fascination

Unseen Passage Writing

2. Read the passage given below and answer the questions that follow.

1 Then all the windows of the grey wooden house (Miss Hilton used to live here. She expired last week.), were thrown open, a thing I had never seen before. At the end of the day, a sign was nailed on the mango tree: FOR SALE.

2. Nobody in the street knew Miss Hilton. While she lived, her front gate was always locked and no one ever saw her leave or saw anybody go in. So even if you wanted to, you couldn’t feel sorry and say that you missed Miss Hilton.

3 When I think of her house I see just two colours. Grey and green. The green of the mango tree, the grey of the house and the grey of the high iron fence that prevented you from getting at the mangoes.

4 If your cricket ball fell in Miss Hilton’s courtyard you never got it back. It wasn’t the mango season when Miss Hilton died. But we got back about ten or twelve of our cricket balls.

5 The house was sold and we were prepared to dislike the new owners even before they came. I think we were a little worried.

6 Already we had one resident of the street who kept on complaining about us to our parents. He complained that we played cricket on the pavement; and if we were not playing cricket he complained that we were making too much noise anyway.

7 One afternoon, when I came back from school Pal, said, “Is a man and a woman. She pretty, but he ugly like hell”. I didn’t see much. The front gate was open, but the windows were shut again. I heard a dog barking in an angry way.

8 One thing was settled pretty quickly. Whoever these people were they would never be the sort of people to complain that we were making noise and disturbing their sleep.

9. A lot of noise came from the house that night. The radio was going at full volume until midnight when the radio station closed down. The dog was barking and the man was shouting. I didn’t hear the woman.

(a) Miss Hilton’s house had just 2 colours ……………………….. (i) Grey and Black (ii) Grey and Blue (iii) Grey and Pink (iv) Grey and Green Answer: (iv)

(b) The sign ‘For Sale’ was hung on a ……………………….. (i) banyan tree (ii) mango tree (iii) oak tree (iv) guava tree Answer: (ii)

(c) Miss Hilton was a very friendly lady. (True/False) Answer: False

(d) You would always get your cricket balls if they fell in Miss Hilton’s courtyard. (True/False) Answer: False

(e) The new owners of the house had a dog. (True/False) Answer: True

The iron fence did not let the boys get at the ………………………. on the tree. Answer: Mangoes

(g) Find a word in paragraph 8 which means the same as “arranged. Answer: settled

(h) Find a word in paragraph 6 which means ‘raised path for pedestrians at the side of a road’. Answer: pavement

3. Read the passage given below and answer the questions that follow.

1. Varieties of birds migrate or travel from one region to another depending on the availability of food, habitat and climatic conditions.

2. In order to escape the severe winter of northern and central Asia, several migratory birds visit the Indian subcontinent during winter. Here they find optimal conditions of food and climate for their survival. Studies reveal that the quality of habitat and human disturbances along their migratory routes are the deciding factors of the number of migratory birds that come to a place.

3. Loss of habitat due to human encroachment, like settlement, grazing and agriculture, is regarded as the main reason for this decline. Scientists have observed that the breeding habitats of these birds are becoming fragmented and the tropical grounds surrounding the habitats are being destroyed.

4. According to ornithologist and professor of Zoology at the Jawaharlal Nehru University, Surya Prakash, the numbers per species of migratory birds have drastically gone down because of the loss of habitat, specially wetlands and aquatic ecosystems, due to commercial use of land and other anthropogenic factors.

5. The changing crop pattern is also seen as a major factor for the reduction in the number of migratory birds in India and certain birds have stopped visiting some areas due to such changes.

6. Delay in rainfall also leads to a delay in the arrival of migratory birds. Rains obstruct their flight and cause low visibility. Another cause of decline in migration to India is the large scale hunting of these birds. Birds like falcons are targets for many illegal hunters. In addition to this, the availability of food is also becoming scarce and various water bodies have dried up, thus robbing these birds of their habitat.

7. Unhygienic water, along with thin tree cover leads to lesser number of birds visiting man-made sanctuaries. High voltage power lines are another threat to the lives of these birds as they can get caught in between these lines and be electrocuted.

8. The World Migratory Bird Day is now celebrated to spread awareness about threats faced by migratory birds.

3.1 On the basis of your reading of the passage, complete the following table.

Reasons for Lessening Numbers of Migratory Birds

3.2 On the basis of your reading of the passage, answer the following questions. (a) Why do birds migrate from one region to another? (b) What are some factors that can lead to delays in the arrival of migratory birds? (c) Name the day that is celebrated to spread awareness about threats faced by migratory birds.

4. Read the passage given below and answer the questions that follow.

1. When was the last time you laughed a hearty, sidesplitting belly laugh that sent you reeling out of control?

2. Modem science confirms that this kind of laughter is not only enjoyable but also health-promoting. Laughter is an invigorating tonic that heightens and brightens the mood, gently releasing us from tensions and social constraints.

3. If you hate to workout, laughter (also called inner jogging) may be the exercise programme for you! A robust laugh gives the muscles of your face, shoulders, diaphragm and abdomen a good workout. The heart rate and blood pressure temporarily rise, breathing becomes faster and deeper and oxygen surges through your bloodstream. Sometimes your muscles go limp and your blood pressure may fall temporarily, leaving you in a mellow euphoria. A good laugh can bum up as many calories per hour as brisk walking.

4. Sadly, our culture seems to inhibit humour. Sometimes we repress our good humour, because we are afraid that others will think we are frivolous or foolish. But we need to cultivate a better sense of humour and to not care about others’ opinions about us; to let our sense of humour live and to seek out things that make us laugh.

6. Focus on yourself rather than others. If you can allow yourself the inevitable mistakes then you can laugh at yourself. A stressful situation can sometimes be transformed into a bit of fun if you can see the humour in it. Make sure that people around you are fun to be with. Certain people make you feel relaxed and happy. Spend more time with people who boost your mood. It can also trigger different thoughts that affect moods of sadness, happiness and anger. And if you can’t smile, fake it.

7. Not all humour is positive and healthy. Watch out for scorn, sarcasm, ridicule and contempt and inappropriate humour. And don’t joke about people’s names. They have to live with them. It is important to be sensitive.

8. Humour can be a powerful medicine and laughter can be contagious. The only side effect is pleasure.

4.1 Complete the answers for the questions given below. Each blank carried 1 mark. Answer questions worth 8 marks.

(a) What are the benefits of laughter? (i) heightens and brightens mood (ii) releases us from tensions and social constraints (iii) workout for ……………………………………………… . (iv) increases level of ……………………………………………… . (v) bums ……………………………………………… .

(b) State a few reasons why adults don’t laugh. (i) ……………………………………………… . (ii) seriousness ……………………………………………… . (c) How can one remain happy? (i) ……………………………………………… . (ii) spend time with ……………………………………………… .

(d) List the few types of negative humour. (i) scorn (ii) ……………………………………………… . (iii) ……………………………………………… . (iv) ……………………………………………… . (v) inappropriate humour (e) In terms of exercise, laughter is also known as ………………………………………………. (f) A good laughter also provides a good workout to the following parts of the body:

5. Read the passage given below and answer the questions that follow.

1. Nowadays everyone is trying to manage thoughts since the popular novel The Alchemist by Paulo Coelho encouraged the reader to chase his or her dreams.

2. Another bestseller The Secret added to the hype by propagating the theory that one can get anything just by thinking about it. On the other hand, serious seekers on the spiritual path are also after thought management and want to tame their thoughts and attain the state of thoughtlessness and to do this they go to any extent, like learning spiritual practices like vipassana and meditation.

3. It is easier to understand the attitude of people who want to get material comfort by changing their thoughts and are not concerned about the afterlife and will do anything to gain a better and comfortable life in the present. However, it is more difficult to understand the attitude of spiritual seekers because the coming and going of thoughts is a natural phenomenon and it is puzzling to understand why anyone would want to interfere with it and think of the mind as an obstacle.

4. The taming of thoughts and control of the mind can take upto many lifetimes and so Yoga Sutras of Patanjali, clearly say that a person in his or her next life starts from the point where he or she had left in the previous one.

5. In the same way, thoughts derive their power from us. They thrive on the energy we provide them when we get attached to them and also when we want to remove them. Hence, it is not surprising that spiritual seekers complain that they are not able to meditate or concentrate, a fact which fills them with guilt.

6. The alternative is to ignore the thoughts and if we don’t give them any attention they will go away.

5.1 On the basis of your reading of the passage, answer the following questions.

(a) Name the books that led to an increase in the concept of people trying to manage their thoughts. (b) Thought management is sought after by which two kinds of people ……………………………………………… and ………………………………………………. (c) What is the writer puzzled at? ……………………………………………… . (d) What do Yoga Sutras of Patanjali say? ……………………………………………… . (e) The two spiritual practices mentioned by the author are ……………………………………………… and ……………………………………………… . (f) Whose attitude does the author find it difficult to understand? (g) According to the author, the only way to make certain thoughts go away is to ……………………………………………….

Type 2 Questions

1. Salsa is a popular dance form considered to be a combination of Latin and Afro-Caribbean dances. It is a mixture of various roots and cultures and cannot be associated to a specific region. But a large part of the dance originated in the Cuban island.

2. The term ‘salsa’ actually means sauce in the Spanish language and is seen as a sensual dance that combines both sensuality and grace. It is a dance form where both the European and African cultures come together.

3. Various notions circulate regarding the origins of this particular dance form. According to one of them, when the French came to Cuba, they brought their own country dance along with them. Their native dance mixed with the native African dances like Guaguanco and gave rise to the salsa. As per another notion, the dance form of the native African slaves was known as rumba. The rumba got mixed with various other western dancing styles when these slaves were brought to America. The salsa thus originated from a mixture of various dance styles and cultures.

4. The term salsa was created in New York and there are different styles of salsa namely Cuban style, New York style, Los Angeles style, Colombian style and the Puerto Rican style, depending on the geographical locations from which they originated. Each style incorporates slightly different dance steps. Salsa branches also include the Colombian Salsa and the Miami-Style Salsa.

5. It is normally a couple dance where the male dancer and the female dancer complement each other perfectly. There are solo forms as well. The dance form usually consists of six different steps for music with eight counts.

6. Nowadays salsa is seen as a mode of exercise for people who want to get a toned body. It is a good way to maintain one’s body and also helps in relieving stress.

1.1 Based on your reading of the passage, answer the following questions.

(a) Salsa represents the cultures of which nations? (b) What did the name rumba originate from? (c) How many types of salsa are there? Name a few types. (d) Write a few lines on how Salsa originated.

1.2 Choose the correct answer for the following questions.

(a) What is the word used for ‘amalgamation’ as given in the passage? (i) popular (ii) combination (iii) form (iv) dance

(b) Which of the following words used in the passage mean the same as ‘idea’? (i) notion (ii) associated (iii) root (iv) form

(c) What does the word ‘native’ used in the passage mean? (i) associated with a particular person to whom it belongs (ii) associated with a particular plant (iii) associated with a particular place to where it belongs (iv) none of the above

(d) What does the word ‘solo’ used in the passage mean? (i) a thing done by three people (ii) a thing particular to a culture (iii) a task done in a hurry (iv) a thing done by one person

1. Popularly known as the Amazon of India, Sundarban is a tidal mangrove forest that gets its name from the Sundari tree that is found there. Mangroves are the different varieties of trees that are found in marshy lands. The Sundarban forest consists of 10,200 sq km area, spread over India and Bangladesh. Sundarban is located at the confluence of the Ganga, Brahmaputra and Meghna rivers and is part of the largest delta in the world. Its name means ‘beautiful forest’ in Bengali.

2. Sundarban has an extremely rich diversity of aquatic and terrestrial flora and fauna. Being the abode of the famous Royal Bengal tigers, Sundarban forms the largest tiger reserve and national park in India.

3. The area is known for its wide range of fauna including the chital deer, rhesus monkey, Javan rhinoceros, water buffalo, swamp deer and gaur. The aquatic fauna includes different varieties of fish, crabs, the well-known Gangetic dolphins, and the olive ridley turtles. The reptiles found there include the estuarine crocodiles, Indian python, king cobra, sea snakes, and terrapins.

4. Large parts of Sundarban have been protected. The tiger reserve was set up as early as 1973. Sundarban has been declared a World Heritage Site by UNESCO. But of late it is seen that the increasing human population of the area has led to an exploitation of timber and fish like shrimps and other biological resources. In some areas, mangrove forests are being cut leading to habitat loss. The regulation of the water flow has led to the reduction of the inflow of freshwater. The increasing level of salinity in the water has adversely affected the migration and breeding patterns of the fish. It is feared that large parts of the area will be underwater because of the rising water level of the Bay of Bengal due to global warming.

2.1 Based on your reading of the passage, answer the following questions.

(a) Name the three rivers that empty into the Bay of Bengal near Sunderban. (b) What is Sundarban is named after?

3.2 Choose the correct answer for the following questions.

(a) Which of the following is a synonym of the word ‘entertainment’ as used in the passage? (i) working i (ii) bore (iii) amusement (iv) none of the above

(b) What does the word ‘jockey’ in the given passage mean? (i) a person who rides in horse races (ii) a person who takes care of all animals (iii) a person who organizes horse races % (iv) none of the above

(c) Which of the following words is an antonym of the word ‘immense’ as used in the passage? (i) major (ii) approved (iii) honest (iv) minute

(d) What does the word ‘subsidies’ mean as used in the passage? (i) a loan of money (ii) a pack of resources (iii) financial aid or discount (iv) expensive goods

1. Anger is usually defined as a strong feeling of hostility, displeasure, indignation, rage or annoyance. It is an emotional state that is accompanied by physiological and biological changes including an increase in heart rate and blood pressure, as well as an increase in the levels of one’s energy hormones, adrenaline, and noradrenaline. An angry mind often loses its capacity to think properly.

2. Anger can be caused by both external and internal events. You could be angry at a specific person (such as a co-worker or classmate) or event (a traffic jam, a cancelled flight), or your anger could be caused by worrying or brooding about your personal problems. Memories of traumatic or enraging events can also trigger angry feelings.

3. Anger in a limited quantity is considered to be good as it gives one an opportunity to unleash negative feelings. Anger is a natural, adaptive response to threats. A certain amount of anger is needed to fight, defend and justify ourselves in society. Hence anger in small quantities is needed for our survival, like it is justified to get angry when one feels that one’s rights have been denied. But excessive anger can lead to destructive results including violent actions.

4. Anger can range anywhere from mild irritation to wild, uncontrollable fury. Anger leads people to behave in an impulsive and rash manner which in turn car^lead to strained relationships in the public as well as private sphere.

5. One of the most commonly used anger management methods includes breathing exercises. A number of anger management courses are available nowadays to help one control one’s anger. It always helps to have someone from whom one can seek help and talk. It is important to learn how to express one’s anger assertively rather than aggressively. One can also substitute the thought that makes one angry with more pleasant thoughts. Nowadays mental health professionals and psychologists are also available to help people control their anger.

4.1 Based on your reading of the passage, answer the following questions.

(a) What are the effects of anger? (b) What are the causes of angry feelings? (c) Why is anger in a limited quantity good? (d) Write a few lines on the range of types of anger.

4.2 Select appropriate synonyms for the following words from the passage out of the options provided.

(a) hostility (i) antagonism (ii) friendliness (iii) proximity (iv) behaviour

(b) personal (i) public (ii) familial (iii) private (iv) closeness

(c) aggressive (i) mild (ii) forceful (iii) passive (iv) calm

(d) pleasant (ii) cold (iii) uninviting (iv) None of the above

1. Calcium is a mineral that is found in abundance in the human body. Besides the fact that it is one of the most abundant minerals in the body, it is also one of the most important minerals.

2. Calcium is required for various functions in the body, but the maximum amount of calcium in the body is found in the bones and teeth. Calcium is required to maintain the strength of the bones. Some of the best sources of calcium are milk, yogurt, cheese, fish, and green leafy vegetables like cabbage and spinach.

3. The most common disease connected to calcium deficiency is rickets. Every time a child drinks a soft drink, he or she lays the groundwork for dangerous bone disease. Fizzy and sugary drinks do not cause osteoporosis. But, because they are often a substitute for a glass of milk, kids do not get the calcium and vitamin D they need to build a strong skeleton. Many of them also lead a sedentary lifestyle, so they do not get the bone-building benefits of vigorous exercise either. These children are not just in jeopardy of getting brittle bones and fractures in the future, but they could be at a risk of osteoporosis at a younger age than ever before.

4. The Indian Society for Bone and Mineral Research, a body of osteoporosis experts, is trying to spread awareness about this bone-crippling disease. Osteoporosis causes bones to become riddled with holes, like the framework of a house that has been attacked by termites. It causes loss of height, pain in the joints and back and fractures.

5. Doctors are of the view that a simple lifestyle and proper nutrition will help save our bones. They recommend adopting bone-friendly habits like—regular exercise, getting enough sunlight, and eating food which will give us adequate calcium. This way, we can keep our bones healthy and prevent osteoporosis.

5.1 Based on your reading of the passage, answer the following questions.

(a) What are the symptoms of people with less calcium in their bodies? – (b) What is osteoporosis caused by? (c) Why has osteoporosis been compared to a framework of ‘a house attacked by termites’? (d) How does osteoporosis affect one’s bones?

5.2 Select appropriate antonyms for the following words from the passage out of the options provided.

(a) abundant (i) plenty (ii) meagre (iii) enough (iv) lot

(b) subsequent (i) following (ii) after (iii) preceding (iv) none of the above

(c) adequate (i) inadequate (ii) enough (iii) plenty (iv) none of the above

(d) prevent (i) hinder (ii) stop (iii) inhibit (iv) cause

CBSE Case Based Passages for Class 9 and 10 Solved Examples | Downloadable PDF- Set- 1

CBSE Case Based Passages for Class 9 and 10 Solved edumanta.net

This page has the examples of case based unseen passage for classes 9 and 10 students and teachers. Our case-based passages are here to help you excel in your practice and exams. Downloadable pdf is also available after each unseen passage. These passages are designed with the latest educational standards after 2023 and 24 sessions. 

Table of Contents

Case based unseen passage: 1 urban farming revolution.

Urban farming, also known as urban agriculture, is gaining traction in metropolitan areas around the world. As cities grow and open spaces diminish, residents are finding innovative ways to cultivate food and greenery within their urban environments. One such method is rooftop farming, where unused rooftops are transformed into productive green spaces. Another emerging trend is the use of hydroponics, a method of growing plants without soil, using mineral nutrient solutions in an aqueous solvent.

The primary objective of urban farming is not just food production but also ecological balance. By integrating farming into urban planning, architects and city planners aim to reduce the heat island effect, improve air quality, and promote biodiversity . These farms become a haven for city dwellers, offering a respite from the concrete jungle and a chance to reconnect with nature .

However, urban farming is not without its challenges. Water management, soil quality, and pest control are some of the issues that urban farmers face. Additionally, the initial setup costs for hydroponic systems can be high, and there’s a learning curve involved in mastering this method. Despite these challenges, the benefits of urban farming, both environmental and social, are undeniable. It fosters community engagement, promotes healthy eating, and reduces the carbon footprint associated with transporting food over long distances.

Survey on Urban Farming (Total number of participants: 400)

i. Urban farming’s primary goal extends beyond food production to also maintain ________ in urban areas.

ii. An unconventional method in urban farming allows plants to grow in the absence of ________.

iii. Which statement best encapsulates the multifaceted benefits of urban farming? 

a) It solely focuses on beautifying the cityscape. b) It plays a role in moderating urban temperatures. c) It is primarily used for recreational purposes. d) It is a source of renewable energy.

iv. What nuanced understanding can be derived about hydroponics from the passage? 

a) It is universally adopted in all urban farms. b) It completely eliminates the need for water in farming. c) It demands expertise and might entail substantial initial investments. d) It is the most economical urban farming method.

v. Referring to the survey, which statement witnessed the least disagreement from the participants? 

a) Statement 1  b) Statement 2  c) Statement 4  d) Statement 5

vi. Why is the integration of farming into urban architectural designs deemed necessary?

vii. Elucidate on the potential merits and demerits of hydroponics in the context of urban farming. (Answer in about 40 words)

viii. How does urban farming potentially transform the urban social landscape? (Answer in about 40 words)

i. Answer: Ecological equilibrium Reason: The passage emphasizes that urban farming’s primary objective is not solely food production but also maintaining ecological balance.

ii. Answer: Soil Reason: Hydroponics, as described in the passage, is a method where plants are cultivated without the use of soil.

iii. Answer: b) It plays a role in moderating urban temperatures. Reason: The passage elucidates that urban farming, through its integration into urban planning, aims to mitigate the heat island effect.

iv. Answer: c) It demands expertise and might entail substantial initial investments. Reason: The passage outlines the complexities of hydroponics, highlighting the steep learning curve and potential high setup costs.

v. Answer: d) Statement 5 Reason: Based on the table, Statement 5 has the least number of “Strongly Disagree” responses, tallying to 10.

vi. Answer: The integration is pivotal to achieve ecological balance, encompassing the reduction of the heat island effect, enhancement of air quality, and fostering of biodiversity. Reason: The passage underscores the ecological imperatives of urban farming.

vii. Answer: Hydroponics conserves water, obviates the need for soil, and is adaptable to limited spaces, often leading to accelerated plant growth. Conversely, it demands expertise and might be capital-intensive initially. Reason: The passage delineates hydroponics, emphasizing its advantages and challenges.

viii. Answer: Urban farming bolsters community interaction, offers an oasis amidst urban sprawl, encourages wholesome dietary habits, and facilitates a reconnection with nature for urbanites. Reason: The passage expounds on the multifarious societal advantages of urban farming.

Case Based Unseen Passage: 2 The Rise of Solar Cities

As the world grapples with the challenges of climate change, cities globally are turning to renewable energy sources to reduce their carbon footprint. Solar energy, harnessed from the sun’s rays, is emerging as a leading solution. Across urban landscapes, solar panels are becoming a common sight, not just on rooftops but also integrated into building designs and public infrastructure.

The primary motivation behind this shift to solar energy is twofold: environmental responsibility and economic viability. Solar energy reduces the reliance on fossil fuels, thereby decreasing greenhouse gas emissions. This transition not only combats global warming but also ensures cleaner air, benefiting urban residents’ health.

However, the journey to becoming a solar city is not without hurdles. Initial installation costs, ensuring consistent energy supply during cloudy days, and the need for efficient energy storage solutions are some challenges cities face. But with technological advancements, such as the development of high-capacity batteries and more efficient solar cells, these challenges are gradually being overcome.

Moreover, solar cities are not just about energy. They represent a broader vision of sustainability, encompassing green transportation, waste management, and water conservation. The holistic approach ensures a better quality of life, fostering community well-being and resilience against climate adversities.

Survey on Solar Cities (Total number of participants: 400)

i. Urban farming not only focuses on food production but also aims to achieve a balance in the ________.

ii. One innovative method of urban farming involves growing plants without the use of ________.

iii. Which of the following is a primary reason for cities adopting solar energy? a) Aesthetic appeal b) Economic viability c) Popularity among residents d) Availability of space

iv. What can be inferred about the challenges of becoming a solar city? a) They are insurmountable. b) They are purely economic. c) They include energy storage and installation costs. d) They are limited to urban areas.

v. According to the survey, which statement had the most “Strongly Agree” responses? a) Statement 1 b) Statement 2 c) Statement 3 d) Statement 5

vi. Why is solar energy considered crucial for urban areas?

vii. Discuss the broader vision of sustainability represented by solar cities. (Answer in about 40 words)

viii. Elaborate on the challenges and solutions associated with the transition to solar cities. (Answer in about 40 words)

i. Answer: Fossil fuels Reason: The passage mentions that solar energy reduces the reliance on fossil fuels.

ii. Answer: Green transportation Reason: The passage states that solar cities encompass green transportation, waste management, and water conservation.

iii. Answer: b) Economic viability Reason: The passage highlights that the motivation behind the shift to solar energy is environmental responsibility and economic viability.

iv. Answer: c) They include energy storage and installation costs. Reason: The passage lists initial installation costs and the need for efficient energy storage as challenges.

v. Answer: a) Statement 1 Reason: Referring to the table, Statement 1 has the most “Strongly Agree” responses with a count of 220.

vi. Answer: Solar energy is considered crucial for urban areas because it reduces reliance on fossil fuels, decreases greenhouse gas emissions, combats global warming, and ensures cleaner air. Reason: These benefits are discussed in the passage, emphasizing the environmental and health advantages of solar energy.

vii. Answer: Solar cities represent a holistic approach to sustainability, encompassing not just solar energy but also green transportation, waste management, and water conservation. Reason: The passage discusses the broader vision of solar cities beyond just energy.

viii. Answer: Challenges of transitioning to solar cities include high initial installation costs, consistent energy supply during cloudy days, and efficient energy storage. Solutions are emerging with technological advancements like high-capacity batteries and efficient solar cells. Reason: The passage outlines both the challenges and the evolving solutions associated with solar cities.

Case Based Unseen Passage: 3 The Digital Detox Movement

In an era dominated by screens and constant connectivity, a counter-movement is gaining momentum: the digital detox. This movement encourages individuals to take intentional breaks from their digital devices, aiming to reduce stress, improve mental well-being, and foster real-world connections.

The primary drive behind the digital detox movement is the growing realization of the adverse effects of excessive screen time. Studies have shown that prolonged exposure to screens can disrupt sleep patterns, reduce attention spans, and even contribute to feelings of loneliness and depression. By consciously unplugging, individuals can recalibrate their relationship with technology and rediscover the joys of face-to-face interactions.

However, embracing a digital detox is not without its challenges. In a world where work, social interactions, and even leisure often revolve around digital platforms, disconnecting can feel daunting. There’s also the fear of missing out (FOMO) on important updates or news. Yet, with structured breaks and setting clear boundaries, many find the transition to be rejuvenating.

Beyond individual benefits, the digital detox movement has broader societal implications. Schools are introducing tech-free days, companies are promoting device-free meetings, and retreats are offering unplugged experiences. These initiatives aim to create a balanced digital ecosystem, where technology enhances rather than dominates our lives.

Survey on Digital Detox (Total number of participants: 400)

i. The digital detox movement encourages individuals to take breaks from their ________.

ii. One of the adverse effects of excessive screen time is the disruption of ________ patterns.

iii. Which of the following is a primary reason for the rise of the digital detox movement? a) Aesthetic appeal b) Economic viability c) Popularity among residents d) Promoting new gadget sales

iv. What can be inferred about the challenges of a digital detox? a) They are universally easy. b) They are primarily financial. c) They involve the fear of missing out and societal norms. d) They are promoted by tech companies.

v. According to the survey, which statement had the most “Strongly Agree” responses? a) Statement 1 b) Statement 2 c) Statement 3 d) Statement 4

vi. Why is the digital detox movement considered beneficial for individuals?

vii. Discuss the broader societal implications of the digital detox movement. (Answer in about 40 words)

viii. Elaborate on the challenges individuals face when trying to reduce screen time. (Answer in about 40 words)

i. Answer: Digital devices Reason: The passage mentions that the digital detox movement encourages individuals to take intentional breaks from their digital devices.

ii. Answer: Sleep Reason: The passage states that prolonged exposure to screens can disrupt sleep patterns.

iii. Answer: c) Improving mental well-being Reason: The passage highlights that the movement aims to reduce stress and improve mental well-being.

iv. Answer: c) It involves the fear of missing out and societal norms. Reason: The passage lists the fear of missing out (FOMO) and the challenges of disconnecting in a digital world as hurdles to a digital detox.

v. Answer: a) Statement 1 Reason: Referring to the table, Statement 1 has the most “Strongly Agree” responses with a count of 230.

vi. Answer: The digital detox movement is considered beneficial because it helps reduce stress, improve mental well-being, foster real-world connections, and recalibrate individuals’ relationship with technology. Reason: These benefits are discussed in the passage, emphasizing the positive outcomes of unplugging.

vii. Answer: The digital detox movement promotes a balanced digital ecosystem, with schools introducing tech-free days, companies advocating device-free meetings, and retreats offering unplugged experiences. Reason: The passage discusses the broader societal implications and initiatives related to the digital detox movement.

viii. Answer: Challenges include the fear of missing out (FOMO) on important updates or news, societal norms around constant connectivity, and the integration of digital platforms in work, social interactions, and leisure. Reason: The passage outlines the challenges individuals face when trying to disconnect from their devices.

Case Based Unseen Passage: 4 The Revival of Traditional Crafts

In a world rapidly advancing towards automation and mass production, there’s a growing appreciation for traditional crafts. Handmade products, from pottery to textiles, are being celebrated for their uniqueness, quality, and the stories they tell.

The resurgence of traditional crafts is driven by multiple factors. Firstly, consumers are becoming more conscious of the environmental impact of mass-produced goods. Handcrafted items, often made from sustainable materials, offer an eco-friendly alternative. Secondly, there’s a desire to preserve cultural heritage. Traditional crafts are a testament to a community’s history, skills, and artistic expression.

However, sustaining traditional crafts in a modern economy is challenging. Artisans often struggle with competition from cheaper, machine-made alternatives. Additionally, the younger generation, lured by urban opportunities, is less inclined to take up age-old crafts. Yet, with the right support, such as training, marketing, and access to global markets, these crafts can thrive.

Communities and organizations worldwide are stepping up to support artisans. Craft fairs, workshops, and online platforms are providing artisans with platforms to showcase their work and connect with a broader audience. Moreover, consumers are willing to pay a premium for handcrafted goods, recognizing the skill and effort involved.

Table 5 – Survey on Traditional Crafts (Total number of participants: 400)

iii. Which of the following is a reason for the resurgence of traditional crafts? a) Rapid technological advancements b) Desire to preserve cultural heritage c) Increase in disposable income d) Popularity of modern designs

iv. What can be inferred about the younger generation’s interest in traditional crafts? a) They are highly interested. b) They prefer modern crafts. c) They are less inclined to take up age-old crafts. d) They dominate the craft market.

v. According to the survey, which statement had the most “Strongly Agree” responses? a) Statement 1 b) Statement 2 c) Statement 3 d) Statement 6

vi. Why are consumers willing to pay a premium for handcrafted goods?

vii. Discuss the role of communities and organizations in supporting artisans. (Answer in about 40 words)

viii. Elaborate on the challenges and opportunities faced by artisans in the modern economy. (Answer in about 40 words)

i. Answer: Uniqueness Reason: The passage mentions that handmade products are celebrated for their uniqueness.

iii. Answer: b) Desire to preserve cultural heritage Reason: The passage highlights that one of the reasons for the resurgence of traditional crafts is the desire to preserve cultural heritage.

iv. Answer: c) They are less inclined to take up age-old crafts. Reason: The passage mentions that the younger generation is less inclined to take up age-old crafts.

v. Answer: d) Statement 6 Reason: Referring to the table, Statement 6 has the most “Strongly Agree” responses with a count of 240.

vi. Answer: Consumers recognize the skill, effort, and uniqueness involved in creating handcrafted goods, and they value the stories and cultural significance behind them. Reason: The passage discusses the appreciation and value consumers place on handcrafted items.

vii. Answer: Communities and organizations provide artisans with platforms like craft fairs, workshops, and online platforms to showcase their work and connect with a broader audience, supporting their craft and livelihood. Reason: The passage discusses the initiatives taken by communities and organizations to support artisans.

viii. Answer: Artisans face challenges like competition from machine-made goods and declining interest from the younger generation. However, opportunities arise from support in training, marketing, access to global markets, and consumers’ willingness to pay a premium for handcrafted goods. Reason: The passage outlines both the challenges and opportunities artisans face in the modern economy.

Case Based Unseen Passage: 5 Urban Farming – A Green Revolution

In the heart of bustling cities, a green revolution is taking root. Urban farming, the practice of cultivating, processing, and distributing food in urban areas, is gaining traction as a sustainable solution to the challenges of food security and environmental degradation.

The appeal of urban farming lies in its multifaceted benefits. For starters, it reduces the carbon footprint associated with transporting food from rural farms to urban markets. Fresh produce can be harvested and consumed within the same locality, ensuring maximum freshness and nutritional value. Moreover, urban farming utilizes innovative techniques like vertical farming and hydroponics, which use less land and water compared to traditional farming methods.

However, urban farming is not without its challenges. Space constraints in cities mean that urban farms are often smaller and yield less produce. Additionally, urban pollution can affect the quality of the crops. But with technological advancements and community involvement, these challenges are being addressed.

Governments and organizations are recognizing the potential of urban farming. Initiatives are being launched to provide training, resources, and funding to budding urban farmers. Schools are incorporating urban farming into their curriculum, teaching students the importance of sustainable agriculture.

Table 6 – Survey on Urban Farming (Total number of participants: 400)

i. Urban farming is gaining popularity as a solution to ________ and environmental degradation.

ii. One of the techniques used in urban farming that requires less land is ________.

iii. Which of the following is a benefit of urban farming? a) Increases carbon footprint b) Requires large tracts of land c) Ensures maximum freshness of produce d) Is unaffected by urban pollution

iv. What can be inferred about the challenges of urban farming? a) They are non-existent. b) They are primarily financial. c) They relate to space constraints and pollution. d) They are due to lack of interest.

vi. Why are governments and organizations supporting urban farming?

vii. Discuss the role of schools in promoting urban farming. (Answer in about 40 words)

viii. Elaborate on the innovative techniques used in urban farming and their benefits. (Answer in about 40 words)

i. Answer: Food security Reason: The passage mentions that urban farming addresses challenges of food security.

ii. Answer: Vertical farming Reason: The passage states that urban farming utilizes innovative techniques like vertical farming.

iii. Answer: c) Ensures maximum freshness of produce Reason: The passage highlights that fresh produce can be harvested and consumed within the same locality.

iv. Answer: c) They relate to space constraints and pollution. Reason: The passage lists space constraints and urban pollution as challenges of urban farming.

v. Answer: a) Statement 1 Reason: Referring to the table, Statement 1 has the most “Strongly Agree” responses with a count of 245.

vi. Answer: Governments and organizations recognize the potential of urban farming in addressing food security, reducing carbon footprint, and promoting sustainable agriculture. They provide training, resources, and funding to support urban farmers. Reason: The passage discusses the initiatives taken by governments and organizations to support urban farming.

vii. Answer: Schools are incorporating urban farming into their curriculum, teaching students the importance of sustainable agriculture and providing hands-on experience. Reason: The passage mentions the role of schools in promoting urban farming.

viii. Answer: Urban farming uses innovative techniques like vertical farming and hydroponics. These methods are space-efficient, use less water, and can produce crops year-round, offering a sustainable approach to agriculture in urban settings. Reason: The passage outlines the techniques and their benefits in urban farming.

Case Based Unseen Passage: 6 Rapid Urbanization and Industrialization

(1) The rapid urbanization and industrialization have led to a significant increase in pollution levels. People, surrounded by pollutants, are seeking innovative ways to combat this issue. One such method that has gained popularity is the use of green vehicles. Green vehicles, such as electric cars and hybrid vehicles, are an innovative approach to transportation that reduces the emission of harmful pollutants.

(2) The concept of green vehicles is not just about reducing pollution; it’s also about creating sustainable transportation systems. These vehicles can help reduce the dependence on fossil fuels, decrease greenhouse gas emissions, and even contribute to energy efficiency. Moreover, they can serve as a catalyst for the development of renewable energy technologies, promoting a shift towards cleaner and greener energy sources.

(3) A recent survey conducted in a metropolitan city revealed interesting insights into the public’s perception of green vehicles. The survey included residents from various age groups and backgrounds. The table below displays the responses:

(4) Despite these concerns, the study concluded that green vehicles have immense potential to transform urban transportation into a green, sustainable system. They not only offer environmental benefits but also provide economic advantages in the long run due to lower fuel and maintenance costs. The study recommended further research to explore innovative solutions for the challenges associated with green vehicles.

1. What does the passage suggest about green vehicles?

2. What is the author’s tone when discussing the potential of green vehicles?

3. Identify two sets of antonyms from the passage.

4. Complete the sentence appropriately: The author suggests that green vehicles are beneficial because ________.

5. Based on the reading of the passage, examine, in about 40 words, how using green vehicles can be like exploring a new city.

6. What is the message conveyed by the author’s discussion of the challenges of using green vehicles?

7. State whether the following lines display an example of a simple / complex / emotionally charged / downright weird, passage: “But whether it’s complex, emotional, simple, or just downright weird, there’s always something to be gained from the experience.”

8. Based on the reading of the passage, explain in about 40 words, why the author believes that using green vehicles is a rewarding pursuit.

Answers- 

1. The passage suggests that green vehicles are an innovative, eco-friendly transportation method that reduces pollution and promotes cleaner energy sources.

2. The author’s tone when discussing green vehicles is optimistic and informative.

3. Two sets of antonyms from the passage could be “increase” and “reduce”, and “benefits” and “challenges”.

4. The author suggests that green vehicles are beneficial because they reduce pollution and promote cleaner energy.

5. Using green vehicles, like exploring a new city, involves navigating unfamiliar terrain and gaining a deeper understanding of the subject.

6. The author conveys that the benefits of green vehicles outweigh the challenges associated with their use.

7. The line “But whether it’s complex, emotional, simple, or just downright weird, there’s always something to be gained from the experience.” is an example of a complex passage.

8. The author believes that using green vehicles is rewarding due to their environmental benefits and potential for sustainable urban transportation.

Case Based Unseen Passage: 7 The Allure of Vintage Timepieces

In an era dominated by digital technology and smartwatches, the charm of vintage timepieces remains undiminished. These mechanical marvels, often handcrafted with precision, represent a bygone era where time was measured with gears, springs, and meticulous craftsmanship.

Collectors and enthusiasts are drawn to vintage watches not just for their intricate designs but also for the history they encapsulate. Each watch tells a story of its era, the technological advancements of the time, and the personal tales of its previous owners.

One of the primary reasons for the enduring appeal of these timepieces is their tangibility. In a world of intangible digital assets, the tactile experience of winding a watch and feeling its gears move is unparalleled. Moreover, the sustainability aspect cannot be overlooked. Vintage watches, when maintained, can last for generations, making them eco-friendly compared to disposable modern gadgets.

However, collecting and maintaining vintage watches is not without its challenges. Authentic pieces are becoming rarer, and the expertise required to repair and restore them is dwindling. Additionally, the cost of acquiring pristine vintage pieces has skyrocketed due to their increasing demand.

Despite these hurdles, the allure of vintage watches remains strong. They serve as a bridge between the past and present, reminding us of the timeless nature of craftsmanship and the transient nature of time itself.

Survey on Vintage Timepieces (Total number of participants: 400)

i. Vintage timepieces are cherished not just for their design but also for the ________ they embody.

ii. One of the unique experiences associated with vintage watches is their ________.

iii. Which of the following is a significant challenge in the world of vintage watch collecting? a) Lack of digital features b) The ubiquity of vintage pieces c) The rising cost of acquisition d) The short lifespan of vintage watches

iv. What can be inferred about the sustainability of vintage watches from the passage? a) They are less sustainable than modern watches. b) They are designed to be disposable. c) They can last for generations if maintained. d) They require frequent replacements.

v. According to the survey, which statement had the fewest “Strongly Disagree” responses? a) Statement 1 b) Statement 2 c) Statement 3 d) Statement 4

vi. Why do collectors and enthusiasts value vintage watches?

vii. Discuss the implications of the dwindling expertise in repairing vintage watches. (Answer in about 40 words)

viii. Elaborate on how vintage timepieces serve as a connection between the past and the present. (Answer in about 40 words)

i. Answer: History Reason: Collectors and enthusiasts are drawn to vintage watches for the history they encapsulate.

ii. Answer: Tangibility Reason: The passage emphasizes the tangible experience of winding a watch and feeling its gears move.

iii. Answer: c) The rising cost of acquisition Reason: The passage mentions that the cost of acquiring pristine vintage pieces has increased due to demand.

iv. Answer: c) They can last for generations if maintained. Reason: The passage highlights that vintage watches, when maintained, can last for generations.

v. Answer: b) Statement 2 Reason: Referring to the table, Statement 2 has the fewest “Strongly Disagree” responses with a count of 5.

vi. Answer: Collectors and enthusiasts value vintage watches for their intricate designs, the history they encapsulate, and the tangible experience they offer. Reason: The passage discusses the reasons why vintage watches are cherished.

vii. Answer: The scarcity of expertise in repairing vintage watches poses a threat to their preservation. As skilled artisans become rarer, the legacy and functionality of these timepieces are at risk. Reason: The passage emphasizes the challenges of maintaining vintage watches due to the scarcity of skilled artisans.

viii. Answer: Vintage watches serve as a tangible link to the past, representing an era of meticulous craftsmanship. They remind us of the evolution of timekeeping and the stories of their previous owners. Reason: The passage discusses the allure of vintage watches as a bridge between the past and present.

Related Posts

Discursive Unseen Passage for Class 10 edumantra.net

Unseen Passage

Unseen Passage

For Class 4 to Class 12

Unseen Passage for Class 10

Is it true that you are searching for a unseen passage for class 10 in English? if indeed, continue to read this article.

we will give you an article on unseen passage for class 10 with numerous passages to rehearse.

That implies in case you’re an student of class 10 and searching for a unseen passage for class 10 in English, you’ve gone to the perfect spot.

Without burning through your significant time we should begin.

Unseen passage for class 10 is the most important part to score higher marks in your exam. .Reading the unseen passage for class 10 in English will help you to write better answers in your exam and improve your reading skill.

Students who are planning to score higher marks in class 10 should practice the unseen passage for class 10 before attending the CBSE board exam. Unseen Passage for Class 10 carries 20 marks in your English exam.

I t is compulsory to solve the unseen passage for class 10 because you need to score higher marks in your exam.

To improve your skills, we have provided you with the English unseen passage for class 10 . We have 15 unseen passage for class 10 given below.

While Solving the passage, you will see some factual passage for class 10 is also present in them. It is provided to make yourself an expert by solving them and score good marks in your exam. You can also practice unseen passage for class 10 in Hindi

Remember don’t start with writing the answer when you did not see unseen passage for class 10.

Important Tips to score good marks in the unseen passage for class 10

1-Read the complete passage carefully .Your reading should be quick.

2- Focus your mind on related detail and highlight them.

3-Read the question carefully and return to passage to write the answer.

4- logical sequence should be present while writing the answer.

5-Do not try to copy the sentence from the passage .write your answer with your own word.

6-while answering the vocabulary-based question like synonyms Replace that word with another word that has the same meaning as that word asked in the question.

7-To Choose the correct option in MCQs.Read the passage until you don’t get the correct answer.

Unseen Passage with multiple choice questions for Class 10 pdf is great for preparing for CBSE board examinations. CBSE Class 10 English Syllabus is much bigger and requires concentrated efforts on the part of the student to face the examinations and pop out a success. Unseen Passage with multiple choice questions for Class 10 Pdf consists of details from the subjects. Each explanation is provided with all the assumptions and good judgment used to determine the conclusion. This will allow the students to study and understand each concept even though they are preparing for the first time.

Unseen Passage for Class 10 with answers

Read the passage carefully and answer the given questions that follow :

India is home to 18% of the global population but has only 4% of the global water resources. Its per capita water availability is around 1,100 cubic meters (m 3 ), well below the internationally recognised threshold of water stress of 1,700 m 3 per person, and dangerously close to the threshold for water scarcity of 1,000 m 3 per person. Paradoxically, India is also the largest net exporter of virtual water (the amount of water required to produce the products that India exports) and has one of the most water-intensive economies. Despite looming water scarcity, India is one of the largest water users per unit of GDP suggesting that the way in which India manages its water resources accounts for much of its water woes. The problem lies with the Government capacities which are lacking in improving water management, while policies and incentives often favour inefficient and unproductive use of water. This coupled with weak or absent institutions (e.g., for water regulation) and poor data collection and assessment results in the increasing state of water woes in the country. When it comes to improving water service delivery, India can learn from Brazil, Colombia, Mozambique and New South Wales (Australia), among others. We don‘t have to go overseas to see good examples of water resources management. The Maharashtra Water Resources Regulatory Authority, established under a World Bank project, is putting in place policies, regulations, institutions and incentives that promote more efficient and more productive use of water, e.g., by ensuring the equitable distribution of water among users, and by establishing water tariffs. Efforts to establish effective authorities are also underway in other states, and Maharashtra is disseminating the lessons learned from its experience. Further, The World Bank‘s Country Partnership Framework for India recognizes the importance of the efficient use of natural resources, including water, in support of the country‘s ambitious growth targets. Several World Bank projects support India‘s efforts in the water sector: Through the National Mission for Clean Ganga, the World Bank is helping the Government of India build institutional capacity for the management and clean-up of the Ganga and investing to reduce pollution. Another World Bank project, the Dam Rehabilitation and Improvement Project, has improved the safety and performance of 223 dams in the country. The National Hydrology Project is providing significant support to strengthen capacities, improve data monitoring and analysis, and laying the foundations for benchmarking and performance-based water management. There are many more such projects that aid an efficient water management system.

On the basis of your understanding of the passage answer the following questions with the help of the given options :

Question. What does the passage tell us about the Indian per capita water availability? (a) It holds the world‘s largest water supply. (b) It is well below the internationally recognised threshold of water stress. (c) It is dangerously close to the threshold for water scarcity. (d) both b and c

Question. In the line ―…. Internationally recognised household‖, the threshold DOES NOT include (a) the magnitude or intensity (b) a point of entry (c) margin (d) a level, rate, or amount

Question. According to the passage, Indian water woes are a result of (a) lack of potable water (b) lack of water management system (c) over pollution in the water bodies (d) excess usage and wastage of water

Question. Based on your understanding of the passage, choose the option that lists the ways in which water can be managed in India. 1. Regulation 2. Recycle 3. Incentives 4. Reuse 5. Policies 6. Treatment (a) 1,4 and 6 (b) 2,3 and 5 (c) 1,3 and 5 (d) 2,4 and 6

Question. What is the National mission for clean Ganga? (a) A mission by the World bank for the management of the river Ganga. (b) A mission by the Government of India in building institutional capacity for the management of the river Ganga. (c) A mission to build institutional capacity for the management and clean-up of the Ganga and investing to reduce pollution. (d) A mission for cleaning the river Ganga while reducing the pollution causing elements.

Based on above given passage, answer the following questions briefly: 

Question. Why are the water woes in the country ever on rise? Answer.  Lacking in the improvement of water management, Policies and incentives favouring inefficient and unproductive use of water,Poor data collection and assessment.

Question. How does the Maharashtra water resources regulatory authority serve as an example for the nation? Answer.  By putting in place policies, regulations, institutions and incentives that promote more efficient use of water, By limiting the use of water to certain hours of the day, By putting strict enforcement of laws against those who pollute or waste potable water.

Question. “Through the National Mission for Clean Ganga, the World Bank is helping the Government of India build institutional capacity for the management and clean-up of the Ganga and investing to reduce pollution.” State whether the statement is True or False. Answer.  True

Question. What are the different ways adopted by the government to save water? Answer.  the Dam Rehabilitation and Improvement Project,Hydrology Project is providing significant support to strengthen capacities, improve data monitoring and analysis, and laying the foundations for benchmarking and performance-based water management

Question. What type of problems does the Government is facing in order to improve water management? Answer.  The problem lies with the Government capacities which are lacking in improving water management, while policies and incentives often favour inefficient and unproductive use of water. This coupled with weak or absent institutions (e.g., for water regulation) and poor data collection and assessment results in the increasing state of water woes in the country

English Unseen Passage for Class 10 pdf with answers

Read the following passage carefully:

1.A decade ago, no one would have believed or imagined that anyone would order medicines from the internet. But with a change in the consumer’s attitude, the growth of e-pharmacies has changed the face of pharmaceutical and medical sectors.

2.Online pharmacies offer good discounts as compared to retail medical stores. They offer doorstep delivery within a short time, though they cannot deliver medicines lightning fast, in case of an emergency, they still seem to have caught the attention of the people. Online pharmacies have less functional costs, unlike conventional ones that must consider rental and operational coasts.

3.Traditional medical stores can stock just few hundred medicines and do not provide alternative drugs for a particular medicine. Online pharmacies have an upper hand in offering varieties due to the presence of warehouses.

4. Despite a bundle of advantages, many experts have pointed out that one-fifth of the medicines sold through online pharmacies are fake. These drugs contain either different active ingredients or none at all. As a result, the drug which is consumed greatly increases the risk of adverse drug reactions or even an overdue.

5. The union health ministry of India has come out with trap rules on sale of drugs by e-pharmacy portals. Any person who intense to conduct business of e-pharmacy shall apply for the grant of registration to the central licensing authority in form 18AA through the online portal of the central government. The draft notification says that the application of registration of e-pharmacy will have to be accompanied by a sum of rupees 50,000 while asserting that an e-pharmacy registration holder will have to comply with provisions of Information Technology act 2000 (21 of 2000). The details of patients shall be kept confidential and shall not be disclosed to any person other than the central government or the state government concerned, as the case may be. The supply of any drug shall be made against a cash or credit memo generated through the e-pharmacy portal and its record maintained.

6. Online pharmacies could be a boon for consumers, but lack of oversight on fake medicines, sugar pills, expired medicines, and contaminated medicines pose a real risk. Since most online pharmacies aren’t monitored, it can lead to fraudulent practices such as sending wrong dosages or generic variants when the doctor has prescribed a brand name. moreover, online pharmacies might increase self-medications, so they must create awareness of the dangers of self-medication.

On the basis of your reading of the given passage answer the following questions attempt any 10 of the following:

Question.The growth of e pharmacies has changed the face of: a. medical sectors b. warehouses c. traditional and medical stores d. businesses

Question. Online pharmacies have less:- a. operational costs b. rental costs c. functional costs d. customers

Question. Online pharmacist have an upper hand in offering varieties due to: a. Presence of warehouses b. presence of traditional medicine stores b. Availability of alternative drugs for a particular medicine c. Different active ingredients

Question. Based on your reading of the passage choose the incorrect statement from the following: a. traditional medicine stores have an upper hand in offering varieties due to the presence of warehouses. b. Online pharmacies offer good discounts as compared to retail medical stores c. Online pharmacies have less functional costs. d. Many experts have pointed out that one-fifth of the medications sold through online pharmacies are fake.

Question. Choose the option that correctly States the two meanings of monitor as used in the passage: i) To observe and check something ii) To cause a disturbance iii) To keep something under synthetic systematic iv) Neglect and ignore something v) To get out of control a. I and 4 b. 2 and 5 c. 1 and 3 d.3 and 5

Question. What shall a person who is intended to conduct the businesses of e-pharmacy do? Answer.  He shall apply for the grant of registration to the Central Licensing Authority in From 18AA through the online portal of the Central Government.

Question. What does the passage speak about? Answer.  The passage speaks about the evolution of the online medicines.

Question. State one drawback of online delivery of medicines. Answer.  They cannot be delivered at a lightning speed.

Question. What does the draft notification mention about the details of patients. Answer.  The details of patients shall be kept confidential and shall not be disclosed to any person other than the central government or the state government concerned.

Question. Mention any one risk of a e pharmacy. Answer.  E-pharmacy can lead to fraudulent practices such as sending wrong dosage.

Unseen Passage with questions and answers for Class 10

Read the passage carefully:

1 Happiness lies within the mind of an individual. No amount of external wealth may be helpful in making him happy. Our forefather’s had lifestyles based mainly on the concept of symbol ‘simple living and high thinking’. Excessive material wealth did not mean much to them.

2 The structural framework of our forefather’s families was different from those of ours. The bond of love which they shared cannot be easily found today. The family provided an emotional cushioning effect against tension and stress. However, in today’s nuclear family, we are detached from feelings towards our kith and kin. This may finally destroy the family psyche.

3 The joint family system provided a proper environment for the child to grow up. The values of respect, tolerance, responsibility and integrity were internalised in the child. In the long run, they became better human beings, compared to the in the present generation. Our forefather’s felt happiest, if their children became honest human beings. But today, we are happy only if we attain our coveted material ends and are ready to resort to any means to attend that end.

4 Our forefathers had a vision to make India the best. To attend their ambitions, they were ready to make all sorts of personal sacrifices. On the other hand, today people are ready to migrate to the west to enjoy a comfortable life. Often, they came successful in foreign lands. But in the process, they become alienated from their motherland. Also, distance from their ailing parents is wearing factor and keeps them perturbed. It is not easy for them to return, as their children will not be able to adjust to the Indian environment and way of life. Thus, this is a crisis and afraid of mind worse than there for fathers.

5 In our generation, tradition and modernity have intermingled to form a special system. We are happier than our forefathers in being able to lessen evils like the rigid caste system, untouchability and child marriage, but we have failed to totally eradicate them. In fact, electoral politics has made use of the caste system, through issuance of party tickets on the basis of caste, community and religion. On the basis of your understanding, choose the correct option: 

Question. How are our forefathers different from us? Answer.  They had lifestyles based mainly on the concept of ‘simple living and high thinking’. They had different structural framework of families. They made all source of personal sacrifices to attend their ambitions.

Question. what impact does a joint family system have on a child?  Answer.  It helps a child to become a better human being and internalizes the values of respect tolerance responsibility and integrity in a child.

Question. Based on your reading of the passage choose the incorrect statement from the following. (i) Our youngsters had a vision to make India the best. (ii) Happiness lies within the mind of an individual. (iii) The family provided and emotional questioning affect against tension and stress. (iv) Our for fathers had lifestyles based mainly on the concept of simple living and high thinking.

Question. Choose the option that correctly States the two meanings of ‘cushioning’ as used in the passage. (i) Sharpening (ii) Shielding (iii) Intensifying (iv) Softening (v) Irritating (i) A E (ii) BD (iii) CE (iv) AC

Question. The………………… provided a proper environment for the child to grow up (i) neighbourhood (ii) joint family system (iii) nuclear family system (iv) friends and relatives

Question. In our generation, tradition and modernity have intermingled to form a special system. Substitute the underlined word with the most appropriate option from the following. (i) Divided (ii) Blended (iii) Scattered (iv) Detached

Question. Who become alienated from their motherland? Answer.   Present generation

Question. For whom excessive material wealth did not mean much to them? Answer.  Our forefathers

Question. Select the option that makes the correct use of ‘sorts’ as used in the passage to fill in the blank space. (i) Policemen have to know all ………………of different things. (ii) The machine……………..it all out. (iii) The computer………………..the words into alphabetical order. (iv) He always…………………. the books according to the author’s name.

Question. What are the values internalised in the child? Answer.  Respect, tolerance and responsibility.

Unseen Passage for Class 10 with questions and answers

1. India is home to an estimated 10% of the global Snow Leopard population spread across five Himalayan states of Jammu and Kashmir, Himachal Pradesh, Uttarakhand, Sikkim and Arunachal Pradesh which is only 5% of the available global habitat for the animal.

2. Project Snow Leopard was introduced in India by the Project Snow Leopard Committee instituted by the Ministry of Environment and Forests, Government of India, in the year 2009 to safeguard and conserve India‘s unique natural heritage of high-altitude wildlife populations and their habitats by promoting conservation through participatory policies and actions.

3. Project Tiger and Project Elephant are prevalent in India with the former introduced in 1973 garnering worldwide attention.

4. The Snow Leopard usually occurs at an altitude of >2700m above sea level and is the apex predator in the Himalayan Region along with the Tibetan Wolf. The project is not only to protect the Snow Leopard, but as with Project Tiger, it is considered an umbrella species and with its protection, comes the protection of other species and the habitat which they are all a part of. Top 3 National Parks in India where the Snow Leopard can likely be seen Kibber Wildlife Sanctuary – Himachal Pradesh (ii) Ulley Valley – Ladakh (iii) Hemis National Park – Himachal Pradesh

Unseen Passage for Class 10

The reasons for introduction of Project Snow Leopard are as follows i) High altitudes of India > 3000m including the Himalayan & Trans-Himalayan biogeographic zones support a unique wildlife assemblage of global conservation. ii) There has been relatively less attention on the region from the viewpoint of wildlife conservation. iii) The region represents a vast rangeland system supporting important traditional pastoral economic and lifestyles. iv) The region provides essential ecosystem services and harbours river systems vital for the nation‘s food security. v) India has ratified international agreements promoting the conservation of high-altitude wildlife species such as the snow leopard. vi) The region is important for the country‘s national security as well as international relations. vii) The high-altitude wildlife in India today faces a variety of threats. viii) The existing high altitude protected areas in India require considerable strengthening. ix) Wildlife Management in the region needs to be made participatory.

Based on your understanding of the passage, answer ANY FIVE questions from the six given below.

Question. What is the purpose of the above passage/study? Answer.  The purpose of the passage is to show that snow leopards are vulnerable species and must be protected.

Question. What do you mean by the phrase- snow leopard is considered an umbrella species? Answer.  Umbrella species refers to such species whose protection will ensure the protection of other species. Hence, option 

Question. According to the above passage why high-altitude regions should be protected? Answer.  High altitude regions should be protected as they support unique wildlife of global significance.

Question.  According to the above passage what is integral to the nation‘s food security? Answer.  River systems are integral to the nation‘s food security.

Question. Which Indian states come under the Himalayan region? Answer.  Jammu and Kashmir, Himachal Pradesh, Uttarakhand, Sikkim and Arunachal Pradesh

Question. List out two important factors why the high-altitude region is important. Answer.  Important for the country‘s national security as well as international relations AND the high-altitude wildlife in India today faces a variety of threats.

Case based factual Passage for Class 10

1. Globalization is the way to open businesses, improve technological growth, economy, etc. at the international level for all countries. It is the way in which manufacturers and producers of the products or goods sell their products globally without any restriction. It provides huge profits to the businessmen as they get low cost labour in poor countries easily. It provides a big opportunity to the companies to deal with the worldwide market.

2. Globalization helps to consider the whole world as a single market. Traders are extending their areas of business by treating the world as a global village. Earlier till the 1990s, there was a restriction on importing certain products which were already manufactured in India like agricultural products, engineering goods, food items and toiletries. However, during the 1990s there was a pressure from the rich countries on the poor and developing countries to allow them to spread their businesses by opening their markets. In India the globalization and liberalization process was started in 1991.

3. After many years, globalization brought about a major revolution in the Indian market when multinational brands came to India and started delivering a wide range of quality products at cheap prices.Prices of good quality products came down because of the cutthroat competition in the market.

4. Globalization and liberalization of the businesses in India have flooded the market with quality foreign products but has affected the local Indian industries adversely to a great extent resulting in the job loss to poor and uneducated workers. Globalization has been a bonanza for the consumers, however, a loss to the small-scale Indian producers. Globalization has had some very positive effects on the Indian consumer in all sectors of society. It has affected the Indian students and education sector to a great extent by making study books and a lot of information available over the internet. Collaboration of foreign universities with the Indian universities has brought about a huge change in the field of education.

5. Globalization of trade in the agricultural sector has brought varieties of quality seeds which have disease resistance property. However, it is not good for poor Indian farmers because the seeds and agricultural technologies are costly. It has brought about a huge revolution in the employment sector by the spread of businesses like cottage, handloom, carpet, artisan carving, ceramic, jewellery, and glassware etc.

Question. What is globalization? Answer.  Globalization is a way to open businesses at international level, which improves technological growth and economy at the international level.

Question. Write any two advantages of globalization? Answer.  The two advantages of globalization are: a) It provides huge profits to businesses as they get low-cost labour in poor countries. b) It provides big opportunities to companies to deal with the worldwide market.

Question. What is the effect of multinational brands entering the Indian market? Answer.  Multinational brands started delivering a wide range of quality products at cheap prices in the Indian markets

Question. How are the prices of quality products affected due to globalization? Answer.  Due to globalization, prices have come down and products have become cheaper.

Question. Find out the synonym of benefit‘ from paragraph number 4. Answer.  BONANZA

Question. Why has globalization had a negative effect on the poor Indian farmer? Answer.  Globalization had a negative effect on the poor Indian farmers because of the expensive seeds and agricultural technology.

Unseen Passage for Class 10 with answers pdf

1. Every form of human activity upsets or changes the wildlife complex of the area unceasingly and unavoidably. Man has destroyed many forms of wildlife for no reasonable purpose. Small sections of the community, for their own narrow, selfish ends, have destroyed many things of general interest.Expediency has often led man to make grave blunders in land use, habitat destruction, and the extermination of many forms of wildlife.

2. In his everyday life, man‘s attitude is determined in the main by purely practical considerations; ethical or moral considerations come afterwards. Looked at in this way, the disappearance from Britain of such animals as the wolf and wild boar can be more easily understood. In our intensively cultivated and over– populated country there was no room for such large mammals, the one a predator of big livestock and the other a pest to agriculture. Thus, man‘s first attitude to animals is the result of their effect on his own survival, or what he considers to be their effect on his survival.

3. Then there is his concern with sport. The animals he sets aside for this purpose are given special protection and war is waged unceasingly on any other creatures that may be a danger to them. This creates many problems and man has made serious errors in his destruction of predators. Until recent years all hawks and falcons were destroyed as ―vermin‖ by game preservers. This meant the destruction of kestrels, which are useful to the farmer: it meant the destruction of owls, which are useful to the farmer; so here you had sport acting against the interests of food production. agedy of all this is that all the killing of predators did not in any way improve man‘s sport. It has been clearly shown by modern research that eagles, hawks, falcons and predatory mammals have not the slightest effect on the numbers of game birds anywhere.

4. Broadly speaking, man wages war against the creatures which he considers harmful, even when his warfare makes little or no difference to the numbers of his enemies. And he encourages those creatures which are useful, even though their attacks on pests make little difference to the numbers of those pests. It would be true to say, therefore, that our attitude to song–birds, to most birds of prey and to many of our predatory animals, arises from the fact that they have either been proved useful or of no consequence. Either way, from this, we have developed the idea of conservation which means preserving what we have left of our heritage of wildlife and even finding room for rarities which may do a little damage on the side.

Question. What has been man‘s attitude towards wildlife? Answer.  Man is selfish and self–centred. He has no morals and ethics. For him it is practical aspects first and then other things. He can kill any animal, over–cultivate crops, cut forests and all this for his survival.

Question. How is the justification given for the killing of wolves and wild boar in Britain? Answer.  Britain, being intensely cultivated and over–populated, destroyed its wolf and wild boar population because of man‘s preoccupation with preserving only those animals that had practical implications therefore, the wolf, a predator of livestock and the boar, a pest to agriculture, were destroyed.

Question. Why has man been killing predators? Answer.  Man has been killing predators because he thinks that they are a danger to the birds useful in sports. But killing these animals has had no effect on the number of game birds. In nature it is a cycle. If man disturbs this cycle everything gets disturbed.

Question. In the last paragraph the writer talks about contradictory opinions. Explain. Answer.  Man kills those animals which he thinks are harmful and protects those which he thinks are worth preserving. But in all this he forgets that it is his idea of conservation and not nature‘s.

Question. Which two large mammals are mentioned in above paragraph which taking more place? Answer.  The wolf, a predator of livestock and the boar, a pest to agriculture, were destroyed.

Question. In para 4, What is the antonym of endangering‘? Answer.  Preserving

1. Reading Comprehension (RC, as it is normally called) is the most peculiar section in almost all scholastic, entrance and employment tests. The skills in RC make a lot of difference to one‘s chances of good grades/selection.

2. Most students find it difficult to tackle topics that are diverse from the field they are in or they are comfortable with. So one needs to develop a taste for even the most obscure and boring topic on this planet. For success in RC one should be able to understand. And even with an average speed one can succeed if one implements the strategies.

3. Broadly speaking, RC passages can be classified in a few categories. Fact based RC is the simplest form of RC. These types of passages have lot of information in the form of names, numbers etc. In this type of passages, one should read very fast.

4. Don‘t try to memorize any facts, numbers or names etc. In fact there is no need to even remember them. Just make yourself familiar with the structure of the passage. Just see in which paragraph author is talking about what. Mark it. Then when you go to the questions, identify in which paragraph information regarding that question is mentioned. Go to that paragraph, read the numbers, names etc. and mark the answer.

5. Inference based RC is the toughest form of RC. Here the passage is fairly tough to understand. This includes passages on topics like Religion, Spirituality, Philosophy, etc. Most of the students will be comfortable attempting these passages at least in RC. The reading speed is fairly slow in this type of passages. The way to master this type of passages is to read them again and again while practising.

6. Topic based RC includes passages on any particular topic like economics, astrology, medical science, etc. Generally what makes -these passages tough is usage of technical terms. If a topic is new to us then presence of technical term scares us even if they are defined in the passage. For success in this type of passages we need to have a fan-understanding of the definition of the term if it is defined in the passage. Read that definition twice if you need to. But don‘t worry about technical terms if they are not defined in the passage. Assume them to be non-existent and proceed. Key principle in these passages is that don‘t go to the next line unless the previous line is clear.

7. Reading passage first and then questions is the most popular strategy for RC. While answering the question you may come back to the passage to find answer as you have just read the passage initially and not crammed it. But you should not come back for each and every question. If you come back for majority of questions then you haven‘t read the passage properly. The key to success for this strategy is that you should understand the passage very well. We will suggest students to follow this technique from the beginning and work upon this.

8. Reading questions first and then passage is the strategy followed by a few students. They just look at the questions and not options. The objective is that after seeing the questions when you read the passage then you read only that part carefully where the answer is given. The flaw with this is that you will not be able to remember all the questions. Besides this, this strategy fails when there are questions that require understanding of the passage.

Question. What should one do for finding the right answers? Answer.  One should mark what the author is talking about in para.

Question. Why is topic-based R C tough? Answer.  Topic based R C is tough as it contains technical terms.

Question. Which step is considered more essential for finding right answers? Why? Answer.  Careful reading of the passage is essential for proper understanding and answering correctly.

Question. What is the most popular strategy for solving Reading Comprehension (RC)? Answer.  Reading passage first and then question is the most popular strategy for solving R C.

Question. Why do most students find Reading Comprehension difficult? Answer.  Because the topics are unrelated to their interest.

(f) Find the words similar in the meaning from passage. (i) Different (para 2) (ii) Accept as true (para 6) Answer.  (i) Diverse (ii) Assume

Unseen passage for class 10 with answers pdf

1. Read the passage given below and answer the questions that follow : 

1 Once a man saw three masons along with some labourers constructing a temple. He observed the masons for three days and found that though the three of them were doing the same kind of work, there was a marked difference in their approach to their job.

2 He saw that the first mason reported for his work late, did his work halfheartedly and sluggishly, enjoyed a longer respite, frequently checked the time on his wristwatch and left the work before time.

3 The second mason was very punctual in arriving and leaving, and did his work methodically. The third mason, however, would come before time, take little rest in the interval and often worked overtime.

4 The man naturally got curious and wanted to know the three masons’ outlook on their work. He asked them what they were doing. The first mason tapped his big belly with his hand and said, ‘‘I am earning food for myself.’’ The second said, ‘‘I am constructing a building.’’ The third mason looked at the huge building and said, ‘‘I am building the house of God.’’

1.1 On the basis of your understanding of the above passage, answer any eight of the following questions :

Question. What were the three masons doing ? Answer. constructing a temple

Question. For how long did ‘a man’ observe them ? Answer.  three days

Question. What made the man curious ? Answer.  the different approach of the three masons towards their job

Question. How did the first mason do his job ? Answer.  reported late/ did his work half-heartedly and sluggishly/ enjoyed a longer respite/ left the work before time

Question. What was the second mason’s approach towards his job ? Answer.  punctual in arriving and leaving/ did his work methodically

Question. How did the third mason view his job ? Answer.  as if he is building the house of God

Question. With whom among these can you associate the phrase ‘work is worship’ ? Answer.  the third mason

Question. Which word in the passage means ‘working lazily’ ? (para 2) Answer.  sluggishly

Question. Which word in the passage means the opposite of ‘carelessly’ ? (para 3) Answer.  methodically

2. Read the passage given below and answer the questions that follow : 

1 An epidemic of obesity or being overweight is affecting many countries in the world; more than 300 million people are obese or overweight. Obesity could be for a particular reason. In India, some people tend to be overweight because of their high calorie diet and lack of physical activities. In the United States, obesity is more prevalent in lower income groups. Obesity is now well recognised as a disease in its own right. Though obesity commonly means being overweight, it is defined as an excess amount of body weight that includes muscle, bone, fat and water. ‘Obesity’ specifically refers to an excess amount of body fat.

2 A certain amount of body fat is needed to store energy, keep warm and absorb the shocks. Usually men with more than 25% body fat and women with more than 35% body fat are regarded as obese. Obesity tends to run in families suggesting a genetic cause. Environmental factors include lifestyle behaviour such as what a person eats and his or her level of physical activity. So, one should choose more nutritious food which is low in fat, and become more active.

3 Then, there are psychological factors. Negative emotions such as boredom, sadness and anger are the main culprits. Then there is also ‘binge eating’ i.e. when people eat large amounts of food thinking that it is beyond their control how much they eat. Those with the most severe binge eating problem are also likely to have more symptoms of depression and low self-esteem.

4 Obesity is a health hazard giving rise to many serious medical conditions like type 2 diabetes, heart disease, high blood pressure and stroke. Obesity is also linked to a higher rate of certain types of cancer. There are many ways of losing weight but exercise is the best as it is free from any type of harmful side effects.

2.1 On the basis of your understanding of the above passage, answer any four of the following questions in 30 – 40 words each :

Question. What are the main reasons for obesity in India ? Answer. high calorie diet, lack of physical activity/exercise.

Question. Why do we need body fat ? Answer. to store energy, keep warm, absorb the shocks

Question. What are the symptoms of people with ‘binge eating problem’ ? Answer. depression, low self-esteem

Question. Which are the most common negative emotions which lead to overeating ? Answer. boredom, sadness, anger

Question. Write any four medical problems that can arise due to obesity. Answer.  type 2 diabetes, heart disease, high blood pressure, stroke, cancer

2.2 On the basis of your understanding of the above passage, answer any four of the following with the help of the given options :

Question. Women having _ body fat are regarded as obese. (i) more than 35% (ii) more than 25% (iii) less than 30% (iv) less than 25%

Question. Obesity can be defined as a/an _ amount of body weight. (i) less (ii) excess (iii) optimum (iv) balanced

Question. In the United States obesity is more prevalent among __ people. (i) high income (ii) homeless (iii) low income (iv) affluent

Question. Environmental factor refers to __ behaviour. (i) indecent (ii) decent (iii) lifestyle (iv) loud

Question. Binge eating is when people eat __ of food. (i) a meagre amount (ii) a small portion (iii) extra organic type (iv) a large portion

Discursive Passage for Class 10

3. Read the passage given below and answer the questions that follow : 

1 Cataract is the major cause of blindness, which is also caused by a damage to the cornea. It occurs more often in old age. As one starts growing old, the lens of the eye hardens, loses it transparency and becomes opaque. It obstructs the light rays from entering the eye.

2 The onset of cataract blurs the vision. Sometimes, the cataract patient sees multiple images instead of a single object image. Because of the gradual development of cataract, the afflicted person loses his/her vision and the world becomes dark to him/her.

3 The development of cataract is a complex process. However, the following factors can be attributed to its formation. Cataract generally develops in old age but sometimes, children are born with cataract because of hereditary defect. Eye injuries too, can cause cataract.

4 People exposed to sunrays for longer periods develop cataract earlier than others. Researchers opine that the smoke inhaled while smoking, carries substances internally damaging the eyes.

5 Ultraviolet radiation, invisible to the human eye, is linked to skin cancer. The victim loses vision and the world becomes dark to him.

Question. What is the major cause of blindness ? Answer. cataract

Question. When does cataract generally occur ? Answer. old age

Question. Why does the lens of the eye become opaque in old age ? Answer. lens of the eye hardens and loses its transparency.

Question. How does one detect cataract in early stages ? Answer. vision becomes blurred / sees multiple images in place of one.(any one

Question. Give any two factors which are responsible for the formation of cataract. Answer. eye injury / long exposure of sun rays / hereditary factors (any two)

Question. How does cataract affect its victim ultimately ? Answer. may lead to complete loss of vision

Question. Which type of cancer is caused by ultraviolet radiation ? Answer. skin cancer

Question. How is smoking responsible for the development of cataract ? Answer. smoke when inhaled carries a substance that damages eye and causes internal harm to the eye.

Question. Find the word from para 1 that means ‘that through which light cannot enter’. Answer. opaque

Unseen Passage with multiple choice questions for Class 10

4. Read the passage given below and answer the questions that follow : 

1 Spread over an area of 11,570 acres, Kalesar Reserve Forest is the only one of its kind in Haryana. It is home to a wide range of wild flora and fauna including three elephants, a male tiger, 16 male and female leopards, 19 panthers and some other animals as well.

2 Spread on both sides of Yamunanagar – Paonta Sahib Road in Yamunanagar district, it was designated a national park through a government notification in December 2003. However, it is alleged, lack of sufficient funds is proving to be a hindrance in wildlife conservation in the national park. The Divisional Forest Officer revealed that it is the only forest in Haryana with a natural ‘sal’ tree belt.

3 As for the steps taken to ensure protection of wildlife and environment, it was disclosed that staff had been given weapons by the State Government to tackle the menace posed by poachers. The State had also set up two special environmental courts, one each at Kurukshetra and Faridabad, to deal with crimes related to poaching and illicit felling of trees from the area. Eight watering holes have been dug up across the forest area to ensure that wildlife does not stray into human habitats on its fringes in search of drinking water. Similarly, earthen dams have been constructed to conserve rainwater for the future use of wildlife.

4 It was disclosed that work to fence the area would be undertaken in the near future and more steps taken to ensure that wild fauna does not face drinking water problem in their natural habitat in the reserve forest.

Question. What is unique about Kalesar Reserve Forest ? Answer. wide range of animals, flora and fauna

Question. Which animals are found in Kalesar forest ? Answer. elephants, panthers, tigers, leopards

Question. What is proving to be a hindrance to conserving wildlife in the national park ? Answer. lack of sufficient funds / poaching of animals / illicit felling of trees (any one)

Question. What is done in the park for the conservation of water ? Answer. 8 watering holes dug up across forest, earthen dams constructed.

Question. Why have two special environmental courts been set up ? Answer. to deal with crimes related to poaching and illicit felling of trees

2.2 On the basis of your understanding of the above passage, answer any four of the following with the help of the given options:

Question. Kalesar Reserve Forest was designated a national park through a government notification in __ . (i) February, 2003 (ii) December, 2003 (iii) March, 2004 (iv) August, 2001

Question. __ watering holes have been dug up across the forest area to stop wildlife from going astray in search of water. (i) Four (ii) Seven (iii) Eight (iv) Ten

Question. Which word in the passage means ‘natural environment in which a particular species live’ ? (i) Habitat (ii) Flora (iii) Fauna (iv) Waterhole

Question. Which word in para 3 is a synonym of ‘danger/risk’ ? (i) Disclosed (ii) Fringes (iii) Poaching (iv) Menace

Question. The word ‘illicit’ means (i) not approved by society (ii) not allowed by rules (iii) obtained dishonestly or unfairly (iv) not allowed by law

5. Read the passage given below and answer the questions that follow : 

1 Choice of profession and career is of great importance. Success depends on the right and timely choice of a career. There are a great variety of careers and professions. The lack of proper counselling worsens the situation. It also results in unemployment and frustration which means a lot of waste of money, time and energy. The students should take the help of elders, teachers, and professionals in deciding their careers. They should choose their career according to their aptitude and resources.

2 Young people are ambitious, impatient and eager to achieve good positions. They want to become high officials, executives and have top positions without considering their qualifications, skills etc. This is a wrong attitude. One should be realistic. One may dream of becoming a super film star and end up as a mere casual artist. Such high ambitions lead to disillusionment and unhappiness.

3 It is better to choose a career at the high school level than go for higher studies aimlessly. One should go for a vocational course at this stage. There are many careers, professions and vocations to choose from. There are teaching, engineering, medicine, law, technology, business, trade, computer application career, government employment, positions in the police and army etc. But each of these requires special aptitude, training, and resources. One may have proper qualifications and aptitude for a career of a doctor or engineer. But if there are no financial resources to train for it, one cannot choose it as one’s career.

4 Choice of a career is not simply a matter of likes and dislikes. Financial resources play a very important role in deciding to choose a profession. For lack of funds, one will have to choose a far humbler career. One can take to politics if one has an aptitude for public life. It is a profession of hard labour and perseverance. If you have strong optimism, ambition for fame and are prepared to face the worst, you are best suited for politics. If you have enough money and resources and want to grow very rich, you can choose business as your career. Therefore, be practical, logical and reasonable in choosing your profession. This is a crucial choice.

1.1 On the basis of your understanding of the above passage, answer any four of the following questions in 30 – 40 words each :

Question. State the factors that can help young people decide their career. Answer. students can take help of elders, teachers and professionals in deciding their careers. They should choose their careers according to their aptitude and resources.

Question. What leads to disillusionment and unhappiness among young people ? Answer. wrong, unrealistic attitude, high ambitions

Question. What is the right time to make a career choice ? Why ? Answer. high school level else left groping aimlessly

Question. Why are financial resources important while choosing a career ? Answer. for lack of funds, one will have to choose a far humbler career

Question. What are the qualities needed to become a politician ? Answer. strong optimism, ambition for fame, prepared to face the worst, has perseverance and to be ready for hard labour (any two)

1.2 On the basis of your understanding of the above passage, fill in any two of the following blanks with appropriate words/phrases :

Question. One’s success in life depends on . Answer.  right and timely choice of a career

Question. Lack of proper counselling leads to . Answer.  unemployment and frustration

Question.  _  play an important role in one’s choice of profession. Answer.  elders, professionals and teachers

1.3 Find out words/phrases from the passage that mean the same as the following. Attempt any two.

Question. guidance (para 1) Answer.  counselling

Question. aspire (para 2) Answer.  dream / ambition

Question. funds (para 3) Answer.  financial resources

Short Unseen Passage Class 10 with questions and answers

6. Read the passage given below and answer the questions that follow : 

1 The time Uncle Ken and I went for a walk and he insisted on going the zigzag way, I did the same. It was fun. But like all things to do with Uncle Ken, it landed us in a situation where we had to make a hasty exit. While we were being chased away by an irate man from his fields and were dashing away, Uncle Ken’s hat flew off. Later, when we had stopped panting, I remembered what Granny had said about looking at the funny side of situations and had a good laugh. Unfortunately, I don’t think Uncle Ken saw the lighter side at that moment.

2 It helps to have such an outlook on finding oneself in the oddest of circumstances. Like the time I was in Mumbai and my belt decided to have some fun by getting entangled with a fellow passenger’s luggage. After that, I sat quietly at the book festival I had been taken to attend, to keep my belt out of trouble. And it’s not just belts that get me into trouble. There was the time I was sharing a taxi to Delhi with a gentleman with a fierce moustache. He was carrying a book written by me with him. When I told him it was my book, he looked offended that I was trying to lay a claim to his property. I gave up on the conversation quickly and filed it away for future use in a story.

3 I have even been accused by some of my younger audience for being too funny. Once a girl asked me disapprovingly why my ghost stories are not scary enough. ‘‘Your ghosts are too funny,’’ she grumbled. So, I tried to write a few scary ghost stories. But I find benign, friendly ghosts just as delightful. So, I also wrote about my friend Jimmy, who discovered he was a jinn. That meant he could elongate his arms at will. This ensured that he had a starring role to play in the school basketball team. Mostly, I feel I have been able to live this life enjoying its small moments of happiness and been able to laugh when needed.

2.1 On the basis of your understanding of the above passage, answer any eight of the following questions :

Question. While on a walk, what did Uncle Ken insist on ? Answer. insisted on going zigzag way

Question. What happened when Uncle Ken was chased away from the field of an irate man ? Answer. he dashed out of the field, was panting and his hat flew off

Question. How did Uncle Ken react to his hat flying off ? Answer. he did not see the lighter / funny side of the incident

Question. What was the oddest situation that the narrator had faced in Mumbai ? Answer. his belt got entangled with fellow passenger’s luggage

Question. ‘I was trying to lay claim to his property.’ What is the ‘property’ referred to in this sentence ? Answer. the book that the gentleman was reading was written by the narrator himself

Question. What did the girl accuse the narrator of ? Answer. that his ghosts were funny and not scary

Question. Why did Jimmy have a starring role to play in the school basketball team ? Answer. because he could elongate his arms at will

Question. What kind of life did the narrator have ? Answer. lived his life to the fullest, enjoying its small moments of happiness

Question. Which word in paragraph 3 means the same as ‘harmless’ ? Answer. benign

7. Read the passage given below and answer the questions that follow : 

1.Thornfield Hall was a large gentleman‟s house in the country near a town called Millcote. There after my sixteen-hour journey, I was welcomed by Mrs. Fairfax. She was a little old lady dressed in black who seemed glad to have someone else to talk to, apart from the servants. Although the house was dark and frightening with its big rooms full of heavy furniture, I was excited at being in a new place and looked forward to my new life there working for kind Mrs. Fairfax.

2. But I was surprised to discover on my first full day at Thornfield that Mrs. Fairfax was not in fact the owner as I had assumed but the housekeeper and that my new master was a Mr. Rochester who was often away from home. My pupil was a girl called Adèle, seven or eight years old, who was born in France and could hardly speak English. Luckily I had learnt French very well at Lowood and had no difficulty in communicating with young Adèle, a pretty, cheerful child. It appeared that Mr. Rochester, who had known Adèle and her mother very well had brought Adèle back to England to live with him after her mother had died. I taught her for several hours every day in the library, although it was not easy to make her concentrate on anything for long as she was clearly not used to the discipline of lessons.

3. One day I took the opportunity of asking Mrs. Fairfax a few questions about Mr. Rochester, as I was curious about him and the little housekeeper seemed happy to talk. „Is he liked by most people?‟ was my first question. „Oh yes, his family has always been respected here. They‟ve owned the land around here for years,‟ she replied. „But do you like him? What is his character like?‟

4. „I have always liked him, and I think he‟s a fair master to his servants. He‟s a little peculiar, perhaps. He‟s travelled a lot you know. I expect he‟s clever, but I can‟t tell, really.‟ „What do you mean peculiar?‟ I asked, interested. It‟s not easy to describe. You‟re never sure whether he‟s serious or joking. You don‟t really understand him, at least I don‟t. But that doesn‟t matter, he‟s a very good master.‟

1.1 On the basis of your understanding of the above passage, answer any eight of the questions that follow :

Question. Why was Mrs. Fairfax glad to receive the narrator? Answer. happy to get someone apart from servants to talk to

Question. Why was the narrator excited? Answer. new place; new life; Mrs. Fairfax, a kind woman (any two)

Question. What wrong assumption was made by her? Answer. had assumed that Mrs. Fairfax was the owner

Question. Why had she come to Thornfield Hall? Answer. to teach Adele

Question. How do we know that Mr. Rochester was a man of noble nature? Answer. had brought Adele to England from France after her mother died

Question. Why was it difficult to make Adèle study for long hours? Answer. Adele not used to the discipline of lessons

Question. Why was Mr. Rochester liked by most people? Answer. had owned the land here for years / respected by all other tenants and he was the landlord

Question. What was peculiar about the nature of Mr. Rochester? Answer. not sure whether he was serious or joking

Question. How does the narrator describe Adèle? Answer. seven or eight years old ; young pretty and cheerful

8. Read the passage given below and answer the questions that follow : 

1. People who smoke are being hounded out of all public places – offices, restaurants, educational institutions, buses, trains etc. Away from home now it is becoming nearly impossible to find a place to smoke. Growing restrictions and the fear of various diseases caused by tobacco are prompting many smokers to stop smoking.19 states in the U.S. have made laws against smoking. Taxes on tobacco products are being raised.

2. Smoking is banned at several places. Still the number of smokers is rising in India. More than 25 crore people in India are addicted to tobacco. And according to a WHO report around half of them will die of tobacco related diseases. Unfortunately most people in India still need a strict warning based on a test report before they decide to give up smoking. Patients too seek medical help only after they show serious symptoms of an illness. Some hospitals have set up centres to identify risk factors among the young so that they can take preventive measures like giving up smoking early in life.

3. Tobacco is the second biggest cause of death in the world. It is responsible for the death of five crore people each year all over the world. Half of the people who are addicted to tobacco will ultimately be killed by this addiction. Many people get rid of this habit on their own and those who are willing to, but can‟t, go to de addiction centres. These centres use counselling, nicotine replacement therapy and prescriptive drugs to deal with withdrawal problems. Quitting cold turkey – stopping the use of tobacco at once – works well but only for those who have control and determination to quit.

2.1 On the basis of your reading of the passage, answer any four of the following questions in 30 – 40 words each :

Question. For which two reasons do smokers give up smoking? Answer.  restrictions on smoking; fear of disease

Question. Name two steps taken in India against smoking. Answer.  higher taxes on tobacco products; prohibition at several designated places

Question. Why is the situation unfortunate in India? Answer.  don’t want to quit willingly- only a bad test report, doctor’s warning makes them give up

Question. How do people get rid of smoking? Answer.  on their own; go to de addiction centers

Question. What does „cold turkey‟ mean? Answer.  stopping use of tobacco at once

2.2 On the basis of your reading of the passage, fill in any two of the following blanks with appropriate words/phrase:

Question. In the U.S. states have passed laws against smoking. Answer. 19

Question. In India one can‟t smoke at a . Answer. public place

Question. Tobacco is the  _  biggest killer in the world. Answer. second

2.3 Choose the meanings of the given words/phrases from the options that follow each word/phrase: (any two)

Question. hounded out (i) driven out (ii) given out (iii) washed (iv) endangered

Question. restrictions (i) checks (ii) imitations (iii) failures (iv) defeats

Question. prompting (i) abusing (ii) initiating (iii) empowering (iv) encouraging

Unseen Passage for Class 10 with questions and answers pdf

9. Read the passage given below and answer the questions that follow : 

1. Necessity is indeed the mother of invention. When areas in and around Leh began to experience water shortages, life didn’t grind to a halt. Why? Because Chewang Norphel, a retired civil engineer in the Jammu and Kashmir government came up with the idea of artificial glaciers.

2. Ladakh, a cold desert at an altitude of 3,000-3,500 metres above sea level, has a low average annual rainfall rate of 50mm. Glaciers have always been the only source of water. Agriculture is completely dependent on glacier melt unlike the rest of river/monsoon-fed India. But over the years with increasing effects of climate change, rainfall and snowfall patterns have been changing, resulting in severe shortage and drought situations. Given the severe winter conditions, the window for farming is usually limited to one harvest season

3. It is located between the natural glacier above and the village below. The one closer to the village and lowest in altitude melts first, providing water during April/May, the crucial sowing season. Further layers of ice above melt with increasing temperature thus ensuring continuous supply to the fields. Thus, farmers have been able to manage two crops instead of one. It costs about Rs.1,50,000 and above to create one.

4. Fondly called the ‘glacier man’, Mr. Norphel has designed over 15 artificial glaciers in and around Leh since 1987. In recognition of his pioneering effort, he was conferred the Padma Shri by President Pranab Mukherjee, in 2015.

5. There are few basic steps followed in creating the artificial glacier River or stream water at higher altitude is diverted to a shaded area of the hill, facing north, where the winter sun is blocked by a ridge or a mountain range. At the start of Winter/November, the diverted water is made to flow onto sloping hill face through distribution channels. Stone embankments are built at regular intervals which impede the flow of water, making shallow pools and freeze, forming a cascade of ice along the slope. Ice formation continues for 3-4 months resulting in a large accumulation of ice which is referred to as an ‘artificial glacier’.

Question. Who was Chewang Norphel? Answer. a retired civil engineer

Question. What kind of land form is Ladakh? Answer. Cold Desert

Question. Why have Glaciers been the only source of water for Ladakh? Answer. Ladakh is a cold desert at 3000-3500 meters ,annual rainfall rate of 50mm

Question. Why has the pattern of snowfall and rainfall changed? Answer. Climate change

Question. How are Glaciers significant for irrigation? Answer. Glaciers provide water in the months of April/May which crucial for irrigation

Question. How do farmers manage to grow two crops instead of one? Answer. Glaciers and their layers close to the village melt and provide water

Question. In which year did President Pranab Mukherjee confer Padmashri to Mr. Norphel? Answer. 2015

Question. How many months does the ice formation continue for? Answer. 3-4months

Question. Find the word in the paragraph 5 which means the same as ‘gathering’? Answer. accumulation

10. Read the passage given below and answer the questions that follow:  

1. Have you ever failed at something so miserably that the thought of attempting to do it again was the last thing you wanted to do?

2. If your answer is yes, then you are ‘not a robot’. Unlike robots, we human beings have feelings, emotions, and dreams. We are all meant to grow and stretch despite our circumstances and our limitations. Flourishing and trying to make our dreams come true is great when life is going our way. But what happens when it’s not? What happens when you fail despite all of your hard work ? Do you stay down and accept the defeat or do you get up again and again until you are satisfied? If you have a tendency to persevere and keep going then you have what experts call, grit. 3. Falling down or failing is one of the most agonizing, embarrassing, and scariest human experiences. But it is also one of the most educational, empowering, and essential parts of living a successful and fulfilling life. Did you know that perseverance (grit) is one of the seven qualities that have been described as the keys to personal success and betterment in society? The other six are: curiosity, gratitude, optimism, self-control, social intelligence, and zest. Thomas Edison is a model for grit for trying 1,000 plus times to invent the light bulb. If you are reading this with the lights on in your room, you know well he succeeded. When asked why he kept going despite his hundreds of failures, he merely stated that what he had been not failures. They were hundreds of ways not to create a light bulb. This statement not only revealed his grit but also his optimism for looking at the bright side.

4.Grit can be learned to help you become more successful. One of the techniques that help is mindfulness. Mindfulness is a practice that helps the individual stay in the moment by bringing awareness of his or her experience without judgement. This practice has been used to quiet the noise of their fears and doubts. Through this simple practice of mindfulness, individuals have the ability to stop the self-sabotaging downward spiral of hopelessness, despair, and frustration.

5. What did you do to overcome the negative and self-sabotaging feelings of failure? Reflect on what you did, and try to use those same powerful resources to help you today.

Question. According to the passage, what are the attributes of a human? Answer. feelings, emotions, and dreams; meant to grow ;flourishing and trying to make our dreams come true is great when life is going our way-(attributes)

Question. What is perceived as grit? Answer. If you have a tendency to persevere and keep going then you have what experts call grit.

Question. How is „failing‟ an educational and empowering part of human life? Answer. Failing teaches how to be perseverance and keep going for ones goals. Each failure makes one understand what to do and what not to do.

Question. In what ways can grit be developed? Answer. Grit can be developed by not accepting defeat / effort to overcome limitations / not to stay down.

Question. How does mindfulness help? Answer. Mindfulness is a practice that helps the individual stay in the moment by bringing awareness of his or her experience without judgement.

2.2 On the basis of your reading of the passage, fill in any four of the following blanks with appropriate words / phrases.

Question. While inventing the light bulb, Thomas Edison had failed____________________ i. 1000 times ii. 10000 plus times iii. 1000 plus times iv. 10000 times

Question. Failure is a part of __________________life. i. normal ii. common iii. human iv. ordinary

Question. In Paragraph 2,_____________________means continue. i. robots ii. keep going iii. agonizing iv. failing

Question. In paragraph 3, the synonym of distressing is____________________ i. embarrassing ii. scariest iii. agonizing iv. failing

Question. ______________helps in preventing individuals from going down the lines of despair. i. success ii. fear iii. doubt iv. mindfulness

11. Read the passage given below and answer the questions that follow: 

1. Cricket is a global passion, everywhere from Test match arenas to village greens, tropical beaches and dusty back lots. Cricket is the world‟s second popular spectator sport after football. 2. The Origin of cricket is somewhere in the Dark Ages. All research concedes that the game derived from a very old, widespread and uncomplicated pastime by which one player served up an object, be it a small piece of wood or a ball, and another hit it with a suitably fashioned club. Cricket was first recorded in 16th Century England, and it was played in grammar schools, farm communities and everywhere in between. But things really took off when 18th Century nobles realised it was a great sport.

3. The Oldest surviving set of cricket laws date from 1744 – printed on a handkerchief, naturally. It‟s now in the MCC Museum at Lord‟s in London. The oldest permanent fixture is the annual Eton v Harrow match, played since 1805. A young Lord Byron turned out for Harrow in the first match, though history doesn‟t record how poetic -or „mad, bad and dangerous‟ – his bowling was. 

4. The First international match was in 1877 when Australia beat England in Melbourne. The match was dubbed a „Test‟, since the gruelling nature of playing over five days was deemed the ultimate „Test‟ for any side. But it was Australia‟s first win on English soil in 1882 at the Oval in London – that led to matches between the two nations being christened the Ashes. Following the defeat, newspapers published an obituary mourning „The death of English Cricket‟, adding that „The body will cremated and the ashes taken to Australia‟.

5. A One Day International (ODI) is a form of limited overs cricket, played between two teams with international status, in which each team faces a fixed number of overs, usually 50. The Cricket World Cup is played in this format. The international One Day game is a late twentieth century development. The First ODI was played on 5 January 1971 between Australia and England at the Melbourne Cricket Ground.

Question. According to the passage, how did the game of Cricket originate? Answer. a Old, widespread and uncomplicated pastime by which one player served up an object, be it a small piece of word or a ball and another hit with a suitably fashioned club.

Question. Where can we find the oldest sets of Cricket laws? Answer. 1744 – printed on a handkerchief, naturally. It’s now in the MCC Museum at Lord’s in London

Question. Which match did Lord Byron play? Answer. Eton vs Harrow

Question. When was the first international match played? Answer. 1877

Question. Which countries played that first international match? Answer. England and Australia

Question. Why the matches between Australia and England were titled „The Ashes‟? Answer. Australia’s first win on English soil in 1882 at the oval in London led to matches between the two nations being christened the Ashes.

Question. In which format is the Cricket world cup played? Answer. ODI / limited over cricket / 50 over match

Question. When did the ODIs begin? Answer. 1971, 50

Question. Which word in paragraph no.5 means the same as „exhausting‟? Answer. gruelling

12. Read the passage given below and answer the questions that follow :

2. If your answer is yes, then you are ‘not a robot’. Unlike robots, we human beings have feelings, emotions, and dreams. We are all meant to grow and stretch despite our circumstances and our limitations. Flourishing and trying to make our dreams come true is great when life is going our way. But what happens when it’s not? What happens when you fail despite all of your hard work? Do you stay down and accept the defeat or do you get up again and again until you are satisfied? If you have a tendency to persevere and keep going then you have what experts call, grit.

3. Falling down or failing is one of the most agonizing, embarrassing, and scariest human experiences. But it is also one of the most educational, empowering, and essential parts of living a successful and fulfilling life. Did you know that perseverance (grit) is one of the seven qualities that have been described as the keys to personal success and betterment in society? The other six are: curiosity, gratitude, optimism, self-control, social intelligence, and zest. Thomas Edison is a model for grit for trying 1,000 plus times to invent the light bulb. If you are reading this with the lights on in your room, you know well he succeeded. When asked why he kept going despite his hundreds of failures, he merely stated that what he had been not failures. They were hundreds of ways not to create a light bulb. This statement not only revealed his grit but also his optimism for looking at the bright side

4. Grit can be learned to help you become more successful. One of the techniques that help is mindfulness. Mindfulness is a practice that helps the individual stay in the moment by bringing awareness of his or her experience without judgement. This practice has been used to quiet the noise of their fears and doubts. Through this simple practice of mindfulness, individuals have the ability to stop the self-sabotaging downward spiral of hopelessness, despair, and frustration. 5. What did you do to overcome the negative and self-sabotaging feelings of failure? Reflect on what you did, and try to use those same powerful resources to help you today.

Question. According to the passage, what are the attributes of a human? Answer. feelings, emotions, and dreams; meant to grow / flourishing and trying to make our dreams come true is great when life is going our way (attributes)

Question. What is perceived as grit? Answer. If you have a tendency to persevere and keep going then you have what experts call grit

Question. In what ways can grit be developed? Answer. It can be developed by not accepting defeat / effort to overcome limitations; not to stay down.

Question. How does mindfulness help? Answer.  Mindfulness is a practice that helps the individual stay in the moment by bringing awareness of his or her experience without judgement. This practice is a technique that many have used to quiet the noise of their fears and doubts

Comprehension Passages for Class 10

13. Read the passage given below and answer the questions that follow :

(1) Caged behind thick glass, the most famous dancer in the world can easily be missed in the National Museum, Delhi. The Dancing Girl of Mohenjo-Daro is that rare artefact that even school children are familiar with. Our school textbooks also communicate the wealth of our 5000 year heritage of art. You have to be alert to her existence there, amid terracotta animals to rediscover this bronze image. 

(2) Most of us have seen her only in photographs or sketches therefore the impact of actually holding her is magnified a million times over. One discovers that the dancing girl has no feet. She is small, a little over 10 cm tall – the length of a human palm – but she surprises us with the power of great art – the ability to communicate across centuries.

(3) A series of bangles – of shell or ivory or thin metal – clothe her left upper arm all the way down to her fingers. A necklace with three pendants bunched together and a few bangles above the elbow and wrist on the right hand display an almost modern art.

(4) She speaks of the undaunted ever hopeful human spirit. She reminds us that it is important to visit museums in our country to experience the impact that a work of art leaves on our senses, to find among all the riches one particular vision of beauty that speaks to us alone.

1.1 On the basis of your reading of the above passage answer the following questions.

Question. The dancing girl belongs to (i) Mohenjo-Daro (ii) Greek culture (iii) Homosapiens (iv) Tibet

Question. In the museum she’s kept among (i) dancing figures (ii) bronze statues (iii) terracotta animals (iv) books

Question. Which information is not given in the passage ? (i) The girl is caged behind glass. (ii) She is a rare artefact. (iii) School books communicate the wealth of our heritage. (iv) She cannot be rediscovered as she’s bronze.

Question. ‘Great Art’ has power because : (i) it appeals to us despite a passage of time. (ii) it is small and can be understood. (iii) it’s seen in pictures and sketches. (iv) it’s magnified a million times.

Question. The jewellery she wears : (i) consists of bangles of shell, ivory or thin metal. (ii) is a necklace with two pendants. (iii) both (i) and (ii) are correct. (iv) neither (i) nor (ii) is correct.

Question. She reminds us (i) of the never say-die attitude of humans. (ii) why museums in our country are exciting. (iii) why she will make us come into money. (iv) of dancing figures.

Question. The synonym of the word “among” in para 1 is __.

Question. The size of the dancing girl is equal to the length of human palm. (True/False)

Solved Unseen Passages for Class 10

14. Read the passage given below and answer the questions that follow :

(1) As the family finally sets off from home after many arguments there is a moment of lull as the car takes off. “Alright, so where are we going for dinner now?” asks the one at the driving wheel. What follows is a chaos as multiple voices make as many suggestions.

(2) By the time order is restored and a decision is arrived at, tempers have risen, feelings injured and there is at least one person grumbling. (3) Twenty years ago, you would step out of home, decision of meal and venue already made with no arguments or opposition and everybody looked forward to the meal with equal enthusiasm. The decision was made by the head of the family and the others fell in line. Today every member of the family has a say in every decision which also promotes a sense of togetherness and bonding.

(4) We empower our kids to take their own decisions from a very early age. We ask them the cuisine they prefer, the movie they want to see, the holiday they wish to go on and the subjects they wish to study.

(5) It’s a closely connected world out there where children consult and guide each other. A parent’s well meaning advice can sound like nothing more than unnecessary preaching. How then do we reach our children through all the conflicting views and make the voice of reason be heard ? Children today question choices and prefer to go with the flow.

(6) What then is the best path to take? I would say the most important thing one can do is to listen. Listen to your children and their silences. Ensure that you keep some time aside for them, insist that they share their stories with you. Step into their world. It is not as complicated as it sounds; just a daily half an hour of the quality time would do the trick.

2.1 On the basis of your reading of the above passage, answer the following questions in 30 – 40 words each:

Question. Write one advantage and one disadvantage of allowing every family member to be a part of the decision-making process. Answer. Advantage: every member of the family has a say in every decision / voice their opinion / promotes a sense of togetherness and bonding. (any one) Disadvantage: there is chaos / feelings injured / grumbling members / tempers rise (any one).

Question. In today’s world, what are parents asking their kids? Answer. to take decisions at a very early age / the cuisine they prefer / the movie that they want to see / the holiday they wish to go on / the subjects they wish to study (any two).

Question. Which two pieces of advice does the writer give to the parents? Answer. listen to children and their silences / ensure that you keep some time aside for them / insist that they share their stories with you /step into their world / spend at least half an hour of quality time with children (any two).

Question. The passage supports the parents. How far do you agree with the author’s views? Support your view with a reason. Answer. Agree with the author’s view. – parent’s well -meaning advice / empowering their kids to take their own decision (any other relevant point to be accepted) Disagree with the author’s views – children question choices and prefer to go with the flow/children consult and guide each other (any other relevant point to be accepted)

2.2 On the basis of your reading of the above passage, answer the following: Marking: – One mark each No penalty for spelling, punctuation or grammatical mistakes

Question. The synonym of ‘hurt’ as given in paragraph 2 is __ Answer. Injured

Question. The word which means the same as ‘a style or method of cooking’ in paragraph 4 is Answer. cuisine

Question. The antonym of ‘agreeable’ as given in paragraph 5 is __ Answer. Conflicting

Question. The antonym of ‘simple’ as given in paragraph 6 is Answer. complicated

Case based Unseen Passage for Class 10

15. Read the passage given below and answer the questions that follow :

1. Physical health affects the mind and vice-versa. Which of the two is more important is like asking the age-old question: what comes first, the chicken or the egg. For our present subject, let us be clear that a sound mind in a sound body are prerequisites for high efficiency and good quality work in the office, factory or field, which lead to a sense of achievement and fulfilment.

2. Our health is, of course, important not only to us, but also to our families and even to our employers. No wonder then that companies everywhere have a stake in the health and fitness of their employees and are willing to spend considerable money towards this purpose. Japanese companies particularly excel in this and the more successful among them start off the day with a body bending and stretching session and even singing the company song designed to promote loyalty and motivation among the workforce! This is considered an important factor contributing to high productivity and quality. Here is a lesson which has been emulated elsewhere in the world and could work wonders in India. It is estimated that some 20,000 American firms have established in-house health clubs. Typical of these is the soft drink manufacturer Pepsi Cola, which has spent about $2 million in setting up a health club at its head office in New York State. Such expenditure has proved to be a wise investment in achieving better efficiency and higher productivity. It has, indirectly, also helped reduce the premium on the health insurance of its staff. This business has grown so large that hundreds of consultancies have sprung up for the management of health clubs and fitness centres. In some cases, there is a nominal contribution by the employees on the premise they will value and use the facilities all the more. Some insurance companies have teamed up with employees in such ventures as better health contributes to a longer life and hence more premium! Though many firms in India have generous medical schemes, the movement towards health clubs and the like is yet to gain momentum.

1.1 Attempt any eight of the following questions on the basis of the passage you have read.

Question. What are the prerequisites for high efficiency and good quality work? Answer. A healthy body and healthy mind are the prerequisites for high efficiency and good quality work.

Question. How do Japanese companies ensure loyalty and motivation? Answer. Japanese companies give a lot of importance to the health fitness of their employees to ensure their employees to ensure their loyalty and motivation.

Question. What do you mean by ‘wise investment’? Answer. Spending a lot of money on an idea that turns out to be very successful.

Question. Why do companies have health clubs? Answer. Companies have health clubs to take care of the physical health of the employees.

Question. What does it mean by saying that “the idea of having health clubs in companies is yet to gain momentum”? Answer. It means that the idea of having health clubs in companies is not very popular in India.

Question. How do successful Japanese companies promote loyalty and motivation among the workforce? Answer. These companies start off the day with a body bending and stretching session and even singing the company song designed for this purpose.

Question. How has expenditure on employee’s health proved to be a wise investment? Answer. It has resulted in achieving better efficiency and higher productivity.

Question. Why have some insurance companies teamed up with employers? Answer. They have teamed up with employers as they understand that a better health contributes to a longer life and hence more premium.

Question. Why are the employers willing to spend money towards the fitness of their employees? Answer.  The employers willing to spend money towards the fitness of their employees to get good quality work from the employees.

Unseen Passage for Class 10 in English | Latest Unseen passage

Students can find different types of unseen passages for class 10 CBSE board exam preparation. At the end of every passage, we have also provided you with answers to unseen passage for class 10 given above.

So, first, solve the above-unseen passage for class 10 and compare your answer with their original answer in this way you can boost your performance. Now, You can easily obtain higher marks in the unseen passage for class 10.

if you take too much time in solving the unseen passage for class 10 take a clock to focus on how much time you are spending. By doing this, you can easily manage your time to solve the unseen passage for Class 10. You can also visit the unseen passage for class 10 in Hindi.

Don’t take stress, just focus on practicing unseen passage for class 10. You will definitely score high marks in your exam.

We believe that unseen passage for class 10 should reach every student who is aiming to score higher marks in the CBSE board exam.This unseen passage for class 10 prepared by our expert at unseenpassage.com

We give unseen passage for class 10 with answers pdf Download, Answers, and Marking Scheme. Here we have provided you with Class 10 English Unseen Passage to make your preparation better. A Student can view or download these Unseen Passage for Class 10 with answers pdf to score good marks in the Class 10 ​board exam.

Eventually, we trust that the article about the unseen passage for class 10 has ended up being extremely useful for you.

Presently, you can accomplish more practice by offering responses to these unseen passages.

On the off chance that you truly loved this article, kindly offer it with the people who need it.

Unseen Passage for Class 10 English with answers helps students to understand the questions asked in the board exam. We have also other study materials for Class 10 like Sample papers, Previous year Questions Paper, NCERT Solutions, NCERT Books, etc. If you have any problem in regard to Class 10 English Unseen Passge, write a comment in the box given below

Frequently Asked Questions-Unseen Passage for class 10(FAQ)

Answer: In the Exam, you will be given a small part of any story and you need to answer them to score good marks in your score. So firstly understand what question is being asked. Then, go to the passage and try to find the clue for your question. Read all the alternatives very carefully. Do not write the answer until you feel that you have selected the correct answer.

Answer: Do not try to write the answer without reading the passage Read all the alternatives very carefully, don’t write the answer until you feel that you have selected the correct answer. Check your all answers to avoid any mistakes.

Answer: Study the question before reading the passage. After that, read the passage and highlight the word which you find related to the question and a line before that word and one after that. With this strategy, you will be able to solve most questions and score higher marks in your exam.

Answer: A Seen passage is a passage that you have already read and know what is in it.While in the unseen passage, you are not familiar with the passage and don’t know what is in it.

Answer: Take a clock and set the time in which you should just complete all questions.If you can’t complete the passage in that time.don’t worry, find that part in which you take a long time to solve the question. By doing this, you can easily manage your time to solve the question of passage.

myCBSEguide

  •  Entrance Exam
  •  Competitive Exams
  •  ICSE & ISC
  •  Teacher Exams
  •  UP Board
  •  Uttarakhand Board
  •  Bihar Board
  •  Chhattisgarh Board
  •  Haryana Board
  •  Jharkhand Board
  •  MP Board
  •  Rajasthan Board
  •  Courses
  •  Test Generator
  •  Homework Help
  •  News & Updates

myCBSEguide

  •  Dashboard
  •   Mobile App (Android)
  •  Browse Courses
  •  New & Updates
  •  Join Us
  •  Login
  •  Register

No products in the cart.

  • English Language and Literature
  • Reading - Case Study Passage
  • CBSE Test Papers

CBSE - Class 10 - English Language and Literature - Reading - Case Study Passage - CBSE Test Papers

Cbse test papers for english language and literature reading - case study passage.

case study passage class 10 english

Student Subscription

Unlock the exclusive content designed for the toppers.

We will update content for this category shortly. Please visit this category after few days or subscribe to our newsletter by email for latest updates

myCBSEguide App

Trusted by 1 Crore+ Students

Test Generator

Test Generator

Create papers online. It's FREE .

CUET Mock Tests

CUET Mock Tests

75,000+ questions to practice only on myCBSEguide app

Google Play

Download myCBSEguide App

All courses.

  • Entrance Exams
  • Competative Exams
  • Teachers Exams
  • Uttrakand Board
  • Bihar Board
  • Chhattisgarh Board
  • Haryana Board
  • Jharkhand Board
  • Rajasthan Board

Other Websites

  • Examin8.com

CBSE Courses

  • CBSE Class 12
  • CBSE Class 11
  • CBSE Class 10
  • CBSE Class 09
  • CBSE Class 08
  • CBSE Class 07
  • CBSE Class 06
  • CBSE Class 05
  • CBSE Class 04
  • CBSE Class 03
  • CBSE Class 02
  • CBSE Class 01

CBSE Sample Papers

  • CBSE MCQ Tests
  • CBSE 10 Year Papers
  • CBSE Syllabus

NCERT Solutions

  • CBSE Revision Notes
  • Submit Your Papers
  • Terms of Service
  • Privacy Policy
  • NCERT Solutions for Class 12
  • NCERT Solutions for Class 11
  • NCERT Solutions for Class 10
  • NCERT Solutions for Class 09
  • NCERT Solutions for Class 08
  • NCERT Solutions for Class 07
  • NCERT Solutions for Class 06
  • NCERT Solutions for Class 05
  • NCERT Solutions for Class 04
  • NCERT Solutions for Class 03
  • CBSE Class 12 Sample Papers
  • CBSE Class 11 Sample Papers
  • CBSE Class 10 Sample Papers
  • CBSE Class 09 Sample Papers
  • CBSE Results | CBSE Datesheet

Please Wait..

IMAGES

  1. Discursive Passage Class 9/Class 10

    case study passage class 10 english

  2. Factual Passage Class 10 English

    case study passage class 10 english

  3. Case Based Factual Passage Class 10 English Term 1 CBSE| Reading Passage with Most Important Example

    case study passage class 10 english

  4. Discursive Comprehension Passages for Practice Grammar Class 9 and 10

    case study passage class 10 english

  5. Discursive Passage for Class 10

    case study passage class 10 english

  6. Term 2 Reading Skills:👉Case Based Factual Passage

    case study passage class 10 english

VIDEO

  1. Passage class 9, 10,11 and 12 board exam

  2. Errors and Passage Class-10

  3. Class 10

  4. UNSEEN PASSAGE SECRET TRICKS + EXAMPLES

  5. UNSEEN PASSAGE (TIPS &TRICKS) HOW TO SOLVE IT IN MINIMUM TIME ? CLASS 7 TO 12 CBSE

  6. Unseen passage class 10|Reading comprehension class 10|Unseen passage tricks|Class 10 English

COMMENTS

  1. CBSE Class 10 English Case Based Factual Passages with Answers for Term

    In CBSE Class 10 Term 2 Exam 2022, the English question paper will have a question based on a factual passage (unseen passage) of 5 marks. Students can easily score full marks in this question ...

  2. Unseen Passage English: 5 case study-based comprehension passage

    Here you will find our collection of case study-based comprehension passage English exercises, a new addition to the CBSE curriculum designed to improve your comprehension passage reading skills.. Unseen Passage English 1: Comprehension Passage Reading and Questions. 1. The UN's 2017 International Year tells that sustainable tourism is an important tool for development, most importantly in ...

  3. Case Based (Factual) Unseen Passages: Practice English Grammar for

    Case Based (Factual) Passages. 1. Read the passage given below: [CBSE SET 2, 2021-22] Milkha Singh, also known as The Flying Sikh, was an Indian track and field sprinter who was introduced to the sport while serving in the Indian Army. He is the only athlete to win gold in 400 metres at the Asian Games as well as the Commonwealth Games.

  4. Case Based Factual Passage for Class 10 2023 Exam

    Case Based Factual Passage for Class 10 mcq. On the basis of your reading of the passage given above, answer the following questions. (1) The purpose of this passage is to provide awareness regarding the _____. Choose the correct option. (a) tiny towns and cities in India. (b) small villages and hamlets in India.

  5. Cbse 10th English Case Study Questions

    10th Standard CBSE Subjects. Maths. Science. Social Science. QB365 Provides the updated CASE Study Questions for Class 10 , and also provide the detail solution for each and every case study questions . Case study questions are latest updated question pattern from NCERT, QB365 will helps to get more marks in Exams.

  6. CBSE 10th Case Study Questions 2021-22 : English Chapter-Wise Case

    CBSE 10th Case Study Questions 2021-22 : English Chapter-Wise Case Study Questions With Multiple Worksheets & Explanations. ... Given Below Are The Class 10th English Chapter Name Wih Respective Multiple Links Containing Case Study Worksheets & Solutions: 1. Factual Passages . Worksheets: Case Study Question: Explanation: Worksheet-1: Click ...

  7. Unseen Passage for Class 10 with Answer PDF

    1) Multiple Choice Questions based on a Discursive passage of 400-450 words to test inference, evaluation and vocabulary. Ten out of twelve questions are to be answered in English unseen passage for the Class 10 exam. (10×1=10) 2) Multiple Choice Questions based on a Case-based factual passage (with visual input statistical data, chart, etc ...

  8. Case Study Class 10 English Questions and Answers (Download PDF)

    Case Study Class 10 English: Here, you will get class 10 English case study questions and answers pdf at free of cost. ... These Case studies can help the students to solve the different types of questions that are based on the case study or passage. CBSE Board will be asking case study questions based on English subjects in the upcoming board ...

  9. CBSE Class 10 English Language and Literature Reading

    Qs 1-2. This section will have two unseen passages of a total length of 700-750. The arrangement within the reading section is as follows: Q. 1: A Factual passage of 300-350 words with eight very short answer type questions. (8 marks) Q. 2: A Discursive passage of 350-400 words with four short answer type questions to test inference,evaluation ...

  10. CBSE MCQ for Class 10 English Case Based Factual Passage Free PDF

    The CBSE MCQ for Class 10 English Case Based Factual Passage are provided below, in detailed and free to download PDF format. The solutions are latest, comprehensive, confidence inspiring, with easy to understand explanation. To download CBSE MCQ for Class 10 English Grammar PDF for Free, just click ' Download pdf ' button.

  11. [Case Based] Keep your watch accurate. For some people ...

    Answer. So, the correct answer is (D) This question asked in [CBSE Outside Delhi Set-3, 2019]Read the following passage carefully: (1) Keep your watch accurate. For some people, moving up the time on their watch will help them get up earlier. For others, they will remember that the time on the watch is wrong and will disregard it altoget.

  12. Case study questions for CBSE 10th

    Download case study question pdfs for CBSE Class 10th Maths, CBSE Class 10th English, CBSE Class 10th Sciece, CBSE Class 10th SST. As the CBSE 10th Term-1 Board Exams are approaching fast, you can use these worksheets for FREE for practice by students for the new case study formats for CBSE introduced this year. ... Factual Passages. 2. Section ...

  13. Unseen Passage for Class 10 English Reading Skills

    Unseen Passage for Class 10 English Reading Skills. Qs. 1-2. This section will have two unseen passages of a total length of 700-750 words. The arrangement within the reading section is as follows : Q. 1. A Factual passage 300-350 words with eight very short answer type questions. (8 marks) Q. 2. A Discursive passage of 350-400 words with four ...

  14. Unseen Passage For Class 10 English With Answers

    Please refer to Unseen Passage for Class 10 with Answers provided below. Unseen passages and Comprehensions are really important in Class 10 English exams. Students are expected to read through the entire passage and then answer various MCQ questions. We have provided here the best collection of Class 10 Unseen Passages with solves questions ...

  15. Unseen Passage for Class 10 English with Answers

    Case based Unseen Passage for Class 10 Read the following passage carefully. More than 87,000 healthcare workers have been infected with Covid-19, with just six states — Maharashtra, Karnataka, Tamil Nadu, Delhi, West Bengal and Gujarat — accounting for three-fourths (around 74%) of the case burden and over 86% of the 573 deaths due to the ...

  16. Reading Comprehension Passages English Grammar CBSE/ICSE

    Two types of passages are given for comprehension in exam. (i) Case Based (Factual Passage ) (300-350 words): A factual passage composed information in a direct manner about a particular subject. These passages focus completely on details or facts. It may include instructions, a report or a description.

  17. CBSE Class 10 English Complete Study Guide

    CBSE Class 10 English Marking Scheme. (Total length of two passages to be 600-700 words) Multiple Choice Questions / Objective Type Questions, and Short Answer Questions (to be answered in 30-40 words) will be asked to assess comprehension, interpretation, analysis, inference, evaluation and vocabulary.

  18. Unseen Passage for Class 10 Factual CBSE With Answers

    Unseen Passage With Answers for Class 10 CBSE. Sample Unseen Passage for Class 10 CBSE - 1. Read the following passage carefully and answer the questions that follow. High-altitude climbing is still a very dangerous task in spite of the availability of oxygen masks and other protective equipment, which modern climbers take with them.

  19. Unseen Passage For Class 10 Factual CBSE With Answers

    5. Read the passage given below and answer the questions that follow. 1. Calcium is a mineral that is found in abundance in the human body. Besides the fact that it is one of the most abundant minerals in the body, it is also one of the most important minerals. 2.

  20. CBSE Case Based Passages for Class 9 and 10 Solved Examples ...

    Case Based Unseen Passage: 2 The Rise of Solar Cities. Case Based Unseen Passage: 3 The Digital Detox Movement. Case Based Unseen Passage: 4 The Revival of Traditional Crafts. Case Based Unseen Passage: 5 Urban Farming - A Green Revolution. Case Based Unseen Passage: 6 Rapid Urbanization and Industrialization.

  21. Unseen Passage for Class 10 in English

    Unseen Passage for Class 10 with questions and answers. 1. India is home to an estimated 10% of the global Snow Leopard population spread across five Himalayan states of Jammu and Kashmir, Himachal Pradesh, Uttarakhand, Sikkim and Arunachal Pradesh which is only 5% of the available global habitat for the animal. 2.

  22. CBSE

    Download CBSE Test Papers for CBSE Class 10 English Language and Literature Reading - Case Study Passage in PDF format. These cbse test papers are arranged subject-wise and topic-wise.